You are on page 1of 165

Questions for Test 2 (Virology module)

1. Which of the following is not a hemorrhagic fever?


Lassa fever
Ebola fever
Marburg fever
West Nile Fever
Crimean-Congo hemorrhagic fever

2. The ability of a virus to infect an organism is regulated by


The host species.
The type of cells.
The availability of an attachment site.
Cell factors necessary for viral replication.
All of these

3. Which of the following is not caused by an arbovirus?


Tick-borne encephalitis
Yellow fever
West Nile encephalitis
Middle East respiratory syndrome
Japanese encephalitis

4. Which of the following is an advantage of an inactivated vaccine?


Only 1 dose of vaccine is required to achieve adequate immunity
Low concentration of antigen causes them to be relatively inexpensive
Immunity is often long lived
They are stable & there are no problems with viral interference
All of these

5. Which of the following is true regarding Ribavirin?


Has very narrow spectrum of action
Synthetic nucleoside of guanosine
Blocks attachment of virions
Capable to interfere with viral proteases
All of these

6. Which treatment option is specifically activated by HSV-induced


thymidine kinase enzyme?
Saquinavir
Ritonavir
Acyclovir
Ibuprofen
Oseltamivir
7. One disadvantage of attenuated vaccines is:
Multiple doses are often required for efficacy
Possible reversion to virulence
Immunity produced is relatively short-lived
Cannot be given by a natural route
No local immunity is produced

8. Cancer viruses are classified into several virus families. The following
virus family contains a human cancer virus with an RNA genome:
Adenoviridae
Herpesviridae
Hepadnoviridae
Papillomaviridae
Flaviviridae

9. Viruses are contributing factors in:


50% of human cancers
20% of human cancers
40% of human cancers
70% of human cancers
All cases of human cancers

10. E6 proteins from human papillomaviruses induce cell transformation by:


Increasing the expression of p53
Inhibiting Rb-mediated apoptosis
Repressing the transcription of the inhibitory form of the Rb protein
Disrupting Rb-E2f complexes
Targeting the p53 protein for proteasomal degradation

11. Infectious particles that cause neurodegenerative diseases such as


Kuru and mad cow disease are called:
Coronaviruses
Prions
Retroviruses
Picornaviruses
Paramyxoviruses

12. A 50-year-old woman is being treated for shingles in a neurology unit.


What reactivated virus causes this disease?
Herpes simplex virus type 1
Varicella zoster virus (chickenpox virus)
Herpes simplex virus type 2
Measles virus
Cytomegalovirus

13. An outbreak of an intestinal infection occurred in a kindergarten on the


eve of New Year holidays. Bacteriological examination of patients’
faeces didn’t reveal any pathogenic bacteria. Electron microscopy
revealed roundish structures with clear outer edges and a thick core
resembling a wheel. Specify the most likely causative agent of this
infection:
Adenovirus
Rotavirus
Coxsacki virus
E. coli
P. vulgaris

14. Adenoviruses, Poxviruses and Herpesviruses are:


Single-stranded DNA viruses
Single-stranded RNA viruses
Double-stranded RNA viruses
+Double-stranded DNA viruses
Mixed-stranded DNA viruses

15. A 17-year-old girl has made an appointment with the doctor. She plans to
begin her sex life. No signs of gynecological pathology were detected. In
the family history there was a case of cervical cancer that occurred to the
patient’s grandmother. The patient was consulted about the maintenance
of her reproductive health. What recommendation will be the most helpful
for prevention of invasive cervical cancer?
Vitamins, calcium, omega-3 C
Vaccination against human papillomavirus (HPV)
Immunomodulators
Antiviral and antibacterial drugs
Timely treatment of sexually transmitted diseases

16. A patient, who works as a milkmaid, has made an appoinment with a


dentist with complaints of aptha-shaped rash on the mucosa of oral cavity.
The doctor detected rash on her hands in the area of nail plates. What
agent causes this disease?
Herpesvirus
Foot-and-mouth disease virus
Cytomegalovirus
Vesicular stomatitis virus
Coxsackie B virus
17. A doctor examines a 17-year-old girl. The following is detected:
pharyngitis, cervical lymphadenopathy, fever. The preliminary diagnosis
is infectious mononucleosis. What method of investigation allows confirm
this diagnosis at the disease onset?
Microscopy of blood smear according to Giemsa method
Determining antibodies IgМ to Epstein-Barr virus
Determining antibodies IgG to Epstein-Barr virus
Sabin-Feldman dye test
Determining the amount of C-reactive protein

18. A 26-year-old man complains of chills, rhinitis, dry cough, and fever up to
38oC. Examination shows him to be in a moderately severe condition;
there are small pale pink non-merging spots on the skin of his back,
abdomen, and extremities. Palpation reveals enlarged occipital and
axillary lymph nodes. No information about vaccination history could be
obtained. What is the likely etiology of this disease?
Epstein-Barr virus
Rubella virus
Streptococcus
Mumps virus
Neisseria meningitis

19. A 45-year-old veterinary worker has made an appointment with the


doctor for regular examination. In his duties he frequently deals with
animals, however he denies working with rabies-affected animals.
Previously he has received no antirabic vaccination. What should the
doctor recommend in this case?
Vaccination in case of contact with sick animal
Preventive immunization with antirabic vaccine
Preventive immunization with rabies immunoglobulin
Administration of antirabic vaccine and rabies immunoglobulin
Preventive immunization with anti-rabies serum

20. A 50-year-old woman is being treated for shingles in a neurology unit.


What reactivated virus causes this disease?
Herpes simplex virus type 1
Herpes simplex virus type 2
Measles virus
Varicella zoster virus (chickenpox virus)
Cytomegalovirus
21. A medical student was hospitalized into the infectious diseases unit on the
2nd Day after the disease on set; the patient is suspected to have infectious
mononucleosis. What results of laboratory analysis can confirm this
diagnosis immediately on the day of the hospitalization?
IgM antibodies to herpes simplex virus were detected
IgM antibodies to Epstein-Barr virus were detected
Four fold increase in number of antibodies to Epstein-Barr virus was detected
Herpes virus was isolated
Cytomegalovirus antibodies were detected

22. A 35-year-old patient, a veterinarian, came to a dentist with complaints of


chills, fatigue, fever up to 38oC, muscle pain, sensations of dryness,
burning, and pain in the oral cavity, excessive salivation, vesicles in the
interdigital folds, on the lips, oral and nasal mucosa. On examination of
the oral cavity there were detected painful bright red erosions with
polycyclic contours against the background of inflammation. The
following was observed: scabs on the lips, enlarged tongue, impaired
speech, salivation up to 4 liters per day. Make the diagnosis:
Murrain
Acute herpetic stomatitis
Chickenpox D
Measles
Erythema multiforme

23. There was an episode of the hospital infection caused by S.aureus in the
surgical department. In this connection, the liquid polyvalent
staphylococcal bacteriophage was used. Name the purpose of preparation
using.
Specific prevention
Phagotyping
For indication of staphylococcus in reaction of phage title growth
Nonspecific prevention
For treatment of patients with chronic forms of staphylococcosis

24. The toxigenic culture of pathogen is isolated from patient S. with a


previous diagnosis “diphtheria”. It is known, that such activity is
connected with the diphtheria bacteria lysogenicity. Which factor does
determine the lysogenicity of bacteria?
Antibodies
Antibiotics
Phagocytes
Factors of autolysis
Temperate phages

25. Which vaccine is used for yellow fever prevention?


Chemical vaccine
Attenuated vaccine
Anatoxin
Inactivated vaccine
Gene engineering vaccine

26. A drugstore received a supply of a drug that is widely used for treatment
of many virus diseases since it is not virus-specific. What drug is it?
Remantadin
Interferon
Metisazone
Immunoglobulin
Vaccine

27. A patient with herpetic stomatitis was prescribed acyclovir for topical
application. What is its mechanism of action?
It inhibits virus penetration into cells
It inhibits synthesis of nucleic acids of virus
It inhibits virus maturation
It increases the resistance of macroorganism cells to the viruses
It inhibits virion assembly

28. A patient is ill with herpetic stomatitis provoked by immunosuppression.


Which preparation introduced intravenously, internally and locally can
provide antiviral and immunopotentiating effect?
Remantadinum
Acyclovir
Levamisole
Methisazonum
Amoxicillin

29. Because of suspected intrahospital infection in the neonatal department of


the maternity home the inspection was carried out. In some children and
on some general things Staphylococcus aureus was revealed. What
properties of these cultures allow to establish their origin from one
source?
Antibioticogramma
Antigenic structure
Biochemical activity
Phagotyping
Chromogenesis
30. From the fecal sample of a patient Shigella sonne were isolated. What
additional studies are required to identify the source of infection?
Antibiogram
+Phage-typing of the isolated pure culture
Precipitation reaction
Complementfixation reaction
Neutralization reaction

31. Examples of human viruses are West Nile virus, Rabies virus,
Hantaviruses and some other. The source of invasion of such viruses are
animals. Such human parasites cause diseases that are called:
Anthroponotic
Zoonotic
Anthropozoonotic
Infectious
Multifactorial

32. The following statements are true about viruses except:


Obligate intracellular parasites
Do not possess cellular organelle
Invisible with light microscope
Unicellular organisms comprising of either DNA or RNA
Sensitive to antibiotics

33. Epstein-Barr Virus (EBV) is associated with


Rhinitis
Burkitt's lymphoma
Common cold
Influenza
Warts

34. What shape most of bacteriophages have?


Enveloped
Complex
Helical
Spherical
None of this

35. Which capsid symmetry is exhibited by most of the phages?


Filamentous
Icosahedral
Helical
Pleomorphic
None of these

36. Type of life cycle when a bacteriophage integrates its own DNA into the
bacterial DNA and passes it to future generations is called:
Virulent
Lysogenic
Lambda
Lytic
Latent

37. Bacteriophages undergo replication by:


Binary fission and fragmentation
Lytic cycle and lysogenic cycle
Mitosis
Meiosis
None of these

38. The viral genome integrated to the bacterial genome is called:


Plasmid
Prophage
Capsid
Virion
Tail

39. What is NOT true about bacteriophages?


Each bacteriophage species is specific to a certain bacteria
There are two types of phage life cycles, lytic and lysogenic
All bacteriophages have heads and tails
During lysogenic life cycle virus exists in the form of prophage
Phages can reproduce only inside bacterial cell

40. How many phages there are on the Earth?


1021
1031
1029
1 billion
1 million
41. What structure do bacteriophages use to introduce their genome into the
host cell?
Head
Fibres
Tail
Envelope
Capsid

42. What method can be used to calculate titer of bacteriophages in sample?


Phage typing
Double-layer agar method
Hemagglutination test
ELISA
Immunofluorescence

43. During lytic cycle, phages undergo all these stages except:
Adsorption
Penetration
Maturation
Integration of viral DNA
Assembly

44. During lysogenic cycle, phages undergo a specific stage called:


Penetration
Integration
Attachment
Release
Replication

45. Bacteriophages can be used in medicine for:


Prevention and treatment of bacterial infections
Treatment of agricultural plants
Veterinary medicine purposes
Wastewater treatment
All of these

46. What is NOT true about phage therapy?


Bacteriophages are highly specific
Phages don’t affect human microflora
Production of phage drugs is less expensive than antibiotics
Phages therapy helps to cure viral infections
Phage therapy can be used in agriculture

47. What is transduction?


Releasing of new virions from bacterial cell
Transferring of bacterial DNA by bacteriophages
Method by which titer of phages can be calculated
Assembly of viruses
None of them

48. From the fecal sample of a patient Shigella sonnei were isolated. What
additional studies are required to identify the source of infection?
Antibiogram
Phage-typing of the isolated pure culture
Precipitation reaction
Complement-fixation reaction
Neutralization reaction

49. Process of transferring of bacterial DNA by bacteriophages is called:


Integration
Transduction
Attachment
Release
Replication

50. What method is not useful for bacteriophages?


Double-layer agar method
Electron microscopy
PCR
Hemagglutination
test Spot-test

51. Phages that can infect more than one bacterial species are called:
Lysogenic
Polyvalent
Lytic
Abnormal
None of these

52. Type of life cycle when phage infects bacterial cell and then lyse it:
Lysogenic
Polyvalent
Lytic
Normal
None of these

53. Which family from the list does not consist of bacteriophages?
Myoviridae
Siphoviridae
Podoviridae
Lipothrixviridae
Picornaviridae

54. How phage DNA enters the cell?


Through direct penetration of virus
Through injection
During mitosis
In the vesicles
None of these

55. What is the main genetic characteristic of the herpesvirus family?


Small ssDNA genome
Large linear DNA genome
Segmented DNA genome
Circular dsDNA genome
ds RNA

56. Which clinical presentation is typical of a primary first episode HSV-2


infection?
No lesions or symptoms, HSV-1 antibodies may or may not be present, HSV-2
antibodies are present
Lesions present, symptoms usually mild, HSV-1 antibodies may or may not be
present, HSV-2 antibodies are present
Lesions present, symptoms usually severe, HSV-1 and HSV-2 antibodies are not
present
Lesions present, symptoms usually moderate, HSV-1 antibodies are present, HSV-2
antibodies are not present
Abdominal pain or discomfort, especially on the upper right side beneath your lower
ribs.

57. What is the morphology of the herpes virion?


Baggy virion with over 50 types of spikes
Complex with a membrane and tegument and icosahedron core
Compact icosahedron structure
Small round virus
Flexuous and filamentous particles

58. What disease do herpes viruses not cause?


Encephalitis
Infantile paralysis
Sexually transmitted disease
Cancer
Gingivostomatitis

59. A 50-year-old woman is being treated for shingles in a neurology unit.


What reactivated virus causes this disease?
Measles virus
Varicella zoster virus (chickenpox virus)
Herpes simplex virus type 1
Herpes simplex virus type 2
Cytomegalovirus

60. There are 57 serotypes of adenovirus currently recognised. Which has the
most important clinical impact?
Diarrhoea
Eye infections
Respiratory tract infections
Haemorrhagic cystitis
Jaundice and abdominal discomfort

61. The adenovirus virion has which unique structural feature?


Icosahedron with slender fibres
Helical capsid
'Complex' structure not yet fully explored
Flexuous lipid containing structure
Flexuous and filamentous particles

62. Adenovirus infection is characterized mainly by the mucous membrane


lesions of:
nose
pharynx
larynx
trachea
bronchi

63. How many types of human herpesviruses are known:


8
9
5
6
11

64. Select zoonotic human herpesvirus


VZV
HSV-1
HSV-2
CMV
Herpes B virus

65. Which antiviral drug can be used in treatment of herpesvirus infections?


Ribavirin
Rimantadin
Sofusbuvir
Acyclovir
Amantadine

66. Vaccination is available for which herpesvirus?


KSHV
VZV
HSV-1
HSV-2
CMV

67. Where does VZV multiply during the primary infection?


Liver
Lungs
Gastrointestinal tract
Skin
Spine

68. Which herpesvirus can be vertically transmitted and is one of the


pathogens that cause congenital abnormalities?
KSHV
VZV
HSV-1
HSV-2
CMV

69. What diseases adenoviruses can cause?


Pharyngitis
Tonsillitis
Conjunctivitis
Adenoiditis
All of these

70. What is true about bacteriophages?


Always have lipid membrane
Are the most abundant biological entity on earth
Infect eukaryotic cells
Pathogenic for human
All of these

71. What is true about viral infections?


Can be treated with antibiotics
Are universal for all
organisms Always display
symptoms
All of these
None of these

72. What is human virome?


Causative agent of viral infection
The total collection of viruses in the human body
The total collection of viruses on the human body
The total collection of viruses in and on the human body
The total collection of viruses outside of the human body

73. What is true about human virome?


Viruses in the human body may infect only eukaryotic cells
Viruses in the human body may infect both human cells and other microbes
Viruses in the human body may infect only human cells
Viruses in the human body play only pathogenic role
None of these
74. Which of the following is the smallest?
Rhinovirus
Yellow fever
Adenovirus type 5
Herpes simplex virus
B19

75. Which viruses have oval or 'brick-shaped' particles


Adenoviruses
Retroviruses
Picornaviruses
Poxviruses
All of these

76. The largest genome is characteristic for


Adenoviruses
Retroviruses
Picornaviruses
Poxviruses
Viroids

77. Types of viral DNA are following


Linear single stranded
Linear double-stranded
Circular single- or double-stranded
Circular single- or double-stranded
All of these

78. Types of viral RNA are following


Linear single stranded positive
Linear single stranded negative
Linear single stranded segmented
Linear double-stranded segmented
All of these

79. Which viruses have segmented genomes?


Orthomyxo- and Paramyxoviruses
Orthomyxo- and Rotaviruses
Pox- and Adenoviruses
Parvo- and Adenoviruses
All of these
80. A prophage is an early stage in the development of a/an
Bacterial virus
Lytic virus
Poxvirus
Enveloped virus
All of these

81. The target cells for B19:


Nerve cells
Epithelial cells
T-lymphocytes
B-lymphocytes
Erythroid precursor cells

82. Which viruses are assembled in nucleus?


Picornaviruses
Bunyaviruses
Rhabdoviruses
Adenoviruses
All of these

83. Name infection which is caused by normally nonpathogenic organisms:


Acute
Persistent
Latent
Slow
Opportunistic

84. Select features of vertical transmission of viruses:


Virus travels from one host to another between two individuals, generally in the same
generation
Virus can be transmitted directly from a parent to members of the next generation
Direct contact can occur due to touching, biting or licking or sexual intercourse
Indirect contact can occur through vectors
All of these

85. Which DNA viruses cause a verity of syndromes including common colds,
acute pharyngitis, tonsillitis, conjunctivitis and gastroenteritis?
Herpesviruses
Poliomaviruses
Papilomaviruses
Adenoviruses
All of these

86. Which viruses are the most common cause of acute adult viral
gastroenteritis?
Adenoviruses
Noroviruses
Enteroviruses
Herpesviruses
Flaviviruses

87. Which viruses are the most common cause of acute children
gastroenteritis?
Adenoviruses
Rotaviruses
Enteroviruses
Herpesviruses
Flaviviruses

88. What member of herpesvirus group causes chickenpox?


HSV-1
HSV-2
VZV
CMV
All of these

89. Dengue and Zika viruses are representatives of:


Picornaviruses
Paramyxoviruses
Orthomyxoviruses
Retroviruses
Flaviviruses

90. Which virus causes painful vesicular lesions on various mucocutaneous


sites?
HIV
HSV
CMV
HPV
All of these

91. Many zoonotic viruses that capable to cause encephalitis or hemorrhagic


disease belong to group of:
Picornaviruses
Bunyaviruses
Herpesviruses
Paramyxoviruses
All of these

92. Name a family of viruses whose members are generally associated with
rodent-transmitted diseases in humans
Picornaviridae
Filoviridae
Herpesviridae
Hantaviridae
All of these

93. Causative agents of hemorrhagic fever with renal syndrome are:


Picornaviruses
Hantaviruses
Herpesviruses
Paramyxoviruses
All of these

94. The representatives of which virus family may appear in several shapes
Picornaviridae
Filoviridae
Herpesviridae
Arenaviridae
All of these

95. Middle East respiratory syndrome is caused by:


Hantavirus
Coronavirus
Rotavirus
Paramyxovirus
Filovirus

96. Severe Acute Respiratory Syndrome is caused by:


Hantavirus
Coronavirus
Ortomyxovirus
Paramyxovirus
Filovirus

97. The most distributed arbovirus in Europe is:


Dengue virus
Yellow fever virus
Tick-borne encephalitis virus
Japanese Encephalitis virus
Crimean-Congo hemorrhagic fever virus

98. What is true about Japanese Encephalitis?


JE is the leading cause of viral neurological disease in Asia
More than 3 billion people live in areas where JE is transmitted.
Up to 10,000 to 15,000 deaths are reported each year.
JE can be controlled using inactivated vaccines
All of these

99. Which animal species serves as a reservoir for herpes B virus?


Cattle
Sheep
Monkey
Horse
Dog

100. Select human polyomaviruses:


HIV and HTLV-1
BK-virus and JC-virus
HSV-1 and HSV-2
HCV and HEV
CMV and VZV
Telegram : @tarasmedicalstudents

Which of the following is not a hemorrhagic fever?


Lassa fever
Ebola fever
Marburg fever
+West Nile Fever
Crimean-Congo hemorrhagic fever

Which of the following diseases is caused by a retrovirus?


Lassa fever
+HIV
Influenza
Rabies
All of the above

Which of the following is an advantage of an inactivated vaccine?


Only 1 dose of vaccine is required to achieve adequate immunity
Low concentration of antigen causes them to be relatively inexpensive
Immunity is often long lived
+They are stable & there are no problems with viral interference
All of the above

Which virus do you associate with antigenic drift & shift?


+Influenza A virus
HIV
Poliovirus
Measles virus
All of the above

A virus with RNA-dependent RNA polymerases


Synthesizes DNA from an RNA template.
+Synthesizes RNA from an RNA template.
Synthesizes double-stranded RNA from a DNA template.
Transcribes mRNA from DNA.
None of the above

The ability of a virus to infect an organism is regulated


by The host species
The type of cells
The availability of an attachment site Cell
factors necessary for viral replication
+All of the above
Telegram : @tarasmedicalstudents

Which of the following are unique viral enzymes that are targeted
for treatment?
Interferons
+Reverse transcriptases
Kinases
Polyglycols
All of the above

Which of the following is true regarding Ribavirin?


+Synthetic nucleoside of guanosine
Has very narrow spectrum of action
Blocks attachment of virions
Capable to interfere with viral proteases
All of the above

Which treatment option is specifically activated by HSV-induced


thymidine kinase enzyme?
+Acyclovir
Saquinavir
Ritonavir
Ibuprofen
Oseltamivir

Many antiviral drugs act by inhibition of a DNA polymerase enzyme. Select


the virus for which this class of drugs would be effective
+Cytomegalovirus
Influenza
Measles
Mumps
Rabies

Who discovered the first virus?


The microscopist Antonie van
Leeuwenhoek The bacteriologist Charles
Chamberland +The botanist Dmitri
Iwanowsky The virologist Jonas Salk
The biologist, microbiologist and chemist Louis Pasteur

Protection against influenza A virus in a non-immune individual can be achieved


through the administration of a drug that interferes with:
Viral endonuclease activity
Binding of host messenger RNA (mRNA) caps by the viral P1 protein
Synthesis of viral progeny RNA
+Virus release from cell
Viral protease activity
Telegram : @tarasmedicalstudents

Attenuated vaccines tend to generate a strong immune response:


Never revert to virulence
+Because the virus replicates in the host
Only when administered in multiple
doses Only when administered
intramuscularly All of the above

One disadvantage of attenuated vaccines


is: Multiple doses are often required for
efficacy +Possible reversion to virulence
Immunity produced is relatively short-lived
Cannot be given by a natural route
No local immunity is produced

One disadvantage of inactivated vaccines is:


They are generally considered to be unstable
They stimulate interferon
+Immunity may be short lived
They are inexpensive to produce due to low concentrations of
antigen There are no disadvantage

Viruses belonging to which of the following groups are likely to establish latent
infections?
Poxviruses
Filoviruses
+Herpesviruses
Influenza viruses
Caliciviruses

Which of the following statements regarding Arboviruses is


true? Replicate only in humans
Replicate only in rodents
+Are transmitted by vectors
Cause respiratory infections
All of the above

What reason do scientists give for saying that viruses are not living organisms?
They cannot reproduce without the aid of a host
They do not metabolize food into energy
They do not have organized cells
+All of the above
None of the above

Which of the following is smallest?


Influenza virus
+Rhinovirus
Telegram : @tarasmedicalstudents

Adenovirus type 5
Smallpox virus
HIV

Which of the following is smallest?


HCV
HSV-1
Bacteriophage T4
+B19
HBV
Viruses are often named by:
Where they were first discovered
The scientist who discovered them
The animal that carries them
Disease they cause
+All of the above

HTLV-1/2 encodes an enzyme called reverse transcriptase. The function of the


reverse transcriptase enzyme is:
DNase activity
+RNA-dependent DNA polymerase activity
DRNA-dependent RNA polymerase activity
Topoisomerase activity
Proteinase activity

Cancer viruses are classified into several virus families. The following
virus family contains a human cancer virus with an RNA genome:
Adenoviridae
Herpesviridae
Hepandoviridae
Papillomaviridae
+Flaviviridae

Viruses are contributing factors in:


50% of human cancers
+20% of human cancers
40% of human cancers
70% of human cancers
All cases of human cancers

E6 proteins from human papillomaviruses induce cell transformation by:


Increasing the expression of p53
Inhibiting Rb-mediated apoptosis
Repressing the transcription of the inhibitory form of the Rb
protein Disrupting Rb-E2f complexes
Telegram : @tarasmedicalstudents

+Targeting the p53 protein for proteasomal degradation

Which of the following viruses have not been associated with human cancer?
Hepatitis B Virus
+Ebola virus
Adenovirus
Hepatitis C virus
Epstein-Barr virus

Edward Jenner began inoculating humans with material from __________


lesions against chickenpox:
Monkeypox
+Cowpox
Avianpox
Smallpox
Humanpox

The viruses in an attenuated vaccine


Have no genome
+Continue to replicate
Are usually larger than bacteria
Are altered with chemicals
Are altered with physical treatment

In cell culture, measles virus may lead


to Nuclear pyknosis
Transformation of cells
+Syncytium formation
Rounding and aggregation of cells
All of the above

Which of the following viruses belong to family Flaviviridae?


Dengue virus
Yellow fever virus
Hepatitis C virus
Zika virus
+All of these

Which of the following viruses show transformation of infected cells?


Hepatitis B virus
Hepatitis C virus
Human T cell lymphotropic virus type I
Epstein-Barr virus
+All of these

The size of viruses is usually measured


in Centimeters
Telegram : @tarasmedicalstudents

Micrometers
+Nanometers
Millimeters
None of the above

Enzyme neuraminidase is carried by which of the following


viruses? Human immunodeficiency virus
Epstein-Barr virus
+Influenza virus
Ebolavirus
Poliovirus

Which virus has the highest mortality rates?


+Sudan ebolavirus
Reston ebolavirus
Bombali ebolavirus
Dengue virus
Zika virus

Which virus has the highest mortality rates?


+Zair ebolavirus
Reston ebolavirus
Bombali ebolavirus
Dengue virus
Zika virus

What part of the structure of a virus would include


phospholipids? Nucleic acid
Capsid
+Envelope
Genetic material
Glycoprotein spikes

Infectious particles that cause neurodegenerative diseases such as Kuru


and mad cow disease are called:
Coronaviruses
+Prions
Retroviruses
Picornaviruses
Paramyxoviruses

During integration:
The virus attaches to a host cell receptor The
virus takes over the host cell machinery +The
viral DNA is inserted into the host DNA The
viral DNA enters the host cell All of the
above
Telegram : @tarasmedicalstudents

A disease, muscle twitching and lack of coordination, occurs due to a


prion, known as
Marasmus
+Kuru disease
Cancer
Tetanus
Influenza

A patient, who works as a milkmaid, has made an appointment with a dentist


with complaints of aptha-shaped rash on the mucosa of oral cavity. The
doctor detected rash on her hands in the area of nail plates. What agent
causes this disease?
Herpesvirus
+Foot-and-mouth disease virus
Cytomegalovirus
Vesicular stomatitis virus
Coxsackie B virus

Dentists are at increased risk of being infected with the type B hepatitis.
What preparation should be used for reliable active prevention of this
disease? Monoclonal HBsAg antibodies
Live type B hepatitis vaccine
Specific immunoglobulin
Antihepatitis serum
+Recombinant vaccine of HBsAg proteins

Enzyme-linked immunosorbent assay has detected HBs antigen in blood


serum. What disease is it characteristic of?
+Viral hepatitis type B
Viral hepatitis type A
AIDS
Tuberculosis
Syphilis

HIV has gp41 and gp 120 on its surface interacts with target cells of an
organism. Which of the following human lymphocyte antigens is gp
120 complementary bound with?
CD8
CD19
+CD4
CD28
CD3

The immunoblot detected gp120 protein in the blood serum. This protein
is typical for the following disease:
Tuberculosis
Telegram : @tarasmedicalstudents

Poliomyelitis
Syphilis
+HIV-infection
Virus B hepatitis

A patient consulted with an immunologist about diarrhea, weight loss within


several months, low-grade fever, enlarged lymph nodes. The doctor suspected
HIV infection. What immunocompetent cells must be studied in the first
place? +Helper T-lymphocytes
Suppressor T-lymphocytes
B-lymphocytes
Monocytes
Plasma cells

In a township there was registered an outbreak of hepatitis, which had


supposedly spread through the water supply. What hepatitis virus could be the
cause of the outbreak in this township?
+Hepatitis E virus
Hepatitis C virus
Hepatitis D virus
Hepatiti G virus
Hepatitis B virus

A 4-year-old girl died suddenly with symptoms of asphyxia. Autopsy revealed


white spots on the buccal mucosa; large blotches of rash on the skin of face,
trunk and extremities; conjunctivitis, edema with foci of necrosis on the
laryngeal mucosa; giant-cell pneumonia on microscopy. What is the most
likely diagnosis?
+Measles
Scarlet fever
Influenza
Meningococcal infection
Typhus

A patient has herpetic conjunctivitis. What etiotropic drug should be


administered?
+Acyclovir
Ampicillin
Methisazonum
Furagin
Tetracycline

To prevent the seasonal influenza epidemics in the city hospitals, sanitary


epidemic station gave orders to immunize health care workers. Which of the
following preparations should be used for immunization?
+Subunit vaccine
Interferon
Telegram : @tarasmedicalstudents

Gamma-globulin
Rimantadine
Amantadine

Blood serum of a newborn contains antibodies to measles virus. What kind of


immunity is this indicative of?
+Natural passive
Natural active
Artificial passive
Artificial active
Heredoimmunity

During winter epidemics of influenza caused predominantly by virus


А/California/04/2009 (H1N1), on the 2nd day after the disease onset a 30-year-
old hospitalized man presented with high fever, dry cough, myalgia, headache,
and general weakness. What should be prescribed as etiotropic treatment in
this case?
+Neuraminidase inhibitors (Oseltamivir)
Antibiotics
Immunoglobulin
Interferon inducers
Acyclovir

A doctor examines a 17-year-old girl. The following is detected: pharyngitis,


cervical lymphadenopathy, fever. The preliminary diagnosis is infectious
mononucleosis. What method of investigation allows confirm this diagnosis
at the disease onset?
+Determining antibodies IgМ to Epstein-Barr virus
Microscopy of blood smear according to Giemsa
method Determining antibodies IgG to Epstein-Barr
virus Sabin-Feldman dye test
Determining the amount of C-reactive protein

Hepatitis B is diagnosed through laboratory tests that determine the presence


of HBA DNA in blood serum of the patient. Which reference method is used for
this purpose?
+Polymerase chain reaction
Hybridization method
Hybridization signal amplification method
Ligase chain reaction method
ELISA diagnostic method

A patient has been hospitalized with provisional diagnosis of hepatitis B


virus. Serological reaction based on complementation of antigen with
antibody chemically bound to peroxidase or alkaline phosphatase was used
for disease diagnostics. What is the name of the applied serological reaction?
Telegram : @tarasmedicalstudents

+Enzyme-linked immunosorbent assay


Radioimmunoassay
Immunofluorescence test
Bordet-Gengou test
Antigen-binding assay

A 50-year-old woman is being treated for shingles in a neurology unit. What


reactivated virus causes this disease?
+Varicella zoster virus (chickenpox virus).
Herpes simplex virus type 1.
Herpes simplex virus type 2.
Measles virus.
Cytomegalovirus.

An outbreak of an intestinal infection occurred in a kindergarten on the eve of


New Year holidays. Bacteriological examination of patients’ faeces didn’t reveal
any pathogenic bacteria. Electron microscopy revealed roundish structures with
clear outer edges and a thick core resembling a wheel. Specify the most likely
causative agent of this infection:
+Rotavirus
Adenovirus
Coxsacki virus
E. coli
P. vulgaris

A three-year-old child has had marked diarrhea for three days. Immune
electron microscopy of his excrements revealed bi-layer pseudocovered
capsid viruses that looked like small spoke wheels. What viruses have been
revealed? Retroviruses
+Rotaviruses
ECHO viruses
Coxsackie viruses
Coronaviruses

Regarding the clinical importance of viral envelopes, which of the following is


true for enveloped viruses?
Resistant in the environment
Can modify to be non-enveloped
+Environmentally labile & spread in large droplet
secretion Persist in environment for a long time All is
correct
Telegram : @tarasmedicalstudents

Adenoviruses, Poxviruses and Herpesviruses are:


Single-stranded DNA viruses
Single-stranded RNA viruses
Double-stranded RNA viruses
+Double-stranded DNA viruses
Mixed-stranded DNA viruses

Are any drugs available to prevent or treat influenza?


Interferon
Ascorbic acid
Amantadine
+Oseltamivir (Tamiflu) and zanamivir (Relenza)
Lamivudine

How long does it take to develop symptoms of influenza after being exposed?
From 6 to 10
+From 1 to 4 days
From 10 to 21 days
From 1 to 6 month
More than 1 year

What are the symptoms of influenza?


Dark urine, yellow skin or eye whites, loss of
appetite Skin rash, runny nose, Koplik's spots, fever
+Fever, aching muscles, sore throat, and nonproductive cough
Pain in the swollen salivary glands on one or both sides of your face
Fever blisters on the lips, pain and itching
How is influenza spread?

How is influenza spread?


+By direct contact and by airborne droplets expelled during breathing, talking,
sneezing or coughing
Fecal-oral
Transplacental
Sexual
Parenteral

Are antibiotics effective against influenza?


Antibiotics are effective against influenza because the flu is a bacterial infection
Antibiotics are effective against influenza because they treat viral infections
+Antibiotics are ineffective against influenza because the flu is a viral infection
Telegram : @tarasmedicalstudents

Antibiotics are ineffective against influenza because there is no treatment for this
infection
None of above

Influenza viruses genome consists of:


5 segments of RNA
6 segments of DNA
1 ss RNA
11 segments of RNA
+8 segments of RNA

Paramyxoviruses are:
Enveloped viruses only with hemagglutinin
+Enveloped viruses with hemagglutinin and neuraminidase spikes and fusion (F)
protein
Naked viruses with hemagglutinin and neuraminidase spikes and fusion (F)
protein Complex viruses
Enveloped viruses only with neuraminidase

Which type of symmetry have Paramyxoviruses?


Complex
Icosahedral
+Helical
Spherical
Pleomorphic

The cytopathic effect of mumps virus?


+Giant multinucleated cells formation
Total destruction of the cell layer
Subtotal (partial) destruction
Localized damage of the cell layer due to direct cell-to-cell transfer of the virus rather
than diffusion through the extracellular medium
Cells swell significantly and clump together in clusters (‘bunch of grapes’)

Which influenza viruses can cause human pandemics?


Influenza viruses B and C
Only influenza viruses B
Influenza viruses A and B
+Only influenza viruses A
Influenza viruses don’t cause pandemics
Telegram : @tarasmedicalstudents

Specific treatment for rhinoviruses includes:


Interferon
Lamivudine
+There is no specific treatment
Oseltamivir
Zanamivir

Foot-and-mouth virus mostly infects:


Insects
Children under age 10
+Animals
Adults
Monkeys

Chose enteroviruses:
Coxsacki A and B viruses
ECHO virus
Rhinovirus
Poliovirus
+All of above

Coxsacki B viruses belong to family:


Retroviridae
Reoviridae
Paramyxoviridae
+Picornaviridae
Orthomyxoviridae

Picornaviruses genome is:


8 segments of (-)RNA
+Linear ss (+) RNA
Circular ss (+) RNA
Linear ss (-) RNA
Linear ss (+) DNA

Paramyxoviruses genome is:


+Not segmented ss (-) RNA
Segmented ss (-) RNA
Circular ss (+) RNA
Linear ss (-) RNA
Not segmented ss (+) RNA
Telegram : @tarasmedicalstudents

Chose paramyxoviruses:
Rhinovirus
Influenza virus
+Parainfluenza virus 1
Coxsacki B virus
Hepatitis A virus

Which virus resulting in the characteristic “chipmuck cheeks”?


Influenza A virus
HIV
Hepatitis C
Ebola virus
+Mumps virus

Poliovirus belong to family:


Togotaviridae
Papillomaviridae
Paramyxoviridae
+Picornaviridae
Enteroviridae

Chose viruses, which have non-segmented (-) RNA genome and cause rash:
Mumps virus
Influenza virus
Varizella zoster virus
+Measles virus
Respiratory-syncytial virus

The defensive mechanisms against some infectious diseases can be greatly


reinforced with interferon. Interferon preparations will be the most advisable in
cases of the following type of infections:
+ Viral
Helminthic
Protozoal
Microbioses
Fungal

A 32-year-old pregnant woman at the term of 5-6 weeks was vaccinated against
influenza along with her whole family. At that time she was not aware of her
pregnancy. The pregnancy is wanted. The woman needs an advice from the
family doctor regarding the maintenance of her pregnancy, namely whether
Telegram : @tarasmedicalstudents

there is a risk of fetal malformations because of received vaccination. What


advice should the doctor give in this case?
+Vaccination against influenza is safe during
pregnancy Therapeutic abortion is recommended
Immediate ultrasound of the lesser pelvis is necessary
Test for antibodies against influenza virus is necessary
An infectious diseases specialist must be consulted

A 17-year-old girl has made an appointment with the doctor. She plans to begin
her sex life. No signs of gynecological pathology were detected. In the family
history there was a case of cervical cancer that occurred to the patient’s
grandmother. The patient was consulted about the maintenance of her
reproductive health. What recommendation will be the most helpful for
prevention of invasive cervical cancer?
+Vaccination against human papillomavirus
(HPV) Vitamins, calcium, omega-3 C.
Immunomodulators
Antiviral and antibacterial drugs
Timely treatment of sexually transmitted diseases

After a thorough examination the patient who had returned from Central Asia
to Ukraine was diagnosed with spring-summer encephalitis. Its pathogen might
have entered the body through the bite of the following arthropod:
Dog-louse
+Taiga tick
Argasid tick (ornithodorus papillipes)
Itch mite
Mosquito

A patient, who works as a milkmaid, has made an appoinment with a dentist


with complaints of aptha-shaped rash on the mucosa of oral cavity. The doctor
detected rash on her hands in the area of nail plates. What agent causes this
disease?
Herpesvirus.
Foot-and-mouth disease virus.
+Cytomegalovirus.
Vesicular stomatitis virus.
Coxsackie B virus.

A doctor examines a 17-year-old girl. The following is detected: pharyngitis,


cervical lymphadenopathy, fever. The preliminary diagnosis is infectious
mononucleosis. What method of investigation allows confirm this diagnosis at
the disease onset?
+Determining antibodies IgМ to Epstein-Barr virus
Microscopy of blood smear according to Giemsa method
Telegram : @tarasmedicalstudents

Determining antibodies IgG to Epstein-Barr virus


Sabin-Feldman dye test
Determining the amount of C-reactive protein

Some viral infections are diagnosed through laboratory tests that determine the
presence of DNA in blood serum of the patient. What reference method is
applied for this purpose?
+Polymerase chain reaction
Hybridization method
Hybridization signal amplification method
Ligase chain reaction method
ELISA diagnostic method

Some viral infections are diagnosed through laboratory tests that determine the
presence of RNA in blood serum of the patient. What reference method is
applied for this purpose?
Polymerase chain reaction
+Reverse transcriptase polymerase chain
reaction Hybridization signal amplification
method Ligase chain reaction method ELISA
diagnostic method

A pregnant woman was detected with IgM to rubella virus. An obstetrician


gynecologist recommended therapeutic abortion due to the high risk of
teratogenic affection of the fetus. Detection of IgM was of great importance as it
is these specific immunoglobulins that:
+Indicate recent infection
Penetrate placental barrier
Have the largest molecular weight
Are associated with anaphylactic reactions
Are the main factor of antiviral protection

The doctor has an appointment with a patient, who 2 days ago developed
severe chest pain on the left, general weakness, high temperature, and
headache. Objectively along the 4th and 5th intercostal nerves on the left the
skin is hyperemic and there are tight clusters of small vesicles filled with clear
serous content. What is the most likely diagnosis?
+Herpes zoster
Herpes simplex
Streptococcal impetigo
Pemphigus
Dermatitis herpetiformis (Duhring’s disease)
Telegram : @tarasmedicalstudents

A 25-year-old patient was delivered to an infectious diseases unit on the 3rd day
of illness with complaints of headache, pain in lumbar spine and gastrocnemius
muscles, high fever, chill. Objectively: condition of moderate severity. Scleras
are icteric. Pharynx is hyperemic. Tongue is dry with dry brown coating.
Abdomen is distended. Liver is enlarged by 2 cm. Spleen is not enlarged.
Palpation of muscles, especially gastrocnemius muscles, is painful. Urine is dark
in colour. Stool is normal in colour. The most likely diagnosis is:
Leptospirosis
+ Viral hepatitis type A and C.
Malaria
Infectious mononucleosis
E. Yersiniosis

A 26-year-old man complains of chills, rhinitis, dry cough, and fever up to 38oC.
Examination shows him to be in a moderately severe condition; there are small
pale pink non-merging spots on the skin of his back, abdomen, and extremities.
Palpation reveals enlarged occipital and axillary lymph nodes. No information
about vaccination history could be obtained. What is the likely etiology of this
disease?
+Rubella virus
Epstein-Barr virus
Streptococcus
Mumps virus
Neisseria meningitis

A 45-year-old veterinary worker has made an appointment with the doctor for
regular examination. In his duties he frequently deals with animals, however he
denies working with rabies-affected animals. Previously he has received no
antirabic vaccination. What should the doctor recommend in this case?
+Preventive immunization with antirabic vaccine Vaccination
in case of contact with sick animal Preventive immunization
with rabies immunoglobulin Administration of antirabic
vaccine and rabies immunoglobulin Preventive immunization
with anti-rabies serum

A 50-year-old woman is being treated for shingles in a neurology unit. What


reactivated virus causes this disease?
+ Varicella zoster virus (chickenpox virus)
Herpes simplex virus type 1
Herpes simplex virus type
2 Measles virus
Cytomegalovirus
Telegram : @tarasmedicalstudents

A medica lstudent was hospitalized into the infectious diseases unit on the 2nd
Day after the disease on set; the patient is suspected to have infectious
mononucleosis. What results of laboratory analysis can confirm this diagnosis
immediately on the day of the hospitalization?
+IgM antibodies to Epstein-Barr virus were detected
IgM antibodies to herpes simplex virus were detected
Four fold increase in number of antibodies to Epstein-Barr virus was detected
Herpes virus was isolated
Cytomegalovirus antibodies were detected

A patient complains of painless ”sores” on his penis and inguinal lymph nodes
enlargement. Synthomycin emulsion that the patient have been applying to the
”sores” was ineffective. Objectively: on the inner leaf of the foreskin there are
three closely situated rounded erosions, 0,5 cm in diameter, with dense
infiltration that can be palpated at their bases. Make the preliminary diagnosis:
Primary syphilis B
+Herpes simplex (Herpes pro genitalis)
Candidiasis of the inner leaf of the foreskin
Shingles
Erythema multiforme

A 26-year-old woman has been undergoing treatment for community-acquired


pneumonia for 10 days. It is known that her husband had been treated for drug
addiction. Sequential intravenous administration of Amoksiklav
(Amoxicillin+Clavunate) + Levofloxacin combination and vancomycin in the
prescribed dosage was ineffective. Within the last two days the patient’s dyspnea
and intoxication acutely exacerbated, bilateral pulmonary infiltrates are
observed. What is the most likely cause of the medication ineffectiveness?
+HIV infection and pneumocystic pneumonia
Tuberculosis mycobacterium infection with development of
tuberculosis Idiopathic fibrosing alveolitis
Infection with polyresistant bacterial strains
Cancer metastases in the pulmonary tissues

A 57-year-old retired man complains of attacks of burning pain and rashes on


the skin of his face and oral mucosa on the right. Anamnesis: a course of
radiation therapy for treatment of gastric disease, past case of chickenpox.
Objectively: along the third branch of the trigeminal nerve the skin of the face
presents with isolated erosions covered in fibrinous coating. There are multiple
vesicles on the hyperemic and swollen oral mucosa. Right-sided lymphadenitis is
observed. What is the most likely diagnosis?
Telegram : @tarasmedicalstudents

+Herpes zoster
Neuralgia
Smallpox
Acute recurrent herpes
Neuritis

A 35-year-old patient, a veterinarian, came to a dentist with complaints of chills,


fatigue, fever up to 38oC, muscle pain, sensations of dryness, burning, and pain
in the oral cavity, excessive salivation, vesicles in the interdigital folds, on the
lips, oral and nasal mucosa. On examination of the oral cavity there were
detected painful bright red erosions with polycyclic contours against the
background of inflammation. The following was observed: scabs on the lips,
enlarged tongue, impaired speech, salivation up to 4 liters per day. Make the
diagnosis:
+Acute herpetic stomatitis
Murrain
Chickenpox D
Measles
Erythema multiforme

A culture of monkey cells (Vero) and a group of mouse sucklings were infected
with an inoculum taken from a child with provisional diagnosis "enterovirus
infection". There was no cytopathic effect on the cell culture but mouse
sucklings died. What enteric viruses might have caused disease of this child?
+Coxsackie A
ECHO virus.
Unclassified enteric viruses 68-71.
Coxsackie B
Polioviruses

Various biological preparations can be used for poliomyelitis prevention. What


drug induces the type of local intestinal mucosal immunity that lasts the
longest? +Oral vaccination with live vaccine
Parenteral vaccination with inactivated vaccine
Oral introduction of poliovirus-specific immunoglobulin
Parenteral vaccination with live vaccine
Parenteral introduction of normal human immunoglobulin

Preventive vaccination against poliomyelitis is made with inactivated


vaccine introduced parenterally. What immunoglobulin create the
postvaccinal immunity in this case?
+IgM, IgG
IgG,secretoryIgA
Telegram : @tarasmedicalstudents

IgM, secretoryIgA
Serum IgА,IgM
IgЕ, IgM

An 7-year-old child was hospitalized for fever up to 39 oC, inertness, headache,


vomiting. Examination revealed meningeal symptoms. Lumbar puncture was
performed. The obtained fluid was characterised by increased opening pressure,
was transparent, with the cell count of 450 cells per 1 mcL (mainly lymphocytes
- 90%), glucose level of 3,6 mmol/l. What agent could have caused the disease
in the child?
+Enterovirus Neisseria
meningitidis
Mycobacterium tuberculosis
Staphylococcus aureus
Streptococcus pneumoniae

A 7-year-old boy complains of increased body temperature, up to 38oC,


headache, sore throat. Objectively: there are erosions on the slightly hyperemic
mucosa of the soft palate, anterior palatal bars, and tonsils. The
submandibular lymph nodes are slightly enlarged, painless. Name the causative
agent of this disease:
+Coxsackie virus
Herpes simplex virus
Epstein-Barr virus
Klebs-Loeffler bacillus
Bordet-Gengou bacillus

Parents of an 8-year-old child complain of rashes in the child’s oral cavity.


Lately the child has been inert, refused to eat. On the oral mucosa there are
small round erosions with clear margins. There are vesicles with turbid content
on the child’s face and scalp. Make the provisional diagnosis:
+Chickenpox
Measles
Hypertensive-hydrocephalic syndrome
Infectious mononucleosis
Scarlet fever

Which viruses are causative agents of aseptic meningitis:


+Coxsackie B group
Rhinoviruses
Poxviruses
Papilomaviruses
Telegram : @tarasmedicalstudents

Polioviruses

Which of the following diseases is caused by a retrovirus?


Lassa fever
+HIV
Influenza
Rabies
All of the above

There were cases of the hospital staphylococcal infection caused by cultures


which had plural antibiotic resistance in the surgical department of stomatology
clinic. Such sign is determined by a presence of:
Exotoxins B
F-plasmid
+Temparate bacteriophages
R-plasmid
Virulent bacteriophages

There was an episode of the hospital infection caused by S.aureus in the surgical
department. In this connection, the liquid polyvalent staphylococcal
bacteriophage was used. Name the purpose of preparation using.
Specific prevention
+Phagotyping
For indication of staphylococcus in reaction of phage title
growth Nonspecific prevention
For treatment of patients with chronic forms of staphylococcosis

Bacteriological researches was conducted with the purpose of source of


shigellosis infection revealing. Research did not give a result. Phagodiagnostics
need to use. Such research provides by:
Phagocytosis activity of blood study
Bacteriophages revealing in path.material
+Phage typing of isolated cultures of pathogens
Growth of bacteriophage title test use
Digestive system functional activity study

The group of students which contacted with patients need to shigellosis


phagoprevention. What mechanism will provide their defense?
Phagotyping
Phagocytosis
+Phagolysis
Pinocytozis
Diffusion
Telegram : @tarasmedicalstudents

There are cases of purulent postoperative complications of the staphylococcal


infection in a surgical hospital. How can you determine a source of a
staphylococcal infection in a hospital?
+Determination of phagovaries
Determination of hemotoxin
Determination of enzymes
Determination of biovaries
Determination of sensitiveness to antibiotics

The toxigenic culture of pathogen is isolated from patient S. with a previous


diagnosis “diphtheria”. It is known, that such activity is connected with the
diphtheria bacteria lysogenicity. What factor determines lysogenicity of
bacteria?
Antibodies
Antibiotics
Phagocytes
Factors of autolysis
+Temperate phages

The reaction of bacteriophages title growth was put with standard choleraic
bacteriophages for estimation of quality of sea water. The reaction was positive.
Such result is the certificate:
+Presence of cholera pathogens in the
samples Absence of cholera pathogens in the
samples Presence of organic substanses in the
samples Absence of organic contamination in
water Presence of BGCB in the samples

A 12-year-old boy had a head pain, nausea, chill, periodic pain in muscles, loss
of appetite, sickliness after school. For what period of illness such symptoms
belong to?
Prodromal
Incubative
Reconvalescence
Latent
+Acute disease

Recently the method of gene indication for nucleic acids fragments revealing in
examined samples are used. Name this reaction.
+Polymerase chain reaction
Phage titration
Radio-immune assay
Reaction of precipitation
Enzyme immunoassay
Telegram : @tarasmedicalstudents

The blood serum is examined for specific antibodies revealing by the solid-phase
method for HIV-infection diagnostics. What type of enzyme-marked antibodies
are used?
Against human’s immunoglobulin
Against HIV anti-genes
+Against gp120
Against gp17
Against gp 41

Which vaccine is used for yellow fever prevention?


Chemical vaccine
+Attenuated vaccine
Anatoxin
Inactivated vaccine
Gene engineering vaccine

For human’s active immunity creation many vaccines are used. What
preparation is presented by live attenuated viruses?
Agains Influenza
Against Papilomaviruses
+Against yellow fever
Vaccine against hepatitis B
All of the above

Mass serological diagnosis of HIV infection is made by means of enzymelinked


immunosorbent assay techniques. What standard component of the reaction
must be adsorbed on the solid phase of the test system?
+HIV antigens
Monoclonal HIV antibodies
Enzyme-marked HIV antibodies
Specific immunoglobulins
Substrates to determine enzyme

A drugstore received a supply of a drug that is widely used for treatment of


many virus diseases since it is not virus specific. What drug is it?
+Interferon
Remantadin
Metisazone
Immunoglobulin
Vaccine
Telegram : @tarasmedicalstudents

Preventive vaccination against poliomyelitis is made with inactivated vaccine


introduced parenterally. What immunoglobulins create the postvaccinal
immunity in this case?
Serum. IgA, IgM
IgE, IgM
IgM, secretory IgA
+IgM, IgG
IgG, secretory IgA

A patient with herpetic stomatitis was prescribed acyclovir for topical


application. What is its mechanism of action?
+It inhibits synthesis of nucleic acids of
viruses It inhibits virus penetration into cells It
inhibits virus maturation
It increases the resistance of macroorganism cells to the
viruses It inhibits virion assembly

A patient is ill with herpetic stomatitis provoked by immunosuppression. What


preparation introduced intravenously, internally and locally can provide
antiviral and immunopotentiating effect?
+Acyclovir
Remantadinum
Levamisole
Methisazonum
Amoxicillin

From the nasopharynx of a 5-year-old child it was excreted a microorganism


which is identical to Corynebacterium diphtheriae dose according to
morphological and biochemical signs. Microorganism does not produce
exotoxin. As a result of what process can this microorganism become toxigenic?
Cultivation in the telluric environment
Chromosome mutation
Passing through the organism of the sensitive
animals +Phage conversion
Growing with antitoxic serum

Because of suspected intrahospital infection in the neonatal department of the


maternity home the inspection was carried out. In some children and on some
general things Staphylococcus aureus was revealed. What properties of these
cultures allow to establish their origin from one source?
Antibioticogramma
Antigenic structure
Biochemical activity
+Phagotype
Telegram : @tarasmedicalstudents

Chromogenesis

From the fecal sample of a patient Shigella sonne were isolated. What additional
studies are required to identify the source of infection?
+Phage-typing of the isolated pure culture
Antibiogram
Precipitation reaction
Complementfixation reaction
Neutralization reaction

A doctor was addressed by a 30-year old man. There is a probability of the


patient being HIV-positive. To clarify the diagnosis, the doctor proposed to
perform polymerase chain reaction. The basic process in this kind of
investigation is:
+Gene amplification
Transcription
Genetic recombination
Genomic mutation
Chromosome mutation

Examples of human viruses are West Nile virus, Rabies virus, Hantaviruses and
some other. The source of invasion of such virus is animal. Such human
parasites cause diseases that are called:
Anthroponotic
+Zoonotic
Anthropozoonotic
Infectious
Multifactorial

Genetic variation in viruses contributes to their ability to evade the immune


response. Select the principal means by which antigenic shift occurs in influenza
A virus:
Low fidelity of DNA dependent DNA
polymerase Low fidelity of RNA dependent DNA
polymerase Low fidelity of reverse transcriptase
+Reassortment of fragments of the RNA genome
Recombination between DNA genomes

Many antiviral drugs act by inhibition of a viral DNA polymerase enzyme.


Select the virus for which this class of drugs would be effective.
+Herpes simplex type 1
Ebola
Measles
Mumps
Telegram : @tarasmedicalstudents

Rabies

Viruses are:
Larger than bacteria
Same size as bacteria
+Smaller than bacteria
Obligate extracellular parasites
Facultative intracellular parasites

The following statements are true about viruses except:


Obligate intracellular parasites
Do not possess cellular organelle
Invisible with light microscope
Unicellular organisms comprising of either DNA or
RNA +Sensitive to antibiotics

Non-enveloped viruses are called:


Identical viruses
+Naked viruses
Viral subunits
Progeny virions
Ventices

Describe mechanism of action of Enfuvirtide (T-20)


It inhibits synthesis of nucleic acids of
viruses +It inhibits virus penetration into cells
It inhibits virus maturation
It increases the resistance of macroorganism cells to the
viruses It inhibits virion assembly

Ideal and specific test to identify nucleic acid material in patients sample is:
RIA
FFT
+PCR
ELIZA
RPHA

Bacteriophage are readily counted by the process of:


Immunoassays
ELISA
+Plaque assays
Telegram : @tarasmedicalstudents

Tissue cell culture


Electron Microscopy

Which of the following is not an RNA virus?


Retrovirus
Enterovirus
Rhabdovirus
+Adenovirus
Rubellavirus

What is the most common cause of aseptic meningitis of viral etiology?


+Enteroviruses
Herpesviruses
Arboviruses
Retroviruses
Orthomyxoviruses

The finding of large, multinucleated, clumps of cells in the bronchial


secretions of a 2 year old girl with acute bronchopneumonia suggests that this
infection is caused by
Bordetella pertusis
Epstein-Barr virus
Mycoplasma hominis
Rhinovirus
+Respiratory syncytial virus (RSV)

…….. are the only viruses that produce genome DNA by reverse transcription
with mRNA as the template.
Hepadnaviruses
+Retroviruses
Influenza viruses
SARS viruses
Polioviruses

HIV can infect


Microglial cells
M (membranous) cells
CD4 + T lymphocytes
+All of these

Parasitic infection/s frequently observed in HIV disease is / are:


Cryptosporidiosis
Toxoplasmosis
Isosporiasis
+All of these
None of these
Telegram : @tarasmedicalstudents

What is the average incubation period for development of


AIDS? One year
Five years
+Ten years
Twenty years

Which of the following is transmembrane pedicle antigen of HIV-


I? +gp 41
gp 140
gp l20
gp 36

The cells most often infected by HIV are


Null cells
CD8 + T lymphocytes
+CD4 + T lymphocytes
Hepatocytes
None of these

Which of the following is the spike antigen of HIV-I?


+gp 120
gp 140
gp 36
gp 41

………… virus is a defective virus?


Influenza
SARS
Marburg
+Hepatitis D

Vaccines have been developed to protect against which hepatitis


viruses? A and C
+A and B
B and E
A and E
All of the above

Which form of hepatitis can exist for years without symptoms?


A and C
A and B
B and E
A and E
+B and C

Which form of hepatitis can be passed on through contaminated food or


water? A and C
Telegram : @tarasmedicalstudents

A and B
B and E
+A and E
B and C

What are the symptoms of hepatitis?


Tiredness (fatigue)
Low fever
Nausea
+All of the above
None of the above

How many main forms of viral hepatitis have been found so far?
8
4
+5
6
7

A blood test can confirm hepatitis. Physicians look for an elevated amount of
which of these?
White blood cells
Calcium
Interferon
+Liver enzymes
All of the above

The following viruses are transmitted from animals to humans except:


Rabies Virus
+Polioviruses
Hantaviruses
Arenaviruses
Lassa Fever Virus

Epstein-Barr Virus (EBV) is associated with


Rhinitis
Common cold
Influenza
+Infectious Mononucleosis
Warts

Epstein-Barr Virus (EBV) is associated


with +Burkitt's lymphoma
Rhinitis
Telegram : @tarasmedicalstudents

Common cold
Influenza
Warts

Choose Nucleoside/Nucleotide Reverse Transcriptase Inhibitors


(NRTIs) Ritinovir, Indinavir
+Azidothymidine, didanosine, lamivudine, abacavir
Maraviroc
Dolutegravir
Elvitegravir

What are the goals of cell culture in virology?


Virus indication
Primary isolation of viruses from pathological material
Virus identification
Virus accumulation and/or maintaining for research, vaccine making,
etc +Аll of the above

What are primary purpose of virus cultivation?


+To isolate and identify viruses in clinical
samples To create new strains of viruses For
growing bacteria
For serological tests
To study the molecular biological properties of viruses

What kind of virus is such a type of cytopathogenic effect-syncytium/symplast


formation?
Rabies virus
Smallpox virus
Herpesviruses
+Measles virus
Paramyxoviruses

Drawbacks of embryonated chicken eggs’ use in virology research:


Gold standard for isolation and cultivation of some viruses,
Ideal substrate for the viral growth and replication,
Extremely cost-effective, easy maintenance,
The embryonated eggs are readily available
+Reaction on virus inoculation differs significantly from the reaction of the host
organism.

Research method, based on ability of some (but not many) viruses to


nonspecifically bind red blood cells of certain laboratory animals:
+Hemagglutination test
Telegram : @tarasmedicalstudents

Enzyme-linked immunosorbent assay (ELISA)


Polymerase chain reaction (PCR)
Neutralization reaction
RT-PCR

Determine what is not a source of intra-laboratory contamination of


cell cultures:
Media and serums
Air and water in water baths
Laboratory staff and their clothes
Laboratory dishes and tubes contaminated by
microorganisms +Antibiotics and antifungal drugs

What factors do not influence the growth of cells in culture?


Quality of dishes for cell cultivation Gas
phase and osmotic pressure Quality of the
nutrient medium Temperature and pH of
the nutrient medium +Weather conditions

What properties aren`t common for organ culture?


Preserves histological differentiation of cells
Keeps homo and heterotype relationships between cells for a long
time. The possibility of interaction with the matrix
Supports biochemical cell differentiation
+Has the ability to unlimited reproduction

What properties aren`t inherent in the culture of transfected cells?


Ability to unlimited reproduction and production of biomass of cells
Ability to clone, breed and clean
Ability to quantify cell culture
Does not change its properties in the process of prolonged
cultivation +Preserves histological differentiation of cells

What doesn`t characterize primary culture of cells?


Heterogeneity of the cell population
Small proliferative cell pool
Ability to support viral reproduction
Expression of the properties of the tissue from which the cell is obtained
+Ability to unlimited reproduction and production of biomass of cells

What characterizes primary trypsinized cultures?


High growth rate
Possibility of achieving higher density of monolayer
cells Less dependent on serum animals
Telegram : @tarasmedicalstudents

Big biomass output per unit area


+Low growth rate

What Acidification of the nutrient medium (pH less than 7.0) indicates?
+The need to replace the nutrient medium
Contamination of cells by microorganisms
Contamination of cells with yeast
Contamination of cells by mycoplasma
Infection of cells with viruses

What does the method of organ culture provide?


+Growing an entire body or its parts with the preservation of the anatomical and
functional interconnection between cells
Growing of individual cells of the embryonic organs without preserving the
anatomical and functional interconnection between cells High growth rate of cells

Cells with chromosome instability


Big biomass output

Indicate the most specific of the listed serological reactions used in the diagnosis
of viral infections:
+Neutralization reaction
Complement Fixation Assay
Hemagglutination inhibition assay
ELISA
Hemagglutination assay

Indicate the most sensitive of the listed serologic reactions used in the
diagnosis of viral infections:
Neutralization reaction
Complement Fixation Assay
hemagglutination inhibition assay
+ELISA
Hemagglutination assay

What doesn`t characterize simple / rapid tests:


Receive the result within 15 minutes
Possibility of rational testing (1 test-1 patient)
No need for laboratory equipment
Wide temperature range of storage and application
+Possibility of obtaining a quantitative result

What methods are used to identify isolated viruses?


+Complement Fixation Assay, hemagglutination inhibition assay,
ELISA Transmission electron microscopy
Telegram : @tarasmedicalstudents

Simple / rapid tests based on immunochromatographic analysis


Bioindication methods
Optical microscopy

What can separate cell culture from glass?


+A solution of Versene or trypsin for cell
culture Earls or Hanks media for a cell culture
Nutrient media 199 and Eagle Sequential
freezing and thawing

What role does the serum of animal blood do NOT play when cultivating cells
in vivo?
Provides attachment and growth factors
Provides transport proteins
Provides hormones
Participates in the creation of biomatrix
+Provides cell separation from the glass

What conditions don’t have to be kept when working with cell cultures?
Rules of asepsis and antiseptics
Work performs in a box or in a laminar cabinet
Safety Rules
Use special sterile clothing
+To work only at night

What is the viral envelope?


Nucleic acid
Major protein component of the shell of the core.
Single virus particle.
+Plasma membrane surrounding virus core.
Protein shell

Which of the following is not true regarding viruses?


May be DNA, RNA, Single, or Double-stranded
All are obligate intracellular parasites +All
have a protein capsid and envelope
Release virions during cell lysis or
budding All of the above

What is a capsid?
+Protein shell surrounding nucleic acid
Single virus particle
Major protein component of the shell of the core
Telegram : @tarasmedicalstudents

Membrane surrounding virus core


Head without tail

A structural component that is never found in viruses:


The envelope
DNA
+Mitochondria
Tail fibers
Spikes

Which statement is true?


A virion contains DNA and RNA
+Viruses are non-cellular
Viruses replicate outside of the cell
Most viruses are easily visualized with a light
microscope Viruses multiply by division

Which is not true regarding capsids?


Composed of repeating subunits
Able to self-assemble into virions
+Helical structure has fixed number of subunits and is spherical in shape
Protects viral genome
All of the above

The basic structure of most human viruses is:


Dodecahedron.
+Icosahedron.
Square.
Tetrakaidecagon.
Tetradecagon.

Which of the following is not an enveloped DNA virus?


Herpesviruses
Poxviruses
Hepadnaviruses
+Adenoviruses
Asfarviruses

A common polyhedral capsid shape of viruses is a:


Pentagon
Cube
+Icosahedron
Pyramid
Telegram : @tarasmedicalstudents

Sphere

What shape Picornaviruses have?


Helical
+Icosahedral
Filamentous
Complex
Enveloped

What shape most of bacteriophages have?


Enveloped
+Complex
Helical
Spherical
None of this

What is virion?
Nucleic acid
Major protein component of the shell of the core.
+Single virus particle.
Plasma membrane surrounding virus core.
Surrounding protein shell

What shape Herpesviruses have?


Helical
Icosahedral
Filamentous
Complex
+Enveloped

Protein shell surrounding nucleic acid is called:


+Capsid
Envelope
Virion
Nucleic acid
Tegument

Single viral particle is called:


Capsid
+Virion
Telegram : @tarasmedicalstudents

Envelope
Nucleic acid
Tegument

Plasma membrane surrounding virus core is called:


Capsid
Virion
+Envelope
Nucleic acid
Tegument

Viral morphology can be characterized by:


+Electron microscopy
ELISA
PCR
Hemagglutination test
None of them

What is not true about electron microscopy?


In EM a beam of electrons is used as source of
light Special facilities are necessary for EM
EM is more expensive then light microscopy
+EM has the same magnification as light microscope
In EM lenses are made of electromagnets

Which capsid symmetry is exhibited by most of the phages?


+Icosahedral
Filamentous
Helical
Pleomorphic
None of this

Type of life cycle when a bacteriophage integrates its own DNA into
the bacterial DNA and passes it to future generations is called:
Virulent
+Lysogenic
Lambda
Lytic
Latent

Bacteriophages undergo replication by:


Telegram : @tarasmedicalstudents

Binary fission and fragmentation


+Lytic cycle and lysogenic cycle
Mitosis
Meiosis
None of them

The viral genome integrated to the bacterial genome is called:


Plasmid
+Prophage
Capsid
Virion
Tail

What is NOT true about bacteriophages?


Each bacteriophage species is specific to a certain bacteria
There are two types of phage life cycles, lytic and
lysogenic +All bacteriophages have heads and tails
During lysogenic life cycle virus exists in the form of
prophage Phages can reproduce only inside bacterial cell

How many phages there are on the Earth?


+1031
1021
1029
1 billion
1 million

What structure bacteriophages use to insert its genome into host cell?
Head
Fibres
+Tail
Envelope
Capsid

What method can be used to calculate titer of bacteriophages in sample?


+Double-layer agar method
Phage typing
Hemagglutination test
ELISA
Immunofluorescence
Telegram : @tarasmedicalstudents

During lytic cycle, phages undergo all these stages except:


Adsorption
Penetration
Maturation
+Integration of viral DNA
Assembly

During lysogenic cycle, phages undergo a specific stage called:


+Integration
Penetration
Attachment
Release
Replication

Bacteriophages can be used in medicine for:


Prevention and treatment of bacterial infections
Treatment of agricultural plants
Veterinary medicine purposes
Wastewater treatment
+All of the above

What is NOT true about phage therapy?


Bacteriophages are highly specific
Phages don’t affect human microflora
Production of phage drugs is less expensive than
antibiotics +Phages therapy helps to cure viral infections
Phage therapy can be used in agriculture

What is transduction?
+Transferring of bacterial DNA by bacteriophages
Releasing of new virions from bacterial cell
Method by which titer of phages can be calculated
Assembly of viruses
None of them

From the fecal sample of a patient Shigella sonnei were isolated. What
additional studies are required to identify the source of infection?
+Phage-typing of the isolated pure culture
Antibiogram
Precipitation reaction
Complement-fixation reaction
Telegram : @tarasmedicalstudents

Neutralization reaction

Process of transferring of bacterial DNA by bacteriophages is called:


Integration
+Transduction
Attachment
Release
Replication

What method is not useful for bacteriophages?


Double-layer agar method
Electron microscopy
PCR
+Hemagglutination test
Spot-test

Phages that can infect more than one bacterial species are called:
+Polyvalent
Lysogenic
Lytic
Abnormal
None of them

Type of life cycle when phage infects bacterial cell and then lyse it:
Lysogenic
Polyvalent
+Lytic
Normal
None of them

Which family from the list does not consist of bacteriophages?


Myoviridae
Siphoviridae
Podoviridae
Lipothrixviridae
+Picornaviridae

How phage DNA entries the cell?


Through direct penetration of virus
+Through injection
During mitosis
Telegram : @tarasmedicalstudents

In the vesicles
None of them

What is the main genetic characteristic of the herpesvirus


family? +Large linear DNA genome,
Small ssDNA genome,
Segmented DNA genome,
Circular dsDNA genome.
ds RNA

Which of the following statements is true about the transmission of


HSV? +The average incubation period is 10 days.
Likelihood of transmission does not change with increased duration of infection.
HSV is readily inactivated by drying and soap and water.
Most sexual transmission occurs while the source contact case is symptomatic.
HSV is transmitted primarily by the faecal-oral route.

Sexual transmission of genital herpes is less efficient from


+Women to men
Men to women
Transmission efficiency is the same from men to women and from women to men
Men to men
Women to women

Which clinical presentation is typical of a primary first episode HSV-2


infection?
No lesions or symptoms, HSV-1 antibodies may or may not be present, HSV-2
antibodies are present
+Lesions present, symptoms usually mild, HSV-1 antibodies may or may not be
present, HSV-2 antibodies are present
Lesions present, symptoms usually severe, HSV-1 and HSV-2 antibodies are not
present
Lesions present, symptoms usually moderate, HSV-1 antibodies are present, HSV-2
antibodies are not present
Abdominal pain or discomfort, especially on the upper right side beneath your lower
ribs.

What is the morphology of the herpes virion?


+Complex with a membrane and tegument and icosahedron
core Baggy virion with over 50 types of spikes Compact
icosahedron structure
Telegram : @tarasmedicalstudents

Small round virus


Flexuous and filamentous particles

What disease do herpes viruses not cause?


+Infantile paralysis
Encephalitis
Sexually transmitted disease
Cancer
Gingivostomatitis

A doctor examines a 17-year-old girl. The following is detected: pharyngitis,


cervical lymphadenopathy, fever. The preliminary diagnosis is infectious
mononucleosis. What method of investigation allows confirm this diagnosis at
the disease onset?
+Determining antibodies IgМ to Epstein-Barr virus
Microscopy of blood smear according to Giemsa
method Determining antibodies IgG to Epstein-Barr
virus Sabin-Feldman dye test
Determining the amount of C-reactive protein

An HIV-positive patient’s cause of death is acute pulmonary insufficiency


resulting from pneumonia. Pathohistological investigation of lungs has revealed
interstitial pneumonia, alveolocyte desquamation and methamorphoses:
alveolocyte enlargement, large intranuclear inclusions surrounded by lightly-
coloured areas. Transformed cells resemble owl’s eye. Name the causative agent
of pneumonia:
+Cytomegalovirus
Pneumococcus
Influenza virus
Candida fungi
Toxoplasma

A 50-year-old woman is being treated for shingles in a neurology unit. What


reactivated virus causes this disease?
+Varicella zoster virus (chickenpox virus)
Herpes simplex virus type 1
Herpes simplex virus type 2
Measles virus
Cytomegalovirus
Telegram : @tarasmedicalstudents

There are 57 serotypes of adenovirus currently recognised. Which has the most
important clinical impact?
Diarrhoea
Eye infections
+Respiratory tract infections
Haemorrhagic cystitis
Jaundice and abdominal discomfort

The adenovirus virion has which unique structural feature?


+Icosahedron
Icosahedron with slender fibres
'Complex' structure not yet fully explored
Flexuous lipid containing structure
Flexuous and filamentous particles

Adenovirus infection is characterized mainly by the mucous membrane lesions


of:
nose
+pharynx
larynx
trachea
bronchi

How many types of human herpesviruses are known:


8
+9
5
6
11

Select zoonotic human herpesvirus


VZV
HSV-1
HSV-2
CMV
+Herpes B virus

Which antiviral drug can be used in treatment of herpesvirus infections?


Ribavirin
Rimantadin
Sofusbuvir
Telegram : @tarasmedicalstudents

+Acyclovir
Amantadine

Vaccination is available for which herpesvirus?


KSHV
+VZV
HSV-1
HSV-2
CMV

Where VZV multiplies during the primary infection?


+Lungs
Gastrointestinal tract
Liver
Skin
Spine

Which herpesvirus can be vertically transmitted and is one of the pathogens that
cause congenital abnormalities?
KSHV
VZV
HSV-1
HSV-2
+CMV

What diseases adenoviruses can cause?


Pharyngitis
Tonsillitis
Conjunctivitis
Adenoiditis
+All of the above

What is true about bacteriophages?


+Are the most abundant biological entity on earth
Always have lipid membrane Infect eukaryotic
cells
Pathogenic for human
All of the above

What is true about viral infections?


+Are universal for all organisms
Telegram : @tarasmedicalstudents

Can be treated with antibiotics


Always display symptoms
All of the above
None of the above

What is human virome?


Causative agent of viral infection
The total collection of viruses in the human body
The total collection of viruses on the human body
+The total collection of viruses in and on the human body
The total collection of viruses outside of the human body

What is true about human virome?


+Viruses in the human body may infect both human cells and other microbes
Viruses in the human body may infect only human cells
Viruses in the human body may infect only eukaryotic cells
Viruses in the human body play only pathogenic role None
of above

Which viruses are integrated into human genome


Adenoviruses
+Retroviruses
Picornaviruses
Poxviruses
All of the above

Name common properties for virus classification


Virion morphology
Nature of genome
Presence or absence of lipids
Biological properties
+All of above

Which of the following is the largest?


Influenza virus
Rhinovirus
Adenovirus type 5
+Smallpox virus
HIV
Telegram : @tarasmedicalstudents

Which of the following is the smallest?


+Yellow fever
Rhinovirus
Adenovirus type 5
Herpes simplex virus
B19

Name viral capsid protein subunits


Spikes
+Capsomeres
Capsules
Peplomeres
Glycoproteins

Determine viral capsid function


Protection of genome from atmosphere
Interaction with cellular receptor to initiate infection
Interaction with vector for specific transmission
Delivery of genome in infectious form
+All of the above

Select the basic viral structure for naked


viruses DNA-Capsid protein
+RNA or DNA-Capsid protein
DNA+RNA-Capsid protein
Nucleocapsid-lipid membrane
RNA-Capsid protein

Select the basic viral structure for enveloped


viruses DNA-Capsid protein
RNA or DNA-Capsid protein
DNA+RNA-Capsid protein
+Nucleocapsid-lipid membrane
RNA-Capsid protein

Which viruses have oval or 'brick-shaped' particles


Adenoviruses
Retroviruses
Picornaviruses
+Poxviruses
All of the above
Telegram : @tarasmedicalstudents

Which viruses have 2 membranes


Adenoviruses
Retroviruses
Picornaviruses
+Poxviruses
All of the above

The smallest genome is characteristic for


Adenoviruses
Retroviruses
Picornaviruses
Poxviruses
+Viroids

The smallest genome is characteristic for


Adenoviruses
Retroviruses
Picornaviruses
+Poxviruses
Viroids

Types of viral DNA are following


Linear single stranded
Linear double-stranded
Circular single- or double-stranded
Circular single- or double-stranded
+All of the above

Types of viral RNA are following


Linear single stranded positive
Linear single stranded negative
Linear single stranded segmented
Linear double-stranded segmented
+All of the above

Which viruses have segmented genomes?


+Orthomyxo- and Rotaviruses Orthomyxo-
and Paramyxoviruses Pox- and
Adenoviruses
Parvo- and Adenoviruses
Telegram : @tarasmedicalstudents

All of the above

Viral envelop is sensitive to


Drying
Heat
Detergents
Acids
+All of the above

Viral capsid is sensitive to


Drying
Heat
Detergents
Acids
+None of the above

According to their stability in environment enveloped


viruses +Must stay wet during transmission
Capable to survive in gastrointestinal tract
Retain infectivity after drying
Spread easily via fomites
Can survive on environmental surfaces

According to their stability in environment naked viruses


Must stay wet during transmission
Unable to survive in gastrointestinal
tract Transmitted in large droplets
+Spread easily via fomites
Unable survive on environmental surfaces

What is a viral infection at the microscopic


scale Using host cells to integrate into genome
Using host cells to realize reverse transcription
Using host cells to inhibit immune response
+Using host cells to make more copies of
viruses All of above

Ability of most viruses infect only certain types of cells within tissues is called
+Tissue tropism
Tissue specifyty
Telegram : @tarasmedicalstudents

Tissue recognition
Tissue compatibility
Tissue viability

What virus recognizes chemokine coreceptors?


+HIV
Ad5
HSV-1
HSV-2
VZV

The target cells for rabies virus are:


+Nerve cells
Epithelial cells
T-lymphocytes
B-lymphocytes
All of the above

The target cells for B19:


Nerve cells
Epithelial cells
T-lymphocytes
B-lymphocytes
+Erythroid precursor cells

The target cells for Rinoviruses are:


Nerve cells
+Epithelial cells
T-lymphocytes
B-lymphocytes
All of the above

The target cells for Influenza virus A are:


Nerve cells
+Epithelial cells
T-lymphocytes
B-lymphocytes
All of the above

Directional assembly around genome is characteristic of:


+Rod-shaped viruses
Telegram : @tarasmedicalstudents

Spherical viruses
Complex viruses
All of the above
None of the above

Which viruses are assembled in nucleus?


Picornaviruses
Bunyaviruses
Rhabdoviruses
+Adenoviruses
All of the above

Name infection caused by normally nonpathogenic organisms:


Acute
Persistent
Latent
Slow
+Opportunistic

Select features of vertical transmission of viruses:


Virus travels from one host to another between two individuals, generally in the same
generation
+Viruse can be transmitted directly from a parent to members of the next
generation Direct contact can occur due to touching, biting or licking or sexual
intercourse Indirect contact can occur through vectors All of above

Enveloped viruses mature by:


Repturing the cell membrane causing cell death and relishing the virus
Incorporating cell DNA into the viral genome
Allowing the recombination of RNA sequences in cells
+Budding from the cell mamraine Assembly in
nucleus

Which DNA viruses cause a verity of syndromes including common colds, acute
pharyngitis, tonsillitis, conjunctivitis and gastroenteritis?
Herpesviruses
Poliomaviruses
Papilomaviruses
+Adenoviruses
All of above
Telegram : @tarasmedicalstudents

Which virus is transmitted by sexual contact and by exposure to secretions or


blood from infected individuals?
Poliovirus
+HBV
HAV
HEV
All of above

Which virus is not transmitted by exposure to blood from infected individuals?


HCV
HBV
+HEV
HIV
All of above

Which virus cases a lacey rash on the face and syndrome called “slapped face
fever”
+Parvovirus
Measles virus
Mumps virus
Herpesvirus
HIV

Which viruses are the most common cause of acute adult viral gastroenteritis?
Adenoviruses
+Noroviruses
Enteroviruses
Herpesviruses
Flaviviruses

Which viruses are the most common cause of acute children gastroenteritis?
Adenoviruses
+Rotaviruses
Enteroviruses
Herpesviruses
Flaviviruses

What member of herpesvirus group causes chickenpox?


HSV-1
HSV-2
Telegram : @tarasmedicalstudents

VZV
CMV
All of above

Measles and mumps viruses are representatives of:


Picornaviruses
+Paramyxoviruses
Orthomyxoviruses
Retroviruses
Herpesviruses

Dengue and Zika viruses are representatives of:


Picornaviruses
Paramyxoviruses
Orthomyxoviruses
Retroviruses
+Flaviviruses

How many distinct types of influenza virus have been identified?


2
+3
4
5
7

Which virus causes painful vesicular lesions on various mucocutaneous


sites? +HSV
HIV
CMV
HPV
All of the above

Which virus cases regular winter and spring outbreaks of bronchitis and
pneumonia in young infants and children?
+RSV
Influenza virus A
Influenza virus B
VZV
HEV

Gradually change of influenza virus amino acid composition is called:


Telegram : @tarasmedicalstudents

+Antigenic drift
Antigenic shift
Antigenic properties
Silent mutations
Nonsense mutations

Many zoonotic viruses that capable to cause encephalitis or hemorrhagic disease


belong to group of:
Picornaviruses
+Bunyaviruses
Herpesviruses
Paramyxoviruses
All of the above

Biosafety level 4 includes a number of viruses belonging to group of:


Picornaviruses
+Bunyaviruses
Herpesviruses
Paramyxoviruses
All of the above

Biosafety level 4 includes a number of viruses belonging to group of:


Picornaviruses
+Filoviruses
Herpesviruses
Paramyxoviruses
All of the above

Name a family of viruses whose members are generally associated with rodent-
transmitted diseases in humans.
Picornaviridae
Filoviridae
Herpesviridae
+Hantaviridae
All of the above

Name a family of viruses whose members are generally associated with rodent-
transmitted diseases in humans.
Picornaviridae
Filoviridae
Telegram : @tarasmedicalstudents

Herpesviridae
+Arenaviridae
All of the above

Causative agents of hemorrhagic fever with renal syndrome are:


Picornaviruses
+Hantaviruses
Herpesviruses
Paramyxoviruses
All of the above

The representatives of which virus family may appear in several shapes


Picornaviridae
+Filoviridae
Herpesviridae
Arenaviridae
All of the above

Middle East respiratory syndrome is caused by:


+Coronavirus
Hantavirus
Rotavirus
Paramyxovirus
Filovirus

Severe Acute Respiratory Syndrome is caused by:


+Coronavirus
Hantavirus
Ortomyxovirus
Paramyxovirus
Filovirus

About half of the world's population is now at risk to be infected with the
following arbovirus:
+Dengue virus
Yellow fever virus
Tick-borne encephalitis virus
Japanese Encephalitis virus
Crimean-Congo hemorrhagic fever virus

The most distributed arbovirus in Europe is:


Telegram : @tarasmedicalstudents

Dengue virus
Yellow fever virus
+Tick-borne encephalitis virus
Japanese Encephalitis virus
Crimean-Congo hemorrhagic fever virus

What is true about Japanese Encephalitis?


JE is the leading cause of viral neurological disease in Asia More
than 3 billion people live in areas where JE is transmitted. Up to
10,000 to 15,000 deaths are reported each year.
JE can be controlled using inactivated
vaccines +All of above

The most common bacterial infection in HIV-infected patients is


+Mycobacterial infection
Salmonella infection
Bartonella infection
Serratia infection
all of these

Viral infections frequently observed in HIV-infected patients is /are


Herpes simplex type 1
Herpes simplex type 2
Varicella- zoster
Cytomegalovirus
+All of these

Which animal species serves as a reservoir for herpes B virus?


Cattle
Sheep
+Monkey
Horse
Dog

What is true about human polyomavirus infection?


Capable of causing tumors in animals and humans
Can cause other diseases
BK and JC viruses are endemic worldwide
Infection outcome depends on the individual’s immune system
+All of the above

Select human polyomaviruses?


HIV and HTLV-1
Telegram : @tarasmedicalstudents

+BK-virus and JC-virus


HSV-1 and HSV-2
HCV and HEV
CMV and VZV
No Question
1 What is included in MALT?
A. thymus
B. Peyer’s patches
C. spleen
D. bone marrow
E. none of the above
2 A patient with periodontitis of the lower molar came to the doctor. It
was determined that the inflammatory process spread to the lymph
nodes. What lymph nodes were the first to be affected by the
inflammatory process?
A. Lateral cervical
B. Submandibular
C. Anterior cervical
D. Submental
E. Facial
3 A specimen shows an organ covered with connective tissue capsule
with trabeculae radiating inward the organ. The organ’s cortex
contains lymph nodules; there are medullary cords made of lymphoid
cells. What organ is under study?
A. Tonsils
B. Thymus
C. Spleen
D. Red bone marrow
E. Lymph node
4 The internal immune system is mainly enforced by leukocytes, the
leukocytes have different half-lives and are constantly generated, about
1011 each day in a human adult. Where are they generated?
A. Innate immune cells originate from the bone marrow, lymphocytes from
lymphoid tissue
B. All leukocytes except T cells originate from the bone marrow
C. B cells originates from the bone marrow, T cells from the thymus and
innate immune cells are formed throughout the body
D. Leukocytes originate from most tissues in the adult body
E. All leukocytes originate from the bone marrow
5 A child falls and suffers a deep cut on her leg. The cut went through
her skin and she is bleeding. Which of the following defense
mechanisms will participate in eliminating contaminating microbes?
A. mucociliary escalator
B. normal skin flora
C. phagocytosis in the inflammatory response
D. acidic skin secretions
E. lysozyme
6 Margination refers to
A. the adherence of phagocytes to microorganisms
B. the chemotactic response of phagocytes
C. adherence of phagocytes to the lining of blood vessels
D. dilation of blood vessels
E. the movement of phagocytes through walls of blood vessels
7 Thymus is a central lymphoid organ for cellular maturation and
selection. Which of the following cells are negatively selected in the
thymus?
A. Autoreactive B-cells
B. Autoreactive NK-cells
C. T-cells recognizing allogeneic MHC
D. T-cells with strong affinity to autologous MHC
E. T-cells with low affinity to autologous MHC
8 Which of the following is mismatched?
A. diapedesis — movement of leukocytes between capillary walls cells out
of blood and into tissue
B. chemotaxis — chemical degradation inside a phagolysosome
C. abcess — a cavity created by tissue damage and filled with pus
D. pus — tissue debris and dead phagocytes in a white or yellow fluid
E. scab — dried blood clot over injured tissue
9 Innate immunity includes all of the following EXCEPT
A. phagocytosis
B. inflammation
C. production of antibody
D. production of interferon
E. activation of complement
10 Which of the following defense systems would be involved in
eliminating virally-infected cells?
A. Humoral immunity
B. Phagocytosis
C. Complement system
D. T lymphocytes
E. AMPs
11 Which of the following statements is true?
A. Memory B cells are typically established when the B cell binds to an
antigen
B. Adaptive defenses include both humoral and cellular immunity
C. Adaptive defenses include humoral immunity only
D. Innate defenses are enough to keep a person healthy
E. Innate defenses include only cell immune responses
12 Which statement regarding the lymphatic system is true?
A. Lymph nodes are sites of activation of neutrophils, which destroy
microbes
B. Lymphatic capillaries possess one-way valves. These valves permit the
uptake of fluid from the body but do not allow the fluid to flow back out of
the capillaries into the intracellular spaces
C. The pancreas contains T-lymphocytes and macrophages that monitor the
blood for microbes
D. The thymus serves as the site for differentiation of B cells
E. Peyer’s patches contain connective tissue capsule
13 Which defense mechanism is considered a specific category of
immunity?
A. Epithelial membranes that cover the body surfaces
B. Strong acidity the gastric juice
C. Phagocytosis of unwanted substances
D. Activation of lectin pathway of complement cascade
E. Activation of B-lymphocytes
14 Haptens are
A. small antigentic organic molecules
B. antigenic when bound to carbohydrates
C. antigenic when bound to proteins
D. cells that release chemotaxtic agents
E. only short peptides
15 The thymus gland is most active during
A. middle age
B. infancy
C. old age
D. adolescence
E. A and D
16 Where are the tonsils located?
A. beneath sternum overlying heart
B. throat
C. armpits, groin, neck
D. small intestine
E. abdomen
17 The organ responsible for positive selection of B-lymphocyte, is the
A. thymus
B. spleen
C. bone marrow
D. Peyer’s pathce
E. liver
18 During an infection, lymph nodes enlarge and become tender because
A. macrophages are rapidly dividing
B. edema accumulates within the fibrous capsules
C. the nodes are not function properly
D. B and C
E. microorganisms are accumulating
19 Chronic inflammation is characterized by:
A. neutrophilic infiltration
B. lymphocytic and macrophagic infiltration
C. metaplasia
D. hypertrophy
E. All of the above are correct
20 Which of the following is NOT a function of the lymphatic and immune
system?
A.Draining excess interstitial fluid
B.Maintaining water homeostasis in the body
C.Transporting dietary lipids
D.Carrying out immune response
A and B
21 Which of the following is NOT considered an organ of the immune
system?
A.Spleen
B.Lymph node
C.Red bone marrow
D.Thymus
E.Pancreas
22 Which of the following is considered a primary lymphoid organ?
A.Spleen
B.Lymph node
C.Red bone marrow
D.Tonsils
E.Peyer’s patches
23 Which lymphoid organ is not responsible for the antigen presentation
to the naive T-cells?
A. Spleen
B. Lymph node
C. Red bone marrow
D. Tonsils
E. Peyer’s patches
24 Organized mucosa-associated lymphoid tissue is
A. found in the lymph nodes
B. associated with initial immune response to antigen
C. filled with plasma cells that are producing antibodies
D. composed primarily of M-cells and L-cells
E. all of the above
25 The lymphoid organs are organized tissue where lymphocytes (B- and
T-cells) interact with non-lymphoid cells (APC) and trapped antigen.
They are divided into primary central and secondary peripheral
organs and have different roles in immunity. What is the major
function of the peripheral organs?
A. Provide the microenvironment for maturation of T and B cells
B. Maximize contact between antigen and lymphocytes
C. Produce antigen-specific lymphocytes from stem cells in response to
antigen
D. Provide a site where memory T-cells reside to ensure a rapid response to
antigen
E. Sequester antigen to minimize its damage to the body
26 An example/examples of a non-specific chemical barrier to infection
is/are
A. unbroken skin
B. blood-brain barrier
C. lysozyme in saliva
D. mucociliary escalator
E. all of the above
27 Which non-specific host defense is associated with the trachea?
A. lacrimation
B. ciliary lining
C. desquamation
D. lactic acid
E. all of the above
28 All of the following protect the skin and mucous membranes from
infection EXCEPT
A. multiple layers of cells
B. tears
C. saliva
D. HCl
E. the “ciliary escalator”
29 The function of the "ciliary escalator" is to
A. propel inhaled dust and microorganisms toward the throat
B. remove microorganisms from the gastrointestinal tract
C. remove microorganisms from the lower respiratory tract
D. remove microorganisms from the upper respiratory tract
E. trap inhaled dust and microorganisms in mucous and propel it away from
the lower respiratory tract
30 Symbiotic microbiota provide protection from infection in each of the
following ways EXCEPT
A. they produce antibacterial chemicals
B. they compete with pathogens for nutrients
C. they make the chemical environment unsuitable for nonresident bacteria
D. they produce lysozyme
E. they change the pH of the environment
31 All of the following occur during inflammation. What is the first step?
A. diapedesis
B. margination
C. phagocyte migration
D. repair
E. vasodilation
32 All of the following are components of the inflammatory process
EXCEPT
A. dilation of blood vessels
B. release of histamines and prostaglandins
C. chemotaxis
D. diapedesis
E. antibody synthesis
33 Which non-specific defense mechanism is mismatched with its
associated body structure or body fluid?
A. lysozyme — tears and saliva
B. mucociliary escalator — intestine
C. very acidic pH— stomach
D. keratin and tightly packed cells — skin
E. cerumen and sebum — ear
34 Mucous membranes are a part of
A. cell-mediated immunity
B. innate defense
C. humoral immunity
D. adaptive defense
E. the complement system
35 First line defenses have what aspect in common with each other?
A. They involve the production of antibodies
B. They create physical barriers against invading pathogens
C. They involve unique cells that attack invading pathogens
D. They recognize specific pathogens
E. All of the above
36 Inflammation :
A. destroys injurious agents.
B. confines injurious agents.
C. stimulated and enhances immunity.
D. promotes healing.
E. All of the above are correct
37 Inflammatory microcirculation changes involve all of the following
EXCEPT:
A. vasodilation
B. days to develop
C. increased vascular permeability
D. exudation
E. leukocyte extravasation
38 Which is NOT a systemic manifestation of inflammation?
A. fever
B. exudation
C. leukocytosis
D. increased acute-phase reactants
E. A and B
39 The inflammatory response:
A. prevents formation of abscesses
B. minimizes injury and promotes healing
C. prevents blood from entering the injured tissue
D. elevates body temperature to prevent spread of infection
E. A and B
40 Swelling during acute inflammation is caused by:
A. neutrophilic margination
B. collagenase
C. fluid exudation and cell extravasation
D. lymphocytic margination
E. anaerobic glycolysis
41 Which of the below do NOT induce vasodilation and permeability
(increased fluid flow) to an infection site?
A.Histamines
B.Kinins
C.Perforin
D.Leukotrienes
E.Complement
42 Mechanical barriers in the innate immune system's first line of defense
include:
A. Sebum (pathogen-inhibiting agents)
B. A continuous barrier formed by skin and mucous membranes - several
layers of densely packed cells and other materials protecting against
invasion by foreign cells
C. Mucus (pathogens stick and are swept away)
D. Enzymes (hydrolyse pathogens)
E. B and C
43 Chemical barriers in the innate immune system's first line of defence
include:
A. Sebum (pathogen-inhibiting agents)
B. HCl in gastric mucosa to destroy pathogens; saliva, bile acids, tears,
urine
C. Mucus (pathogens stick and are swept away)
D. Enzymes (hydrolyse pathogens)
E. All of the above
44 Inflammatory response does not do the following:
A. Prevent spread of damaging agent
B. Stimulates & enhances immune response
C. Repair process
D. Disposal of cell debris & pathogens
E. Attack specific abnormal or non-self agents
45 Transepithelial transport of antigen is important because without it
A. antigen would only be detected by the lymphoid cells in O-MALT
B. antigen would not be detected by the lymphoid cells in the alternative
complement pathway
C. polymeric antibody would be pumped through the epithelial cells
D. vascular addressins would target the wrong cell types
E. none of the above
46 Septic shock is a serious medical condition caused by decreased tissue
perfusion and oxygen delivery as a result of infection. It can cause
multiple organ dysfunction syndrome and death. What is causing the
fatal organ failure?
A. Bacteria in the bloodstream
B. Defective innate immune response
C. Overwhelming immune response
D. Fungi in the bloodstream
E. Defective adaptive immune response
47 This is the ability of an antigen to react specifically with the antibodies
or cells it has provoked.
A.Specificity
B.Immunogenicity
C.Reactivity
D.Valence
E.A and B
48 Which of the following inflammatory signs specifies pain?
A. tumor
B. calor
C. dolor
D. rubor
E. tremor
49 An example of an inflammatory mediator that stimulates vasodilation
is
A. anaphylotoxin
B. histamin
C. C3b
D. collagen
E. interferon
50 The swelling associated with inflammation decreases when the fluid
A. returns to the blood
B. goes into lymph capillaries
C. is excreted in urine
D. is lost as perspiration
E. is transported into macrophages
51 Neutrophil defensins are
A. anti-toxins
B. oxygen-dependent
C. enzymes
D. glycolipids
E. antimicrobial peptides
52 Lysozyme:
A. is a cytoplasmic organelle
B. splits peptidoglycan
C. binds bacterial lipopolysacharide
D. is a proteilytic enzyme
E. is released by natural killer cells
53 Protective function of saliva is based on several mechanisms, including
the presence of enzyme that has bactericidal action and causes lysis of
complex capsular polysaccharides of staphylococci and streptococci.
Name this enzyme:
A. Alpha-amylase
B. Lysozyme
C. Oligo-1,6-glucosidase
D. Collagenase
E. Beta-glucuronidase
54 Lymphocytes and other cells of our body synthesize universal antiviral
agents as a response to viral invasion. Name these protein factors:
A. Interleukin-2
B. Interferon
C. Cytokines
D. Interleukin-4
E. Tumor necrosis factor
55 Which of the following is not an acute phase protein?
A. Fibrinogen
B. C-reactive protein
C. Chondroitin sulfate
D. Mannose binding lectin
E. Serum amyloid P component
56 Interferons:
A. Are divided into 5 main families
B. Are found only in mammalian species
C. Are specific for individual viruses
D. Induce enzyme synthesis in the target cell
E. Only affect infected cells
57 Which of the following is the end product of the complement system?
A. properdin
B. citrulline
C. MAC
D. PAMP
E. anaphylotoxin
58 The complement protein cascade is the same for the classical pathway,
alternative pathway, and lectin pathway beginning with the activation
of
A. C1
B. C2
C. C3
D. C5
E. C6
59 Which of the following statements is TRUE?
A. All three types of interferons have the same effect on the body
B. Alpha interferon promotes phagocytosis
C. Gamma interferon augments macrophage bactericidal activity
D. Alpha interferon acts against specific viruses
E. Beta interferon attacks invading viruses
60 Each of the following is an effect of complement activation EXCEPT
A. increased phagocytic activity
B. bacterial cell lysis
C. opsonization
D. interference with viral replication
E. increased blood vessel permeability
61 The antimicrobial effects of AMPs include all of the following
EXCEPT
A destruction of cell wall
B. lysis of bacterial cells
C. destruction of nucleic acids
D. pore formation in bacterial membranes
E. inhibition of phagocytosis
62 All of the following are effects of histamine EXCEPT
A. vasodilation
B. fever
C. swelling
D. redness
E. pain
63 Which of the following statements is TRUE?
A. Complement increases after immunization
B. All of the complement proteins are constantly active in serum
C. Factors B, D, and P cause cytolysis
D. Complement activity is antigen-specific
E. There are at least thirty complement proteins
64 All of the following are part of the mechanism of action of alpha and
beta interferons EXCEPT
A. they bind to the surface of uninfected cells
B. they are effective for long periods
C. they initiate synthesis of antiviral proteins
D. they disrupt stages of viral multiplication
E. they initiate transcription
65 The alternative pathway for complement activation is initiated by
A. antigen:antibody complex
B. C5C9
C. lipid-carbohydrate complexes and C3
D. factors released from phagocytes
E. factors released from damaged tissues
66 The classical pathway for complement activation is initiated by
A. lipid-carbohydrate complexes and C3
B. C5C9
C. antigen:antibody complex
D. factors released from phagocytes
E. factors released from damaged tissues
67 C3a is
A. opsonin
B. anaphylotoxin
C. prostaglandin
D. pentraxin
E. resistin
68 Activation of C5C9 results in
A. activation of C3
B. fixation of complement
C. lysis of microbial cells
D. phagocytosis
E. inflammation
69 The lectin pathway for complement action is initiated by
A. mannose on host membranes
B. mannose on the surface of microbes
C. lectins of the microbe
D. gram-negative cell walls
E. gram-positive cell walls
70 Which of the following statements about the classical pathway of
complement activation is FALSE?
A. C1 is the first protein activated in the classical pathway
B. The C1 protein complex is initiated by antigen:antibody complexes
C. C3 is not involved in the classical pathway
D. Cleaved fragments of some of the proteins act to increase inflammation
E. C3b causes opsonization
71 Lysozyme and the antibiotic penicillin have similar mechanisms of
action in that they both cause damage to the bacterial
A. cell membrane
B. capsule
C. cell wall
D. DNA
E. ribosomes
72 What direct effect do histamines and leukotrienes have on capillaries?
A. They prevent phagocytes from sticking to the walls of capillaries
B. They allow capillary walls to open and become leaky
C. They decrease the diameter of capillaries
D. All of the above
E. None of the above
73 Which of the complement pathways employs properdin?
A. Classical pathway
B. Alternative pathway
C. Lectin pathway
D. Alternative and lectin pathways all employ properdin
E. Classical, alternative, and lectin pathways all employ properdin
74 Antibodies from adaptive immune responses are used in
A. the lectin complement pathway
B. the alternative complement pathway
C. the classical complement pathway
D. both the classical and alternative pathways
E. both the classical and lectin pathways
75 Which of the following is responsible for cleaving C3?
A. C1
B. C4aC2b
C. C5bC6C7
D. C4bC2a
E. C9
76 Which of the following blood cells function primarily as phagocytes?
A. eosinophils
B. basophils
C. neutriphils
D. lymphocytes
E. platelets
77 A patient with pneumonia has body temperature of 39,2oC. What cells
are the main producers of endogenous pyrogen that had caused such
temperature rise?
A. Neutrophils
B. Eosinophils
C. Monocytes
D. Endotheliocytes
E. Fibroblasts
78 Work in a mine is known to cause inhalation of large amounts of coal
dust. Inhaled coal dust can be detected in the following pulmonary
cells:
A. Capillary endothelial cells
B. Respiratory epithelial cells
C. Secretory epithelial cells
D. Alveolar macrophages
E. Pericapillary cells
79 Cellular composition of exudate largely depends on the etiological
factor of inflammation. What leukocytes are the first to be involved in
the focus of inflammation caused by pyogenic bacteria?
A. Monocytes
B. Neutrophil granulocytes
C. Myelocytes
D. Eosinophilic granulocytes
E. Basophils
80 Alveolar space of acinus was invaded by bacteria that interacted with
the surfactant. This led to the activation of the cells that are localized
in the alveolar walls and on the surface. Name these cells:
A. Alveolocytes type II
B. Alveolocytes type I
C. Endothelial cells
D. Clara cells
E. Alveolar macrophages
81 Natural killer (NK) cells do not:
A. Contain perforins
B. Contain serine proteases
C. Contain tumor necrosis factor (TNF)
D. Kill only by damaging the target cell outer membrane
E. Respond to interferon
82 A signaling molecule from microbes recognized by phagocytes is
A. pyrogen
B. apoptogen
C. PAMP
D. MALT
E. MAC
83 TLRs attach to all of the following EXCEPT
A. flagellin
B. AMPs
C. LPS
D. PAMPs
E. peptidoglycan
84 Which of the following statements about fixed macrophages is FALSE?
A. They are found in certain tissues and organs
B. They develop from neutrophils
C. They are cells of the mononuclear phagocytic system
D. They are mature monocytes
E. They gather at sites of infection
85 Phagocytes utilize all of the following to optimize interaction with
microorganisms EXCEPT
A. trapping a bacterium against a rough surface
B. opsonization
C. chemotaxis
D. lysozyme
E. complement
86 All of the following are true regarding NK cells EXCEPT
A. they are a type of lymphocyte
B. they are found in tissues of the lymphatic system
C. they have the ability to kill infected body cells and some tumor cells
D. they destroy infected body cells by phagocytosis
E. they release toxic substances that cause cell lysis or apoptosis
87 Which of the following cells are innate immunity cells which
participate in immune responses against parasitic helminths?
A. dendritic cells
B. eosinophils
C. lymphocytes
D. monocytes
E. neutrophils
88 What is the role of opsonins?
A. They attract phagocytes to the location of infection
B. They create "handles" that make it easier for the pseudopods of
phagocytes to attach to the microbe invader
C. They are present on the surface of phagocytes
D. They aid in the formation of the phagolysosome
E. All of the above
89 What is a phagolysosome?
A. A vesicle containing only an engulfed invading microorganism
B. A vesicle containing only digestive enzymes and other antimicrobial
compounds
C. The structure that results from the fusion of a phagosome and a
lysosome
D. A protein that covers the surface of an invading microbe, making it
easier for the phagocyte to ingest
E. Immature phagocyte
90 In macrophages and dendritic cells (so-called antigen-presenting cells)
some small parts of the phagocytosed particle are presented to other
cells of the immune system. If you were to draw an arrow leading to
"antigen presentation" as described above, from which word would it
extend?
A. exocytosis
B. digestion
C. ingestion
D. adherence
E. migration
91 Pus is comprised of
A. unused histamines and leukotrienes
B. excess fluid from leaky blood vessels
C. collected cells from damaged tissue
D. dead phagocytes
E. none of the above
92 Neutrophils with defective lysosomes are unable to:
A. engulf microorganisms and other foreign material
B. undergo chemotaxis (migrate)
C. attach to microorganisms and other foreign material
D. produce toxic oxygen products
E. undergo NETosis
93 Which of the following statements concerning phagocytosis is TRUE?
A. Phagocytes cannot ingest microorganisms unless they are coated with
antibodies
B. Engulfment always requires opsonization
C. Phagocytes ingest microorganisms by using protein transporters that are
specific for the bacteria
D. Bacteria are digested when the phagosome fuses with a lysosome
E. Phagocytes are only lymphoid cells
94 Natural killer cells
A. mature in thymus gland
B. recognize and destroy tumors in a specific manner
C. require prior exposure to tumor cells to be effective against the tumor
cells
D. do not need to be activated by macrophages that present foreign tumor
antigens
E. derive from myeloid progenitors
95 Monocytes and macrophages are both which type of leukocytes?
A. polymorphonuclear phagocytes
B. mononuclear phagocyte system
C. organ-specific phagocytes
D. fixed phagocytes
E. circulating phagocytes
96 Which of the following types of white blood cells contain preformed
granules of vasoactive amines?
A. monocytes
B. neutrophils
C. macrophages
D. mastocytes
E. B-lymphocytes
97 Primary cells of innate immunity do not include:
A. Epithelial barrier cells
B. Phagocytic cells (neutrophils, macrophages)
C. Natural killer cells
D. Myocytes
E. A and D
98 ________ undergo chemotaxis. They are the predominant phagocytic
cell, which removes debris via phagocytosis
A. Eosinophils
B. Neutrophils
C. Natural killer cells
D. Platelets
E. Monocyte/macrophage
99 Dendritic cells are characterized by
A. The presence of TCR
B. Expression of CD3
C. Expression of IgM molecules
D. Their ability to release histamine
E. Their interface between the innate and adaptive immune systems
100 An essential part of the innate immune system is phagocytosis followed
by destruction of the phagocytosed particle/microbe followed by
antigen- presentation. What cells are considered professional
phagocytes?
A. Granulocytes, monocytes, macrophages, dendritic cells, B cells
B. Granulocytes, monocytes, macrophages, dendritic cells
C. Granulocytes, monocytes, macrophages, B cells
D. Antigen presenting cells
E. All innate immunity cells
101 Which of the following is NOT TRUE concerning the secondary
immune response?
A. Low IgM titre
B. High IgG
C. Low affinity Ab
D. Lag-phase lasts 5-7 hours
E. High Ab affinity
102 Clonal selection occurs when antigen is encountered by:
A. Basophils
B. Eosinophils
C. Mast cells
D. Neutrophils
E. B-cells
103 The major function of plasma cells is to secrete large amounts of
antibody. Which of the following statements regarding plasma cells is
correct?
A. Are derived from T-cells
B. Develop into B-cells
C. Have a highly developed rough endoplasmic reticulum
D. Have a thin layer of cytoplasm
E. Secrete large amounts of gamma interferon
104 After the contact with foreign antigens, body produces specific
antibody. These specific antibodies are readily detectable in serum
following primary contact with antigen after:
A. 10-15 min
B. 1-2 h
C. 5–7 days
D. 3–5 weeks
E. Only following a second contact with antigen
105 Which immunoglobulin is found secreted in the secondary response?
A. IgM
B. IgG and IgM
C. IgE
D. IgG
E. IgD
106 This isotype of antibodies is mainly found in sweat, tears, breast milk
and GI secretions.
A.IgG
B.IgA
C.IgM
D.IgD
E.IgE
107 IgG binding to neutrophils cells is mediated by
A. Fc-dependent cellular homing mechanisms
B. sensitization of Mast cells and basophils
C. Fc receptors specific for IgG
D. ICAM’s
E. None of the above
108 IgD participates in antigen recognition by
A. immature T cells
B. NK-cells
C. macrophages
D. B-cells
E. A and D
109 Antibody affinity is determined by the amino acid sequence in
A. the constant regions of the immunoglobulin molecule
B. the variable regions of the immunoglobulin molecule
C. the Fc of the immunoglobulin molecule
D. the J-chain
E. A, C and D
110 Avidity is important because
A. Fc receptor binding depends on it
B. it amplifies the binding strength of low affinity Fab’s
C. G-protein-mediated signal transduction will not occur without it
D. it result in the activation of high affinity antibody-producing clones
E. none of the above
111 The lag phase of the secondary response is shorter than the primary
response because
A. the assays for detecting a primary response are not as sensitive
B. the primary response requires considerable cell proliferation and
differentiation to
achieve a critical mass of cells to produce immunity
C. of the lack of cytokines produced during the primary response
D. A and B
E. all of the above
112 The lag phase of the booster response is
A. very short due to the lack of antigen presenting cells
B. very short, due to memory cells
C. very short when dendritic cells are absent
D. very short, due to the presence of accessory cells
E. prolonged
113 Which of the following gene clusters do NOT contribute to antigen
binding
A. VL
B. VH
C. DH
D. CH
E. JH
114 B cell DO NOT express
A. CD19
B. MHC II
C. MHC I
D. CD8
E. CD20
115 A given Ig isotype is
A. A light chain constant region encoded by allelic genes
B. A heavy chain variant encoded by allelic genes
C. Present in all normal individuals
D. A collection of hypervariable region epitopes recognized by an anti-
idiotype
E. Monoclonal
116 IgG
A. Appears early in the immune response
B. Kills bacteria directly
C. Is important in mucosal immunity
D. Usually exists as a pentamer
E. Opsonizes bacteria
117 IgA in seromucus secretions
A. Has no J-chain
B. Has no secretory piece
C. Is dimeric
D. Cannot bind to virus
E. Activates the classical complement pathway
118 The cytokine which is most involved in the class switch to IgE
production is
A. IL-2
B. IL-4
C. IL-8
D. IL-10
E. none of the above
119 The molecules mediating signal transduction following antigen binding
to cell surface immunoglobulin on a B-cell are called:
A. Ig Fc
B. Ig-alpha and Ig-beta
C. MHC
D. CD4
E. CD3
120 Carbohydrate is always present on normal immunoglobulin:
A. H-chains
B. L-chains
C. Fab-fragments
D. H and L chains
E. none of the above
121 Which of the following cells may eventually differentiate into an
immunoglobulin-bearing memory cell?
A. naïve B-cell
B. naïve T-cell
C. hematopoietic stem cell
D. A and B
E. activated B-cell
122 All of the following are true with respect to IgM antibodies EXCEPT
which one
A. they fix complement
B. they occur on the surface of lymphocytes
C. they predominate in the primary response to antigen
D. they are glycoproteins
E. they mediate allergic reaction
123 All of the following are true with respect to IgE molecules, EXCEPT
which one?
A. They are the principal immunoglobulin class involved in allergic
reactions
B. They are involved in mediating anti-parasitic immune responses
C. They will cross the placenta and fix complement
D. They can effect the release of histamine and other chemical mediators
E. They are the least abundant immunoglobulin in the serum
124 All of the following are true about antibodies, EXCEPT which one?
A. They fix complement
B. They occur on the surface of B-lymphocyte
C. They predominate the primary immune response to antigen
D. They are glycoproteins
E. They are molecule with a single, defined amino acid sequence
125 The immunoglobulin Joining chain (J-chain) is
A. only produced by T-Cells
B. only produced by neutrophils
C. associated with only multimeric forms of IgM and IgA
D. associated with IgE for histamine release
E. only produced by mast cells
126 Each of the following is the property of T cell epitopes EXCEPT:
A. protein
B. MHC 1: 9 amino acids long
C. MHCII: 12 amino acids long
D. part of native unprocessed antigen
E. MHCII: 15 amino acids long
127 What is the role of helper T cells in the adaptive immune response?
A. Helper T cells directly kill infected host cells
B. Helper T cells activate B cells and cytotoxic T lymphocytes to kill
infected host cells
C. Helper T cells produce and secrete antibodies
D. Helper T cells phagocytize bacteria and viruses
E. None of the above
128 Which of the following is NOT a step used by cytotoxic T cells to kill
infected host cells?
A. Recognition of infected host cell using its TCR
B. Secretion of perforin
C. Secretion of granzyme
D. Recognition of infected host cell using its CD4 glycoprotein
E. All statements are correct
129 Which pair of molecules do NOT directly interact with one another?
A. CD4 and MHC-II
B. BCR and TCR
C. CD8 and MHC-I
D. BCR and epitope
E. PAMP and PRR
130 A person who has AIDS contracts rare and often life-threatening
infections because their helper T cell count is so low. Which of the
following components of the immune response still respond to antigen
despite the low helper T cell count?
A. Clonal expansion and antibody production
B. Clonal selection of B cells
C. Activation of cytotoxic T cells
D. Apoptosis of infected host cells
E. All of the above
131 ___ express CD3 and CD4 molecules
A. All T cells
B. Th1 and Th2 cells
C. NK and B cells
D. TH1 and Tc cells
E. Ts and Tc cells
132 Elevated levels of interleukin-1β would result in which of the following
responses?
A. increased activity of cytotoxic (killer) T cells
B. increased production of antibodies by plasma cells
C. increased activity of helper T cells
D. increased body temperature
E. increased activity of NK cells
133 Class-2 MHC molecules are produced by all of the following cells
EXCEPT
A. peritoneal macrophages
B. dendritic cells
C. helper T cells
D. B-lymphocytes
E. alveolar macrophages
134 These cells display CD4 in their membrane and are associated with
MHC class II molecules.
A.Cytotoxic T cells
B.Helper T Cells
C.Supressor T Cells
D.Cortical thymocytes
E.B cells
135 T Cells secrete this substance that is used to fragment DNA.
A.Perforin
B.Tumor antigen
C.Interferons
D.granzyme
E.Toxin T
136 This is a self-responsive cell that is inactive.
A.Immunocompetent cell
B.Hybridoma cell
C.Naive T-cell
D.Anergy cell
E.Natural killer cell
137 Follicular dendritic cells
A. are presents in the skin
B. engulf extracellular pathogens
C. participate in antigen-presentation to T-cells
D. participate in affinity maturation
E. All of the above
138 Protection against microorganisms inside cells is provided by
A. macrophages
B. antibodies
C. complement
D. T-cells
E. all of the above
139 T cell antigen receptor
A. Recognizes conformational epitopes on the native antigen
B. Has Ig light chains
C. Is made up of a heavy chain and beta-2 microglobulin
D. Are associated with Igαand Igβto form a complex
E. Recognize epitopes on linear peptides associated with MHC molecules
140 The class I MHC processing pathway primarily
A. Processes antigen from the extracellular environment
B. Processes antigens that are present in the cytosol
C. Generates peptides, complexes them with class I MHC molecules for
presentation to helper T cells
D. Generates peptides, complexes them with class I MHC molecules for
presentation to NK cells
E. Is involved in the process of the antibody response
141 ___ express CD3 and CD8 molecules
A. All T cells
B. Th1 and Th2 cells
C. NK and B cells
D. TH1 and Tc cells
E. Ts and Tc cells
142 Cell with specific killing effects is
A. LAK
B. CTL
C. NK
D. A and C
E. all of the above
143 During the cell interactions involved in generating a cytotoxic T-cell
response, the Thelper cell receives the necessary “Signal 2” from an
antigen-presenting cell through which of the following interactions?
A. TCR with MHC Class I
B. IgD with antigen
C. A and B
D. B7 with CD28
E. IL-2 with IL-2R
144 The CD3 complex of the T-cell receptor
A. binds complement
B. functions to transduce a signal to the cell's interior following binding of
complexed antigen
C. causes the histamine
D. mediates Immunoglobulin class switching
E. results in the glycosylation of jmmunoglobuiins
145 Cytokines are produced by cells of the immune system in response to
various physiological stimuli
A. modulate cell function through subsequent cell differentiation or cell
proliferation
B. facilitate cell lysis
C. cause glycosylation of Immunoglobins
D. facilitate Ab generation
E. all of the above
146 Which of the following is NOT true of CD4 and CD8 cell markers?
A. These are both surface glycoproteins expressed on T-cells
B. These serve to distinguish different types of T-cells, e.g., helper,
suppressor and cytotoxic, from each other
C. These are not found associated with immunoglobulins
D. The CD4 proteins serve both to mediate T-cell helper function as well as
the receptor for the AIDS virus.
E. Both of the markers are present on ALL T-cells
147 Which of the following is NOT true of the ability of the T-cell receptor
(TCR) to specifically recognize antigen?
A. The antigen must be "processed" first by an accessory cell of immune
system in order for it to bind to the TCR
B. The recognition of the antigen by the TCR can mediate helper,
suppressor or cytotoxic function
C. The recognition of antigen by the TCR can result in cytokine secretion
and/or an increase in cell proliferation within the immune system
D. The antigen is recognized by the T3-TCR complex only when it is
associated with a protein of the major histocompatibility complex
E. Only the alpha chain of the TCR is necessary for antigen
148 The major histocompatibility complex proteins function to
A. degrade CD4 and CD8 polypeptides
B. bind antibody for lymphokine production
C. bind complement for cell lysis
D. bind antigen fragments for presentation to T-cells
E. none of the above
149 The domain unit of an immunoglobulin or T-cell antigen receptor
A. only include the variable regions
B. only recognizes the paratope
C. only fixes complement
D. is typically about 110 amino acids long
E. A and D
150 The T-cell receptor
A. Is composed of four polypeptide chains
B. is secreted into the plasma by the T-cell
C. Is the recognition element of the humoral arm of the immune system
D. recognizes antigen fragments via the alpha and beta chains
E. None of the above
151 A 13-year-old boy presents with eczematous rashes on his shins and
torso. Anamnesis states cases of otitis, pneumonia, and furuncles in the
patient. Blood test: platelets - 70 · 109/l, low activity of T helper and T
suppressor cells, low IgM, with normal IgA and IgG. What
immunodeficient disease does this boy have?
A. DiGeorge syndrome
B. Louis-Bar syndrome (Ataxiatelangiectasia)
C. Severe combined immunodeficiency (Swiss type)
D. Wiskott-Aldrich syndrome
E. Chediak-Higashi syndrome
152 Parents of a 5-year-old child report him to have frequent colds that
develop into pneumonias, presence of purulent rashes on the skin.
Laboratory tests have revealed the following: absence of
immunoglobulins of any type; naked cells are absent from the lymph
nodes punctate. What kind of immune disorder is it?
A. Autosomal recessive agammaglobulinaemia (Swiss type)
B. X-linked hypogammaglobulinemia (Bruton type agammaglobulinemia)
C. Hypoplastic anemia
D. Agranulocytosis
E. Louis-Barr syndrome
153 Several inherited deficiencies in the complement system occur in
humans. Which of the following would be the MOST severe?
A. deficiency of C8
B. deficiency of C7
C. deficiency of C6
D. deficiency of C5
E. deficiency of C3
154 This is characterized by the inability of the immune system to protect
the body from a pathogen.
A. Graft versus host disease
B. Allergy
C. Autoimmune disease
D. Immunodeficiency diseases
E. Acute graft rejection
155 A deficiency of complement component C6 will affect the:
A. classical pathway
B. MBL-pathway
C. alternative pathway
D. only A and B
E. all complement pathways
156 Human RAG-1 Deficiency is caused by an autosomal recessive
mutation which destroys the Recombinase Activating Genes. This
results in a lack of ability to produce:
A. immunoglobulin
B. cytokines
C. T-cell receptor
D. A and C
E. none of the above
157 Individuals unable to make the J protein found in certain
immunoglobulins would be expected to have frequent infections of the
A. brain
B. blood
C. liver
D. pancreas
E. intestinal tract
158 HIV causes AIDS by attacking and destroying
A. CD20+ cells
B. CD16+ cells
C. CD1d+ cells
D. CD4+ cells
E. A and D
159 People who have a primary immunodeficiency disease are most
at risk for which of the following?
A. heart diseases
B. cancer
C. diabetes
D. infections
E. C and D
160 Which of the following diseases does NOT associated with phagocytosis
and opsonization defect?
A. Chronic granulomatous disease
B. Chedial-Higashi syndrome
C. Leukocyte adhesion deficiency
D. SCID
E. A and C
161 Which of the following diseases does NOT associated with B-cell
defect?
A. Selective IgA deficiency
B. DiGeorge’s syndrome
C. X-linked Bruton’s agammaglobulinemia
D. Х-linked hyper-IgM-syndrom
E. A and B
162 Defects in neutrophil NADPH oxidase system produce:
A. Streptococcal infection
B. Chediak-Higashi disease
C. Hashimoto's disease
D. Chronic granulomatous disease
E. Leukocyte adhesion deficiency
163 Paroxysmal nocturnal hemoglobulinuria results from deficiency in:
A. Myeloperoxidase
B. Decay accelerating factor (DAF)
C. Classical pathway C components
D. C1 inhibitor
E. C8
164 X-linked agammaglobulinemia results from a mutation in:
A. IFNγ receptor
B. The CIITA promoter protein
C. An HLA gene
D. CD40L (CD154)
E. Bruton's tyrosine kinase gene
165 Di George syndrome results from a defect in:
A. Purine nucleoside phosphorylase
B. WASP
C. Thymus development
D. DNA repair
E. CD3
166 Poor skin tests to a range of microbial antigens such as tuberculin and
mumps indicate a deficiency of:
A. NK cells
B. T-cells
C. B-cells
D. phagocytes
E. opsonization
167 HIV binds to:
A. TNF-receptor
B. IL-2 receptor
C. NF kappa B
D. Reverse transcriptase
E. CD4
168 Primary immunodeficiency producing susceptibility to infection by
viruses and molds is due to:
A. B-cell deficiency
B. T-cell deficiency
C. Phagocyte deficiency
D. complement deficiency
E. eosinophil deficiency
169 Deletions in the T-cell CD154 (CD40L) gene produce:
A. Congenital X-linked agammaglobulinemia
B. IgA deficiency
C. Wiskott–Aldrich Syndrome
D. Deficiency in cytotoxic T-cell activity
E. The hyper–IgM syndrome
170 Which of the following does NOT associated with the development of
secondary immunodeficiency?
A. Viral infection
B. Lymphoproliferative disorders
C. Cytotoxic drugs
D. Mutation in RAG-genes
E. Low iron diet
171 What of the following is NOT TRUE about primary
immunodeficiencies?
A. it is a group of disorders characterized by an impaired ability to produce
normal immune response
B. they are caused by the mutations in genes involved in the development
and functions of the organs, cells and humoral factors (soluble molecules)
of immune system
C. Usually diseases of infancy and childhood
D. Malnutrition is the most common cause of the primary
immunodeficiencies
E. A and B
172 What of the following is NOT TRUE about secondary
immunodeficiency?
A. can be a consequence of prolonged use of cytotoxic agents
B. can be associated with physiologic immunosuppression in elderly
persons
C. usually diseases of infancy and childhood
D. can be caused by the malnutrition
E. B and C
173 Which primary immunodeficiency is the most common in the world?
A. T-cell defects
B. B-cell defects
C. complement defects
D. phagocytosis system defects
E. C and D
174 Examples of primary immunodeficiency disorders do not include
A. SCID
B. CVID
C. AID
D. “boy in a bubble” disease
E. A and C
175 Which of the following diseases does NOT associated with T-cell
defect?
A. DiGeorge’s syndrome
B. Ataxia-teleangiectasia
C. Chronic mucocutaneous candidiasis
D. Х-linked hyper-IgM-syndrom
E. A and B
176 A 38-year-old man, who has been suffering from systemic lupus
erythematosus for 3 years, developed diffuse renal lesions accompanied
by massive edemas, marked proteinuria, hyperlipidemia, and
dysproteinemia. What is the most likely mechanism of proteinuria
develoment in this case?
A. Increased blood proteins
B. Inflammatory damage to the nephrons
C. Ischemic damage to the tubules
D. Autoimmune damage to the nephrons
E. Morbid affection of the urinary tracts
177 In Graves' autoimmune disease of the thyroid,
A. normally "hidden" antigens of the thyroid escape surveillance to
stimulate the autoimmune response
B. self-antigens of the thyroid cells became the target of the autoimmune
response
C. thyroid cells produce class-2 MHC molecules that stimulate
autoantibodies that bind to the TSH receptor
D. thyroid soluble self-antigens stimulate autoimmune response
E. all of the above
178 Which statement about systemic lupus erythematosus (SLE) is false?
A. SLE results from the abnormal combination of self-antigens and
autoantibodies
B. SLE victims produce antibodies against their own DNA and nuclear
protein
C. SLE is an autoimmune disease that results in the formation of immune
complexes throughout the body
D. SLE is characterized by unexpected immediate hypersensitivity
reactions
E. SLE is an systemic autoimmune disease
179 An isograft is a tissue graft donated by
A. parent
B. the same person
C. an identical twin
D. an unrelated person
E. an organism of different specie
180 This type of graft is taken from a different animal species and is never
successful:
A. autograft
B. isograft
C. xenograft
D. allograft
E. A and D
181 Which of the following is NOT an autoimmune disease:
A. type I diabetes
B. multiple sclerosis
C. rheumatoid arthritis
D. AIDS
E. SLE
182 In Graves' disease, an antibody causing hypersensitivity may be
present and is directed against:
A. Thyroglobulin
B. Thyroglobulin
C. Thyroid stimulating hormone (TSH) receptor
D. Acetylcholine receptor
E. Thyroxine
183 A graft between members of the same species is termed an
A. isograft
B. allograft
C. autograft
D. xenograft
E. none of the above
184 Graft vs host disease often accompanies transplantation of:
A. cartilage
B. kidney
C. bone marrow
D. heart
E. pancreas
185 The very rapid response to a second allogeneic graft is:
A. Transferred by IgA
B. Dependent on minor histocompatibility antigens
C. Transferred by macrophages to a naive recipient
D. Transferred by platelets
E. Specific for antigens of the major histocompatibility complex (MHC)
186 Hyperacute graft rejection is caused by:
A. Circulating immune complexes
B. CD4 positive lymphocytes
C. CD8 positive lymphocytes
D. Platelets
E. Preformed antibodies
187 Which of the following allogeneic grafts does NOT require
immunosuppression?
A. kidney
B. heart
C. liver
D. bone marrow
E. cartilage
188 Which of the following is a non-organ-specific (systemic) autoimmune
disease?
A. Myasthenia gravis
B. Systemic lupus erythematosus (SLE)
C. Hashimoto's thyroiditis
D. Pernicious anemia
E. Insulin-dependent diabetes mellitus
189 Which of the following antibodies are of most use for the diagnosis of
pernicious anemia?
A. Anti-thyroid peroxidase
B. Anti-parietal cell
C. Anti-nuclear
D. Anti-IgG Fc
E. Anti-TSH receptor
190 The antiphospholipid syndrome is associated with:
A. Infertility
B. Severe anemia
C. Wegener's granulomatosis
D. Raised platelet levels
E. Recurrent fetal loss
191 Neonatal myasthenia gravis is thought to be caused by:
A. An inherited genetic defect
B. Transplacental transfer of maternal IgG against the TSH receptor
C. Anti-idiotype to maternal IgG
D. Transplacental transfer of maternal IgG against the acetylcholine
receptor
E. Maternal T-cells transferred across the placenta
192 In celiac disease there is T-cell sensitivity to:
A. Vitamin B12
B. Gluten
C. β-adrenergic receptors
D. Gastric H+-K+ dependent ATPase
E. Myelin basic protein
193 In type 1 diabetes (insulin-dependent diabetes mellitus), the target of
the autoimmune attack is:
A. All of the cells in the islets of Langerhans
B. The β-cells in the islets of Langerhans
C. The somatostatin-producing cells in the islets of Langerhans
D. The glucagon-producing cells in the islets of Langerhans
E. Cells throughout the body which have an insulin receptor
194 In direct allorecognition,
A. donor alloantigen is internalized, processed, and presented as peptides in
the context of recipient MHC class II by recipient APCs to recipient CD4 T
cells
B. recipient CD4 T cells recognize intact MHC class II alloantigen. This is
present on donor APCs migrating from the graft and on MHC class II–
expressing donor parenchymal cells
C. recipient B-cells recognize CD1c present on donor APCs
D. All of the above
E. None of the above
195 Genes encoding cell surface glycoproteins that are required for antigen
presentation to T-cells and also responsible for rapid graft rejection is
called as
A. B-cell complex
B. T-cell complex
C. receptor complex
D. MHC complex
E. none of the above
196 What is the role of class II MHC proteins on donor cells in graft
rejection?
A. They are the receptors for interleukin-2, which is produced by
macrophages when they attack the donor cells
B. They are recognized by helper T cells, which then activate cytotoxic T
cells to kill the donor cells
C. They induce the production of blocking antibodies that protect the graft
D. They induce IgE which mediates graft rejection
E. All of the above
197 Chronic graft rejection is mediated by
A. Type I and type II hypersensitvity reaction
B. Type II hypersensitvity reaction
C. Type II and type III hypersensitvity reaction
D. Type III and type IV hypersensitvity reaction
E. Type I and III hypersensitvity reactions
198 Acute graft rejection is mediated by
A. Type I hypersensitvity reaction
B. Type II hypersensitvity reaction
C. Type III hypersensitvity reaction
D. Type IV hypersensitvity reaction
E. Type I and III hypersensitvity reactions
199 Reiter's syndrome (arthritis) is an example of
A. Autoimmune complications of immunodeficiencies
B. Paraneoplastic syndromes
C. Neonatal autoimmune pathology
D. Drug-induced autoimmune disorders
E. Autoimmune complications of infectious diseases (cross-reactivity)
200 Immunopathogenic mechanism of autoimune hemolytic anemia is
A. Type I hypersensitvity reaction
B. Type II hypersensitvity reaction
C. Type III hypersensitvity reaction
D. Type IV hypersensitvity reaction
E. A and C
201 A 3-year-old child has eaten some strawberries. Soon he developed a
rash and itching. What was found in the child’s leukogram?
A. Monocytosis
B. Hypolymphemia
C. Neutrophilic leukocytosis
D. Eosinophilia
E. Lymphocytosis
202 After sensitization a test animal received subcutaneously a dose of
antigen. At the site of injection a fibrinous inflammation developed
with alteration of vessel walls, basal substance, and fibrous structures
of connective tissue. The inflammation took form of mucoid and
fibrinoid degeneration, fibrinoid necrosis. What immune response
occurred in the test animal?
A. Delayed hypersensitivity
B. Immediate hypersensitivity
C. Transplantation immune reaction
D. Normergic reaction
E. Granulomatosis
203 A 12-year-old child developed nephritic syndrome (proteinuria,
hematuria, cylindruria) 2 weeks after a case of tonsillitis, which is a
sign of affected glomerular basement membrane in the kidneys. What
immune mechanism is the most likely to cause the basement membrane
damage?
A. Reaginic
B. Granulomatous
C. Antibody-mediated
D. Immune complex
E. Cytotoxic
204 A 22-year-old woman ate some seafood. 5 hours later her torso and
distal parts of her limbs developed small itchy papules which were
partially fused together. One day later the rash disappeared
spontaneously. Specify the hypersensitivity mechanism underlying
these changes:
A. Systemic anaphylaxis
B. Atopy (local anaphylaxis)
C. Cellular cytotoxicity
D. Immune complex hypersensitivity
E. Antibody-dependent cell-mediated cytolysis
205 On examination of a patient with disease onset 5 days ago the doctor
suspected tularemia and prescribed the patient tularin
intracutaneously. What is the purpose of this drug administration in
the patient?
A. Treatment
B. Prevention
C. Allergy diagnostics
D. Treatment evaluation
E. Prognosis for the disease
206 What condition may develop 15-30 minutes after re-administration of
an antigen as a result of the increased level of antibodies, mainly IgE,
that are adsorbed on the surface of target cells, namely tissue basophils
(mast cells) and blood basophils?
A. Serum sickness
B. Antibody-dependent cytotoxicity
C. Delayed-type hypersensitivity
D. Immune complex hyper responsiveness
E. Anaphylaxis
207 30 minutes after drinking mango juice a child suddenly developed a
local swelling in the area of the soft palate, which impeded swallowing
and, eventually, respiration. Mucosa of the swollen area was hyperemic
and painless. Blood test revealed moderate eosinophilia. Body
temperature was normal. Anamnesis states that the elder sister of the
child has been suffering from bronchial asthma attacks. What kind of
edema has developed in the child?
A. Cardiac
B. Inflammatory
C. Allergic
D. Alimentary
E. Hepatic
208 Several minutes after a dentist administered novocaine for local
anaesthesia of a patient’s tooth, the following symptoms sharply
developed in the patient: fatigue, skin itching. What kind of allergic
reaction is this pathology?
A. Stimulating
B. Cytotoxic
C. Anaphylactic
D. Cell-mediated immune reaction
E. Immune complex
209 A 10-year-old child had cut his leg with a glass shard, when playing,
and was delivered to the outpatient department to receive anti-tetanus
serum. To prevent development of anaphylactic shock the serum was
introduced by Bezredka method. This method of organism
hyposensitization is based on the following mechanism:
A. Stimulation of antigen-specific IgG2
B. Blocking of mast cell mediators synthesis
C. Stimulation of immune tolerance to antigen
D. Binding of mast cell-fixed IgE
E. Stabilization of mast cell membranes

210 During blood transfusion a patient has developed intravascular


erythrocyte hemolysis. What kind of hypersensitivity does the patient
have?
A. I type (anaphylactic)
B. II type (antibody-dependent)
C. III type (immune complex)
D. IV type (cellular cytotoxicity)
E. IV type (granulomatous)

211 Which of the following conditions belongs to that form of allergy


known as delayed hypersensitivity?
A. allergic rhinitis (runny or stuffy nose)
B. conjunctivitis (red eyes)
C. allergic asthma (difficulty breathing)
D. contact dermatitis (poison ivy)
E. food allergy (eggs, milk etc.)
212 Sensitization to foods is minimized by secretory IgA antibodies by
A. the inflammatory response that occurs in the presence of food, these
antibodies and complement
B. destroying the antigen presenting cells that would normally present the
food antigens to T cells in the gut
C. blocking the penetration of intact food products into the gut
D. All of the above
E. None of the above

213 Contact dermatitis generally occurs against substances that are too
small to induce an immune response. How do these substances induce
an immune response?
A. These substances form depots and are then slowly released into the
blood
B. These low molecular weight substances react with liver enzymes and are
difficult to eliminate
C. These substances bind to tissues and cells, resulting in a larger total
antigenic size which can then stimulate an immune response
D. The substances trigger the complement cascade and cause neutrophils to
accumulate and to serve as antigen presenting cells
E. A and B

214 Your patient tests positive for the tuberculin antigen. You send him for
a chest x-ray because
A. he may have other lung infections
B. the tuberculin test is only presumptive, indicating that he has been
exposed to a tuberculosis antigen
C. you are looking for fluid in his lungs due to inflammation caused by the
bacillus
D. A and C
E. all of the above

215 Rhesus hemolytic disease of the newborn involves


A. IgE
B. Cytokine release from T-cells
C. A and B
D. Soluble immune complexes
E. Antibody to cell surfaces
216 Immediate hypersensitivity reactions
A. are experienced if the antigen is an allergen
B. result from histamine and other chemical mediator release
C. mediated through antigen specific IgE and mast cells
D. are a consequence of antigen-antibody reactions
E. All of the above

217 Allergic contact dermatitis following skin contact with poison ivy
would normally lead to:
A. acute hypersensitivity
B. anaphylactic shock
C. immediate hypersensitivity
D. IgG-dependent hypersensitivity
E. systemic autoimmune disorder

218 Type I hypersensitivity can be blocked using:


A. an IgA myeloma
B. a myeloma protein of mixed antibody class
C. antiserum
D. sodium cromoglycate
E. histamine
219 Which type of hypersensitivity cannot be transferred with serum
antibody?
A. Type I
B. Type II
C. Type III
D. Type IV
E. A and B
220 Anaphylaxis can be triggered by cross-linking of IgE receptors on:
A. monocytes
B. mast cells
C. B-cells
D. neutrophils
E. eosinophils
221 Rhesus hemolytic disease of the newborn involves:
A. IgE
B. antibodies to cell surface
C. soluble immune complexes
D. Cytokine release from T-cells
E. Stimulatory antibodies
222 The Arthus reaction is characterized by an intense infiltration by
A. mast cells
B. eosinophils
C. neutrophils
D. macrophages
E. Langerhans cells
223 Type IV hypersentivity is often referred to as:
A. immediate
B. delayed
C. anaphylactic
D. anergic
E. allotypic
224 The injection of tuberculin into the skin of a sensitized individual
elicits:
A. Immune complex glomerulonephritis
B. Jarisch-Herxheimer reaction
C. Isohemagglutinins
D. Jones-Mote sensitivity
E. Mantoux reaction
225 The major effector molecules involved in type IV hypersensitivity
reactions are:
A. antibodies
B. complement
C. cytokines
D. prostaglandins
E. 5-hydroxytryptamine (5-HT)

226 In a village a case of anthrax has been registered. Medical services


began epidemiologically indicated specific prophylaxis of population
against anthrax. What preparation was used for this purpose?
A. Anatoxin
B. Inactivated vaccine
C. Chemical vaccine
D. Genetically engineered vaccine
E. Live vaccine

227 Which of the following is not a characteristic of passive immunity?


A. Duration of resistance is days to weeks
B. It can be initiated by injection of antibodies
C. Source of antibodies is the person inoculated
D. Time to develop resistance is immediately after an injection
E. All statements are characteristic of passive immunity

228 Which of these is not associated with adjuvants?


A. forms an antigen depot
B. provides non-specific T cell stimulation
C. activates antigen-presenting cells
D. activates the complement cascade
E. none of the above
229 Factors may influence the induction of an artificial immunity include
A. The nature of an antigen
B. The route of administration
C. The dose of antigen
D. Maturity of the immune system
E. All of the above
230 During the intracellular phase viruses are detected by cytosolic
immune proteins as well as by cytotoxic T cells recognizing viral
peptides presented on MHC class I. Why are live viruses better than
killed viruses as vaccines?
A. Killed viruses are degraded in lysosomes and are therefore inefficiently
presented by the MHC class I pathway
B. Killed viruses are recognized and destroyed faster since they can’t evade
immunity
C. Killed viruses do not activate complement
D. Only replicating viruses contain ligands that activate the Toll-like
receptors in antigen presenting cells
E. Live viruses infect non-immune cells, whereas killed viruses only enter
phagocytes

231 After the initial immune response subsides, B cells that patrol body
tissues for long periods of time
A. are called helper T cells
B. are called memory cells
C. develop into phagocytes
D. can’t react to the original antigen
E. A and B
232 Vaccines are
A. given to establish B-cell and T-cell memory
B. can consist of dead or weakened pathogens
C. effective in preventing a dangerous infection caused by a pathogen
D. all of the above
E. A and C
233 The specific type of acquired immunity that a fetus obtains from
maternal antibodies that cross the placenta is called
A. artificially acquired passive immunity
B. artificially acquired active immunity
C. naturally acquired passive immunity
D. naturally acquired active immunity
E. naturally acquired innate immunity

234 Which of the following descriptions is a characteristic part of the


secondary response?
A. It represents the response to an initial exposure of that pathogen
B. The latent period between the secondary exposure and the appearance of
antibodies is about 5-10 days
C. Antibody concentrations during this response reach a plateau in a few
days and decline after a few weeks
D. The production of antibody can reach a maximum in less than two hours
and is maintained for a long time
E. All of the above
235 Vaccines are NOT for:
A. measles
B. tetanus
C. snake bites
D. diphtheria
E. pertussis
236 The process of weakening a pathogen is called
A. vaccination
B. attenuation
C. immunization
D. virulence reduction
E. none of the above
237 The process of introduction of weakened pathogen into human body is
called
A. opsonization
B. attenuation
C. immunization
D. vaccination
E. none of the above
238 A vaccine can be
A. an antigenic protein
B. weakened pathogen
C. live attenuated pathogen
D. all of the above
E. none of the above
239 Which of the following statement is TRUE regarding vaccination?
A. it is a method of passive immunization
B. it is a method of active immunization
C. it is a method of active artificial immunization
D. it is a method of passive artificial immunization
E. none of the above
240 Active immunity may be gained by
A. natural infection
B. vaccines
C. toxoids
D. all of the above
E. A and B
241 Conjugate vaccine consists of
A. protein component of the microorganism
B. whole microorganism destroyed by heat, chemicals, radiation or
antibiotics
C. live microorganism modified to be less deadly
D. poorly immunogenic component of the microorganism paired with
highly immunogenic protein
E. pathogens that infect other animals but do not cause disease or cause
mild disease in human
242 Heterotypic vaccine consists of
A. protein component of the microorganism
B. inactivated toxic compounds
C. poorly immunogenic component of the microorganism paired with
highly immunogenic protein
D. live microorganism modified to be less deadly
E. pathogens that infect other animals but do not cause disease or cause
mild disease in human
243 Subunit vaccine consists of
A. pathogens that infect other animals but do not cause disease or cause
mild disease in human
B. live microorganism modified to be less deadly
C. poorly immunogenic component of the microorganism paired with
highly immunogenic protein
D. protein component of the microorganism
E. inactivated toxic compounds
244 Multivalent vaccine consists of
A. pathogens that infect other animals but do not cause disease or cause
mild disease in human
B. multiple antigens, strains of microorganisms
C. poorly immunogenic component of the microorganism paired with
highly immunogenic protein
D. inactivated toxic compounds
E. protein component of the microorganism

245 Route of the administration for polio vaccine is


A. subcutaneous
B. intravenous
C. intramuscular
D. intradermal
E. oral
246 Route of the administration for DTaP (Diphtheria, Tetanus, Pertussis)
vaccine is
A. oral
B. intravenous
C. intradermal
D. intramuscular
E. subcutaneous
247 Route of the administration for BCG vaccine is
A. intravenous
B. oral
C. intradermal
D. subcutaneous
E. intramuscular
248 Which of the following route of the vaccine administration is non-
invasive?
A. intravenous
B. oral
C. intranasal
D. rectal
E. B, C, D
249 The concept of vaccination was first developed by
A. Louis Pasteur
B. Carl Landsteiner
C. Joseph Meister
D. Macfarlane Burnet
E. Edward Jenner
250 Which statement regarding the active immunization is TRUE?
A. it is used for prophylaxis and therapy
B. it only provides humoral immunity
C. it can induce neutralizing antibodies
D. it provides immediate effect
E. none of the above
251 Adoptive-acquired immunity may be the result of:
A. immunization with a vaccine
B. transfer of bone marrow from one individual to another
C. exposure to an individual who has an infectious disease
D. a physician administering a gamma globulin shot to someone who has
had a needle stick (immunoglobulins)
E. A and D
252 Rituximab is a monoclonal anti-CD20 antibody used in the treatment
of CD20+ B-cell non-Hodkin’s lymphoma. How can this antibody
prevent tumor progression?
A. By tagging the tumor cells for destruction by activated cytotoxic T-cells
and/or NKT cells
B. By tagging suppressor T-cells for elimination by NK cells
C. By tagging the tumor cells for destruction mediated by the complement
system and/or by inducing antibody-dependent cell-mediated cytotoxicity
D. By targeting toxins to the malignant cells
E. All of the above

253 The immunosuppressive drug which probably attacks DNA by


alkylation and cross-linking is:
A. Azathioprine
B. Cyclophosphamide
C. Cyclosporine
D. Rapamycin
E. Prednisone

254 Which of the following is NOT a glucocorticoid?


A. Cortisone
B. Prednisolone
C. Dexamethasone
D. Ibuprofen
E. Hydrocortisone

255 Which of the following is NOT a non-steroidal anti-inflammatory agent


(NSAID)?
A. Aspirin
B. Naproxen
C. Dexamethasone
D. Ibuprofen
E. A and B
256 Which of the following is antihistaminic?
A. Aspirin
B. Naproxen
C. Dexamethasone
D. Ibuprofen
E. Loratadin
257 Which of the following medications can block mast cell vasoactive
mediator that participate in type I hypersensitivity?
A. Aspirin
B. Loratadine
C. Dexamethasone
D. Ibuprofen
E. Prednisolone
258 Which of the following medications can inhibit leukotriene
production?
A. Aspirin
B. Loratadin
C. Montelukast
D. Ibuprofen
E. Prednisolone
259 Which of the following medications does NOT inhibit cyclooxygenase
II?
A. Aspirin
B. monoclonal antibodies against CD52
C. Dexamethasone
D. Ibuprofen
E. Prednisolone
260 Which of the following medications is NOT immunosupressive agent?
A. Aspirin
B. monoclonal antibodies against CD52
C. Dexamethasone
D. Ibuprofen
E. Ribomunyl
261 Which of the following medications is NOT immunostimulatory agent?
A. Bronchovaxom
B. Dexamethasone
C. Luivac
D. A and B
E. Ribomunyl
262 What is the mechanism of Intravenous Immunoglobulin (IVIG) action
in replacement therapy?
A. inhibition of T-cell signaling
B. antiproliferative effect
C. inhibition of cyclooxygenase II
D. opsonization
E. B and C
263 Which of the following is an example of allergy-specific therapy?
A. the use of glucocorticoids
B. the use of antileukotriens
C. inhibition of cyclooxygenase II
D. replacement therapy with IVIG
E. desensitization
264 Which of the following is NOT used in cancer immunotherapy?
A. cytokines
B. monoclonal antibodies
C. checkpoint inhibitors
D. replacement therapy with IVIG
E. vaccines
265 What therapeutic agent is produced by isolating tumor cells from an
individual and processing those tumor cells into a vaccine formulation
in vitro?
A. allogenic cancer vaccine
B. autologous cancer vaccine
C. dendritic cell cancer vaccine
D. xenogenic cancer vaccine
E. none of the above
266 Which of the following cancer vaccines provides personalized
immunotherapy?
A. allogenic cancer vaccine
B. viral vector-based vaccine
C. autologous cancer vaccine
D. xenogenic cancer vaccine
E. none of the above
267 Which vaccination is marketed and recommended in the prevention of
a virus that is known to cause cervical cancer?
A. Herpes zoster vaccine (Zostavax)
B. Papillomavirus vaccine (Gardasil)
C. Pneumococcal vaccine (Prevnar 13)
D. Hepatitis B virus vaccine (Recombivax HB)
E. None of the above
268 Which of the following is incorrect?
A. monoclonal antibodies can be used to diagnose or treat diseases
B. monoclonal antibodies are polyspecific
C. monoclonal antibodies are used in cancer treatment
D. monoclonal antibodies are monospecific
E. monoclonal antibodies are used for the treatment inflammatory diseases

269 What type of antibody is made in the laboratory by combining a


human antibody with a small part of a mouse or rat monoclonal
antibody?
A. chimeric monoclonal antibody
B. humanized monoclonal antibody
C. xenogenic monoclonal antibody
D. conjugated monoclonal antibody
E. all of the above
270 Cancer vaccine that is composed of a plasmid DNA that contains the
genetic code for a tumor-specific or tumor-associated antigens is called
as
A. carrier vaccine
B. vector-based vaccine
C. peptide vaccine
D. DNA-vaccine
E. none of the above
271 What is the name of a monoclonal antibody that is produced from the
cells of a nonhuman organism, usually a mouse, in which a portion of
the antibody (two thirds) has been replaced with a human sequence of
amino acids?
A. conjugated monoclonal antibody
B. humanized monoclonal antibody
C. xenogenic monoclonal antibody
D. chimeric monoclonal antibody
E. antiidiotypic monoclonal antibody

272 Which of the following therapeutic monoclonal antibodies prevents


tumor growth by blocking receptors for growth factor?
A. Infliximab
B. Adalizumab
C. Herceptin
D. Rituximab
E. Alemtuzumab

273 Which of the following vaccine types is commonly given with an


adjuvant?
A. an attenuated vaccine
B. a modified live vaccine
C. a chemically killed vaccine
D. an immunoglobulin
E. an agglutinating antigen

274 What is the name of compounds designed to improve the


immunogenicity of vaccines by triggering early innate responses?
A. opsonins
B. stabilizers
C. preservatives
D. antibiotics
E. adjuvants

275 Dendritic cells used in cancer vaccine are generated in vitro from
A. peripheral blood neutrophils
B. peripheral blood lymphocytes
C. tumor-associated macrophages
D. peripheral blood monocytes
E. none of the above

276 Determining a patient’s blood group with monoclonal test-reagents


revealed positive agglutination reaction to anti-A and anti-B reagents,
and negative reaction to anti-D. What blood group does this patient
have?
A. III (В) Rh (-)
B. II (А) Rh (+)
C. IV (АВ) Rh (-)
D. IV (АВ) Rh (+)
E. I (0)Rh (+)

277 Blood group of a 30-year-old man has been determined before a


surgery. The blood was Rhesus-positive. Agglutination did not occur
with standard А (II) and В (III) serums. The blood belongs to the
following group:
A. А (II)
B. 0 (I)
C. В (III)
D. АВ (IV)
E. the result is false positive

278 Serological diagnostics of infectious diseases is based on specific


Ag:Ab interaction. Specify the serological reaction that underlies
adhesion of microorganisms when they are affected by specific
antibodies in presence of an electrolyte:
A. Neutralization reaction
B. Precipitation reaction
C. Complement-binding reaction
D. Hem adsorption reaction
E. Agglutination reaction

279 A pregnant woman was detected to have IgM to rubella virus. An


obstetrician gynaecologist recommended therapeutic abortion due to
the high risk of teratogenic affection of the fetus. Detection of IgM was
of great importance as it is this specific immunoglobulin that:
A. Are associated with anaphylactic reactions
B. Penetrate placental barrier
C. Have the largest molecular weight
D. Indicate recent infection
E. Are the main factor of antiviral protection

280 A patient has been diagnosed with upper respiratory tract infection
(URTI). Blood serum contains immunoglobulin M. What stage of
infection is it?
A. Prodromal
B. Acute
C. Incubation
D. Reconvalescence
E. Carriage

281 A 6-year-old child with suspected active tuberculosis process has


undergone diagnostic Mantoux test. What immune-biological
preparation was injected?
A. Tularin
B. BCG vaccine
C. DTP vaccine
D. Tuberculin
E. Td vaccine
282 The clumping of antigen-to-antibody particles during an immunoassay
such as the modern pregnancy test, is known as
A. Clustering
B. Agglutination
C. Chemotaxis
D. Diapedesis
E. Precipitation
283 Which statement about the preparation and properties of monoclonal
antibodies is FALSE?
A. These antibodies are raised when an antigen is injected into animals to
activate B lymphocyte production, which are then isolated from the
spleen
B. The selected B lymphocytes are hybridized (fused) with multiple
myeloma (cancer) cells so that they will survive and reproduce in large
numbers
C. Isolated lymphocyte-myeloma fusions (hybridomas) secrete large
amounts of the desired antibodies
D. Monoclonal antibodies are clinically very specific in their action,
serving as vaccines against very virulent viruses and bacteria
E. Monoclonal antibodies are homogenous by isotype, antigenic specificity
and affinity
284 In the production of monoclonal antibodies, B cells are fused with
.....cell to form a ....... cell. What are the two missing words?
A. T cells, hybridoma
B. tumour, hybridoma
C. tumour, monoclonal
D. T cells, monoclonal
E. None of the above
285 A mother has blood type A, Rh+, her newborn child is 0, Rh–. The
newborn’s blood is likely to contain substantial levels of antibodies to
A. anti-A
B. anti-B
C. anti-A and anti-B
D. anti-Rh
E. none of the above
286 In studying a case of Rh incompatibility, you would carry out an
Indirect Coombs’ Test on:
A. father’s serum
B. child’s RBCs
C. mother’s RBCs
D. mother’s serum
E. child’s serum

287 F(ab’)2 fragments prepared from an IgG antibody to sheep red blood
cells should still be able to carry out:
A. hemolysis
B. agglutination
C. both
D. neither
E. complement fixation

288 In immunology a "clone" is a large population of


A. antibody or immunoglobulin molecules belonging to the same subtype
B. cancer cells growing within a tumor
C. specific antigen molecules found on the membrane surface of
lymphocytes
D. genetically identical cells
E. cells of the same origin
289 The competitive immunoassay can be used
A. to detect very small amounts of antigen
B. to detect antibody associated with allergies (IgE)
C. both (A) and (B)
D. commonly to detect trace amounts of drugs
E. none of the above
290 Which of the following is an immunodiffusion test?
A. direct Coombs’ technique
B. Bezredka technique
C. Ouchterloney technique
D. ELISA
E. All of the above
291 ELISA techniques is widely used for quantitative estimation of
A. hormones
B. growth factors
C. tumor markers
D. different analytes which are present in very small concentration in
biological fluid
E. none of the above
292 Polyclonal antibodies are antibodies that are:
A. derived from multiple cell lines that are directed towards a specific
single epitope on an antigen
B. derived from single line of cells that are directed towards a specific
single epitope on an antigen
C. derived from single line of cells that are directed towards multiple
epitopes on an antigen
D. derived from multiple cell lines that are directed towards multiple
epitopes on an antigen
E. all of the above
293 Suppose your laboratory uses a competitive binding assay with a
labeled antigen that incorporates spectrophotometry for the
measurement of a particular antigen. As the amount of measured
antigen ________ in the patient sample, the absorbance of the test
solution will ________.
A. increases, increase
B. decreases, decrease
C. increases, decrease
D. decrease, increase
E. both C and D are correct

294 What is the sensitivity range typically seen with indicator-labeled


immunoassays?
A. mg/ml
B. mg/L
C. μ g/ml
D. mg/dl
E. ng/ml
295 In a noncompetitive immunoassay reaction format...
A. an unlabeled antigen is first mixed with excess antibody and binding
reaches equilibrium. Labeled antigen is then added
B. antigen from an unknown sample is allowed to react with a solid-phase
antibody, and a second labeled antibody is added that reacts with bound
antigen
C. all reactants are mixed together with both the labeled and unlabeled
antigens binding to the antibody
D. bound label is inversely proportional to unlabeled antigen concentration
E. A and D
296 Substances that have a single known concentration of the analyte being
studied and that are used to set up standard curves to determine
unknown analyte concentration are referred to as
A. absorbers
B. unknowns
C. positive control
D. negative control
E. calibrators
297 In competitive labeled immunoassays, the label is typically placed on
the ________ in the reagent and is used to measure ________ in the
patient sample.
A. antibody, antibody
B. antigen, antigen
C. antibody, antigen
D. antigen, antibody
E. A and B
298 Direct Coombs test detects autoantibodies already bound (in vivo) to
the surface of red blood cells, which is seen in
A. Autoimmune hemolytic anemia (IgG causes premature destruction of
RBCs)
B. HDN/erythroblastosis fetalis (newborn's RBCs have maternal anti-D)
C. Drug induced hemolytic anemia
D. Some patients with Chronic Lymphocytic Leukemia
E. All of the above
299 What is the indication of a positive result in a complement fixation test
(the presence of the antibodies of interest in the examined serum)?
A. Pink serum/non-lysed sheep RBCs
B. Pink serum/lysed sheep RBCs
C. Transparent serum/lysed sheep RBCs
D. Transparent serum/non-lysed sheep RBCs
E. sheep RBC agglutination
300 What is the indication of a negative result in a complement fixation test
(antibodies of interest are absent in the examined serum)?
A. Pink serum/non-lysed sheep RBCs
B. Pink serum/lysed sheep RBCs
C. Transparent serum/lysed sheep RBCs
D. Transparent serum/non-lysed sheep RBCs
E. sheep RBC agglutination
What component gives color to the stain?
Benzene ring
Chromophore
Auxochrome
Chromogen
None of the above

What component gives ionic property to the stain?


Benzene ring
Chromophore
Auxochrome
Chromogen
None of the above

What is a basic structural component of a dye?


Chromophore
Benzene ring
Auxochrome
Chromogen
None of the above

What components does chromogen contain?


Chromophore and auxochrome
Auxochrome and benzene ring
Benzene ring and chromophore
Benzene ring, chromophore and auxochrome
Chromophore

Choose cationic stain from the list


Nigrosin
Safranin
Giemsa stain
Eosin
India ink

Choose anionic stain from the list


Crystal violet
Methylene blue
Malachite green
Nigrosin
Giemsa stain

What type of microscope has the highest resolving power?


Scanning electron microscope
Transmission electron microscope
Dark-field microscope
Fluorescence microscope
Differential interference contrast microscope

What type of microscope produces polarized light?


Bright-field microscope
Dark-field microscope
Fluorescence microscope
Transmission electron microscope
Differential interference contrast microscope

What type of microscope makes it possible to study various layers of relatively thick specimen?
Dark-field microscope
Fluorescence microscope
Transmission electron microscope
Confocal scanning laser microscope
Differential interference contrast microscope

What type of microscope utilizes the difference in the refractive index between bacterial cells
and the surrounding medium?
Confocal scanning laser microscope
Dark-field microscope
Fluorescence microscope
Transmission electron microscope
Phase Contrast microscope

“Complete destruction of all the micro-organisms including spores from an object or


environment” refers to:
Pasteurization
Disinfection
Sterilization
Boiling
Preservation

Choose the most appropriate definition of the term “pasteurization”


The complete destruction of all the micro-organisms including spores from an object or
environment
The process of reducing or eliminating pathogenic microorganisms or viruses in or on material
so that it no longer presents a hazard
Prevention of multiplication of microorganisms in formulated products, including
pharmaceuticals and foods
The process of heating food or other substances under controlled conditions of time and
temperature to reduce the total number of pathogens without damaging the substance
Inhibition of the growth of microorganisms in or on living tissue (eg, skin) or biologic fluids (eg,
mucosal secretions)

Choose the most appropriate definition of the term “preservation”


The complete destruction of all the micro-organisms including spores from an object or
environment
The process of reducing or eliminating pathogenic microorganisms or viruses in or on material
so that it no longer presents a hazard
Prevention of multiplication of microorganisms in formulated products, including
pharmaceuticals and foods
The process of heating food or other substances under controlled conditions of time and
temperature to reduce the total number of pathogens without damaging the substance
Inhibition of the growth of microorganisms in or on living tissue (eg, skin) or biologic fluids (eg,
mucosal secretions)

Most cultural media and solutions are sterilized with:


Chemical agents
Ethyl alcohol
Glutaraldehyde
Radiation
Autoclave

What method is applied to sterilize liquids that are damaged by heat


Use of Vapor-Phase Sterilants
Use of Quaternary Ammonium Compounds
Sterile Filtration
Radiation
Use of Aldehydes

Sodium hypochlorite belongs to:


Aldehydes
Halogen-Releasing Agents
Biguanides
Bisphenols
Organic Acids

Chlorhexidine belongs to:


Aldehydes
Halogen-Releasing Agents
Biguanides
Bisphenols
Organic Acids

Sterilants that have hydrophobic group and hydrophilic group in their strucure:
Organic Acids
Aldehydes
Quaternary Ammonium Compounds
All mentioned above
None of abovementioned

Which one of the listed microorganisms is diplococcus?


Micrococcus luteus
Stomatococcus mucilaginosus
Neisseria gonorrhoeae
Nocardia farcinica
Actinomyces Israeli

Cells of what bacterium are linked in chains?


Micrococcus luteus
Stomatococcus mucilaginosus
Neisseria gonorrhoeae
Streptococcus pyogenes
Staphylococcus aureus

Choose spherical-shaped bacterium from the list


Spirillum
Lactobacillus
Sarcina ventriculi
Nocardia farcinica
Vibrio cholera

Choose pleomorphic bacterium from the list


Neisseria gonorrhoeae
Rickettsia rickettsii
Sarcina ventriculi
Staphylococcus aureus
Vibrio cholera

Choose comma-shaped bacterium


Stomatococcus mucilaginosus
Neisseria gonorrhoeae
Sarcina ventriculi
Vibrio cholerae
Nocardia farcinica

The size of up to 1 mm can have:


Spirochaetes
Mycoplasmas
Rods
Sulphur bacteria
Cocci

The smallest bacteria are found in the group of:


Spirochaetes
Mycoplasmas
Diplococci
Sulphur bacteria
Staphylococci

Pleomorphism is characteristic for


Neisseria gonorrhoeae
Sarcina ventriculi
Corynebacterium diphtheriae
Micrococcus luteus
Vibrio cholera

Which one of the listed microorganisms is pleomorphic?


Salmonella typhi
Escherichia coli
Bifidobacterium bifidum
Vibrio cholera
Staphylococcus aureus

A specimen stained by Ozheshko method contains rod-like microorganisms stained blue with
round terminal components stained red. What are these components?
Pili
Volutin granules
Flagella
Endopores
Capsules
Bacterial endospores are:
Constant structural components of bacterial cells.
Used for nutrient storage.
Easily inactivated by heat.
Easily inactivated by ultraviolet radiation
Visualized using the acid-fast stain

Bacterial endospores are:


Easily inactivated by heat.
Constant structural components of bacterial cells.
Resistant to heat
Used for nutrient storage.
Easily inactivated by ultraviolet radiation

A specimen stained by Ziehl-Neelsen method contains rod-like microorganisms stained blue


with round terminal components stained red. What are these components?
Constant cell structures aimed to store energy
Non constant cell structures aimed to store energy
Non constant cell structures allowing for active movement
Non constant cell structures aimed for defense against unfavorable environmental conditions
Constant cell structures aimed for defense against unfavorable environmental conditions

A specimen stained by Shelfer-Foulton’s method contains rod-like microorganisms stained red


with round terminal components stained green. These structures are:
Constant structural components of bacterial cells
Resistant to ultraviolet radiation
Used for nutrient storage.
Easily inactivated by heat.
Easily inactivated by ultraviolet radiation

Which staining technique will be indicative for causing agents of botulism, gas gangrene and
tetanus?
Capsule staining
Volutin staining
Endospore staining
Neisser Staining
Loeffler flagella staining

A microorganism stained by Neisser’s method contains inclusions stained in brown. These


granules are:
Osmotic neutral reservoirs of carbon and energy
Intracytoplasmic storage form of complexed inorganic polyphosphate
Reserve of electrons (reducing source)
Used to modulate the cell's buoyancy
Reserve carbon and energy source

A patient is suspected to have diphtheria. What staining technique can be applied as a rapid test
for the identification of the causative agent?
Modified Ziehl-Neelsen staining
Burri method
Neisser Staining
Schaeffer-Fulton`s staining
Capsule staining

A microorganism stained by Neisser’s method contains inclusions stained in brown. These


granules are:
Reserve of electrons
Polyphosphate reserve
Used to modulate the cell's buoyancy
Carbon and energy reserve
Osmotic neutral reservoirs of carbon and energy

After staining a microorganism isolated from feces of a patient was shown to possess multiple
flagella located over the entire cell. This bacteria is:
Monotrichous
Amphitrichous
Peritrichous
Lophotrichous
Atrichous

What staining method can be used to detect the cell structure that serves as an H antigen?
Modified Ziehl-Neelsen staining
Loeffler flagella staiing
Neisser Staining
Schaeffer-Fulton`s staining
Burri method

Application of various staining techniques has shown a microorganism isolated from feces of a
patient to be non-motile. This bacterium is:
Monotrichous
Amphitrichous
Peritrichous
Lophotrichous
Atrichous

After staining a microorganism isolated from feces of a patient was shown to possess single
flagellum on each of two opposite ends. This bacterium is:
Monotrichous
Amphitrichous
Peritrichous
Lophotrichous
Atrichous

A patient is suspected to have a cholera disease. The causative agent of the disease:
Is not motile
Is atrichous
Is peritrichous
Is monotrichous
Is lophotrichous

What staining technique is applied to reveal flagella of spirochaetes?


Modified Ziehl-Neelsen staining
Loeffler flagella staining
Neisser Staining
Schaeffer-Fulton`s staining
None of above

A patient is suspected to have syphilis. The causative agent of the disease:


Is not motile
Is monotrichous
Is peritrichous
Has one polar flagellum
Is motile

Representatives of which group of microorganisms are mostly atrichous?


Spore-forming bacilli
Spirilla
Cocci
Spirochaetes
Non-spore-forming bacilli

Which structural component is constant in bacterial cell?


Mitochondria
Nucleus
Nucleoid
Endospore
Capsule

If an infectious agent is a bacteria, then next is true:


It has membrane-bound organelles
It has Golgi apparatus
It has 70S ribosome and DNA associated with histone proteins
It has 80S ribosome and DNA associated with histone proteins
It has a cytoplasmic membrane

Non-constant structural components of bacterial cells are:


Volutin granules, Cell wall, Pili
Pili, Volutin granules, Flagella
Flagella, Endopores, Nucleoid
Endopores, Capsules, Cytoplasm
Cytoplasmic membrane, Capsules, PHB granules

Which structural component is constant in a bacterial cell?


80S Ribosome
Endospore
Sulfur globules
70S Ribosome
Capsule

Oxidative and transport enzymes of a cell are located in:


Cytoplasm
Capsule
Cytoplasmic membrane
Endospore
Cell wall
A structure of a cell that stays clear in Burri-Hins staining method is:
A motility organella
70S Ribosome
A virulence factor
An inclusion
Cell envelope

The protein synthesizing systems of a syphilis causing agent are?


80S Ribosomes
Nucleoid
Sulfur globules
70S Ribosomes
None of the above

Choose bacteria that naturally lack the cell wall


Mycobacterium tuberculosis
Corynebacterium diphtheriae
Mycoplasma pneumoniae
Bacillus megaterium
Pseudomonas fluorescens

Mycolic acid is present in


Outer Membrane of Gram-positive bacteria
Outer Membrane of Gram-negative bacteria
Cytoplasmic Membrane of Gram- positive bacteria
Cytoplasmic Membrane of Gram-negative bacteria
In cell wall of Mycobacterium

Lipopolysaccharide is present in
Outer Membrane of Gram-positive bacteria
Outer Membrane of Gram-negative bacteria
Cytoplasmic Membrane of Gram- positive bacteria
Cytoplasmic Membrane of Gram-negative bacteria
In cell wall of Mycobacterium

Choose the INCORRECT statement about the causative agent of leprosy:


Contains mycolic acid in a cell wall
Contains phospholipids in a cell wall
Contains peptidoglican in a cell wal
Contains LPS in a cell wall
Contains 70S ribosomes

O-antigen is located in
Plasma membrane
Outer membrane
Appendages
Cell coverings
S-layer

Teichoic acid is:


A polymer of glycan (sugar) chains cross-linked by short peptide fragments
A polymer of ribitol or glycerol (alcohols) and phospholipid
Comprised of Lipid A and O polysaccharide
Comprised of phospholipids, lipopolysaccharides, lipoproteins and surface proteins
Comprised of alternating glycans (NAM and NAG) bound together in long strands

What component is absent in gram-positive cell walls?


Teichoic acid
Peptidoglycan
N-acetyl muramic acid
Lipopolysaccharide
Lipoteichoic acid

The results of your patient’s throat culture just arrived, and it have shown that the causing agent
is a pathogenic yeast. What is true for this agent?
It doesn’t have protein synthesizing systems.
It doesn’t have membrane-bound organelles
It doesn’t have nucleus
It doesn’t have a cytoplasmic membrane
It doesn’t contain murein-containing cell wall

Choose a genus of microorganism that lacks a nucleus:


Mucor
Penicillium
Nocardia
Saccharomyces
Candida

Choose a genus of microorganism that lacks a nucleus:


Mucor
Streptomyces
Penicillium
Balantidium
Saccharomyces

Choose a genus of microorganism that lacks a nucleus:


Mucor
Penicillium
Saccharomyces
Actinomyces
Candida

Cryptococcus is a representative of:


Cocci
Streptococci
Tetracocci
Micrococci
Fungi

A cell “mouth” for ingesting fluids or solid particles is called


Cytosole
Cytosome
Cytoplasm
Endoplasm
Endosome

What microorganism can cause malaria?


Naegleria
Plasmodium falciparum
Trypanosoma cruzi
Toxoplasma gondi
Acanthamoeba

What fungus can cause dermatophytosis?


Coccidioides
Rhizopus
Paracoccidioides
Malassezia
Microsporum

What fungus causes Chicago disease?


Coccidioides immitis
Blastomyces dermatitidis
Paracoccidioides brasiliensis
Malassezia furfur
Cryptococcus neoformans

Cryptococcosis is an infection caused by a representative of:


Cocci
Streptococci
Fungi
Actinobacteria
Diplococci

A causative agent of candidiasis is a representative of:


Rod-shaped bacteria
Winding bacteria
Actinobacteria
Yeasts
Protozoa

A causative agent of leishmaniasis is a representative of:


Rod-shaped bacteria
Winding bacteria
Actinobacteria
Yeasts
Protozoa

Choose the term that describes infections, caused by environmental pathogens in the general
population
Hospital-associated infections
Community-acquired infections
Opportunistic infections
Mixed infection
Fungal infections
Choose the term that describes infections, caused by pathogens in clinical settings
Community-acquired infections
Hospital-associated infections
Opportunistic infections
Mixed infection
Fungal infections

Choose the term that describes infections, caused by pathogens infecting already weakened
individuals
Community-acquired infections
Hospital-associated infections
Opportunistic infections
Mixed infection
Fungal infections

What disease can be diagnosed using acid-fast staining?


Tetanus
Tuberculosis
Diphtheria
Plague
Cholera

Choose causative agent of plague


Clostridium perfringens
Pseudomonas fluorescens
Corynebacterium diphtheriae
Mycobacterium leprae
Yersinia pestis

What bacteria have mycolic acid in its cell wall?


Escherichia coli
Pseudomonas fluorescens
Mycoplasma pneumoniae
Bacillus subtilis
Mycobacterium tuberculosis

Mycoplasmas:
Are Gram-positive bacteria that have lost their cell wall
Are Gram-negative bacteria that have lost their cell wall
Are L-forms
Naturally lack a cell wall
None of the above

A complex staining technique that can be used in preliminary diagnosis of tuberculosis:


Gram-staining method
Romanowsky-Giemsa method
Ziehl-Nielsen method
Neisser method
Wet mount

Lipopolysaccharide is not present in a cell wall of


Shigella
Escherichia coli
Corynebacterium diphtheriae
Yersinia pestis
Pseudomonas fluorescens

Clostridium tetani treated with lysozyme in media with the osmotic strength balanced to the
internal osmotic pressure of the cell will form:
Any kind of L-form
Protoplasts
Spheroplasts
Chloroplasts
Acid-fast cell wall

In osmotically protected media, E. coli cells treated with EDTA-lysozyme will form:
Protoplasts
Spheroplasts
Chloroplasts
Acid-fast cell wall
None of the above

What is the target of lysozyme?


Lipid A
Lipopolysaccharide
Teichoic acid
Peptidoglycan
Lipoteichoic acid

Choose gram-positive bacteria from the list


Yersinia pestis
Bacillus subtilis
Pseudomonas fluorescens
Vibrio cholerae
Escherichia coli

Choose the form of bacteria which can cause persistent infections that are often resistant to
antibiotic treatment?
Protoplasts
Archaea
Gram-positive bacteria
Spheroplasts
L-phase variants

Choose gram-negative bacteria from the list


Bacillus megaterium
Clostridium botulinum
Corynebacterium diphtheriae
Yersinia pestis
Staphylococcus aureus

Choose acid-fast bacteria from the list


Clostridium botulinum
Escherichia coli
Vibrio cholerae
Mycobacterium leprae
Bacillus megaterium

Teichoic acid is present in a cell wall of:


Meningococci
Vibrio cholera
Clostridium tetani
Gonococci
Escherichia coli

A patient is suspected for tuberculosis. Which complex staining technique can be used for a
preliminary diagnosis?
Gram-staining method
Ziehl-Neelsen
Romanowsky-Giemsa method
Neisser method
Wet mount

Bacterial strain uses energy of chemical reactions and obtains electrons from sugars. Due to
classification it belongs to:
PhotoAutotrophs
PhotoOrganotrophs
ChemoOrganotrophs
ChemoLitotrophs
OrganoHeterotrophs

Bacterial strain uses solar energy and obtains electrons from sugars. Due to classification it
belongs to:
PhotoAutotrophs
PhotoOrganotrophs
LitoHeterotrophs
PhotoLitotrophs
OrganoHeterotrophs

Bacterial strain uses solar energy and obtains electrons from inorganic compounds. Due to
classification it belongs to:
PhotoAutotrophs
LitoAutotrophs
LitoHeterotrophs
PhotoLitotrophs
OrganoHeterotrophs

Bacterial strain utilizes CO2 as a source of carbon and obtains electrons from inorganic
compounds. Due to classification it belongs to:
ChemoAutotrophs
LitoAutotrophs
LitoHeterotrophs
ChemoOrganotrophs
OrganoHeterotrophs
Bacterial strain utilizes organic source of carbon and obtains electrons from inorganic
compounds. Due to classification it belongs to:
ChemoAutotrophs
ChemoOrganotrophs
LitoHeterotrophs
ChemoOrganotrophs
OrganoHeterotrophs

The causative agent of the disease utilizes organic source of carbon and obtains electrons from
sugars. Due to classification it belongs to:
ChemoAutotrophs
ChemoOrganotrophs
PhotoAutotrophs
ChemoOrganotrophs
OrganoHeterotrophs

The causative agent of the disease utilizes sugars and alcohols as a source of carbon and uses
energy of chemical reactions. Due to classification it belongs to:
ChemoAutotrophs
PhotoHeterotrophs
PhotoAutotrophs
ChemoOrganotrophs
ChemoHeterotrophs

The causative agent of the disease utilizes organic source of carbon and uses energy of chemical
reactions. Due to classification it belongs to:
ChemoAutotrophs
PhotoHeterotrophs
PhotoAutotrophs
ChemoOrganotrophs
ChemoHeterotrophs

The bacteria isolated from water sample utilizes sugars as source of carbon and uses solar
energy. Due to classification it belongs to:
ChemoLitotrophs
PhotoHeterotrophs
PhotoAutotrophs
Autotrophs
Phototrophs

The bacteria isolated from water sample utilizes CO2 as source of carbon and uses solar energy.
Due to classification it belongs to:
ChemoLitotrophs
PhotoHeterotrophs
PhotoAutotrophs
Autotrophs
Phototrophs

Neutrophiles grow best at a pH range of ___


1-4
5-8.0
2-3.5
9-9.5
3-7

Acidophiles grow best at a pH _______


Less than 3
2-3.5
Less than 5
5-8.0
3-7

Alkaliphiles grow best at a pH ____


Less than 3
2-3.5
Less than 5
5-8.0
More than 8.5

Mesophiles are group of bacteria that grow within the temperature range of?
less than 0 degree Celsius
degree Celsius 0-20
degree Celsius 25-45
degree Celsius 45-60
more than degree Celsius 60

Psychrophiles are group of bacteria that grow within the temperature range of?
less than –5 degree Celsius
degree Celsius –5…15
degree Celsius 25…45
degree Celsius 45…60
more than 60 degree Celsius

Thermophiles are group of bacteria that grow within the temperature range of?
less than –5 degree Celsius
degree Celsius –5…15
degree Celsius 25…45
degree Celsius 45…70
more than 60 degree Celsius

The temperature that allows for most rapid growth during a short period of time is known as
Minimum Temperature
Maximum Temperature
Optimum Temperature
Growth Temperature
No correct answer

Microorganisms that can be found thriving in icy-cold climates would be


considered_____________
Barophiles
Termophiles
Mesophiles
Psychrophiles
Bibliophile
Bacteria with optimum growth temperature between 70C and 110C are:
Alkaliphiles
Thermophiles
Mesophiles
Psychrophiles
Hyperthermophiles

Bacteria with optimum growth temperature between 45C and 70C, which are commonly found
in hot springs and in compost heap are:
Alkaliphiles
Thermophiles
Mesophiles
Psychrophiles
Hyperthermophiles

Chose the correct definition for general purpose media:


Designed to suppress the growth of some microorganisms while allowing the growth of others
Is used to increase the relative concentration of certain microorganisms in the culture
Designed in such a way that different bacteria can be recognized on the basis of their colony
color
Simple media that supports most non-fastidious bacteria
Is used to grow nutritionally exacting (fastidious) bacteria

Chose the correct definition for enrichment culture media:


Simple media that supports most non-fastidious bacteria
Designed to suppress the growth of some microorganisms while allowing the growth of others
Is used to increase the relative concentration of certain microorganisms in the culture
Designed in such a way that different bacteria can be recognized on the basis of their colony
color
Is used to grow nutritionally exacting (fastidious) bacteria

Chose the correct definition for enriched media:


Simple media that supports most non-fastidious bacteria
Designed to suppress the growth of some microorganisms while allowing the growth of others
Is used to increase the relative concentration of certain microorganisms in the culture
Designed in such a way that different bacteria can be recognized on the basis of their colony
color
Is used to grow nutritionally exacting (fastidious) bacteria

Chose the correct definition for differential media:


Simple media that supports most non-fastidious bacteria
Designed to suppress the growth of some microorganisms while allowing the growth of others
Is used to increase the relative concentration of certain microorganisms in the culture
Designed in such a way that different bacteria can be recognized on the basis of their colony
color
Is used to grow nutritionally exacting (fastidious) bacteria

Chose the correct definition for selective media:


Simple media that supports most non-fastidious bacteria
Designed to suppress the growth of some microorganisms while allowing the growth of others
Is used to increase the relative concentration of certain microorganisms in the culture
Designed in such a way that different bacteria can be recognized on the basis of their colony
color
Is used to grow nutritionally exacting (fastidious) bacteria

Simple media that support most non-fastidious bacteria:


Selective media
General Purpose media
Differential media
Enriched Media
Enrichment media

Media designed to suppress the growth of some microorganisms while allowing the growth of
others
General Purpose media
Differential media
Enriched Media
Enrichment media
Selective media

Media used to increase the relative concentration of certain microorganisms in the culture
General Purpose media
Differential media
Enriched Media
Enrichment media
Selective media

Media designed in such a way that different bacteria can be recognized on the basis of their
colony color
General Purpose media
Differential media
Enriched Media
Enrichment media
Selective media

Media used to grow nutritionally exacting (fastidious) bacteria:


General Purpose media
Differential media
Enriched Media
Enrichment media
Selective media

Choose a selective media that allows for recovering P. aeruginosa pure culture
Thayer Martin Agar
Mannitol Salt Agar
MacConkey’s Agar
Cetrimide Agar
Endo Agar

Choose a selective media that allows for isolating the Neisseria gonorrhoeae pure culture
Mannitol Salt Agar
MacConkey’s Agar
Cetrimide Agar
Endo Agar
Thayer Martin Agar

Choose a selective media that allows for isolating the S. aureus pure culture
Thayer Martin Agar
Mannitol Salt Agar
MacConkey’s Agar
Cetrimide Agar
Endo Agar

Which agar is selective for Vibrio cholerae?


MacConkey’s Agar
XLD
TCBS
MacConkey
Blood Agar

The isolation of gonorrhea-causing microorganism, Neisseria gonorrhoeae is achieved by the


addition of certain antibiotics into media. This media is an example of:
Differential media
Enriched media
Assay media
Selective media
Non cultivated

Thioglycollate media is used to cultivate


Aerobic bacteria
Anaerobic bacteria
Aerotolerant bacteria
Microaerophilic bacteria
NO correct answer

Thayer Martin Agar is a selective medium for


Staphylococcus aureus
Neisseria gonorrhoeae
Clostridium difficile
Vibrio cholerae
Pseudomonas aeruginosa

Mannitol Salt Agar is a selective medium for :


Neisseria gonorrhoeae
Staphylococcus aureus
Clostridium difficile
Enterobacteria
Pseudomonas aeruginosa

According to type of respiration bacteria are divided into aerobes and anaerobes. Which
group of bacteria can both use oxygen and not use it?
Obligate anaerobes
Obligate aerobes
Facultative anaerobes
Aerotolerant Anaaerobes
None of the above

An organism is completely dependent on atmospheric O2 for growth. This organism is


Osmotolerant
Acidophile
Facultative anaerobe
Aerotolerant anaerobe
Obligate aerobe

Which term best describes an organism that can't exist in the presence of oxygen?
Obligate aerobe
Facultative aerobe
Obligate anaerobe
Facultative anaerobe
Microaerophlile

Which term best describes an organism that cannot use oxygen to transform energy but can
grow in its presence?
Obligate aerobe
Aerotolerant anaerobes
Obligate anaerobe
Facultative anaerobe
Microaerophlile

The term facultative anaerobe refers to an organism that _______


Doesn't use oxygen but tolerates it
Is killed by oxygen
Uses oxygen when present, or grows without oxygen when oxygen is absent
Requires less oxygen than is present in air
Prefers to grow without oxygen

The term obligate aerobe refers to an organism that _______


Doesn't use oxygen but tolerates it
Is killed by oxygen
Uses oxygen when present, or grows without oxygen when oxygen is absent
Requires less oxygen than is present in air
Grows only in the presence of oxygen.

The term microaerophile refers to an organism that _______


Doesn't use oxygen but tolerates it
Is killed by oxygen
Uses oxygen when present, or grows without oxygen when oxygen is absent
Rrequires a low concentration of oxygen for growth, but higher concentrations are inhibitory.
Prefers to grow without oxygen

Unknown bacterial strain can utilize inorganic source of carbon. Due to classification it
belongs to:
Auxotroph
Prototroph
Chemotroph
Autotroph
Heterotroph
Wich organisms obtains carbon from both organic compounds and by fixing carbon dioxide
Chemotrophs
Mixotrophs
Lithotrophs
Phototrophs
Heterotrophs

Wich organisms obtains carbon for synthesizing cell mass from organic compounds
Chemotrophs
Organotrophs
Lithotrophs
Phototrophs
Heterotrophs

The causative agent of the disease utilizes sugars, alcohols, and aminoacids as source of
carbon. Due to classification it belongs to:
Litotrophs
Heterotrophs
Prototrophs
Autotrophs
Chemotrophs

Bacterial strain isolated from soil can utilize inorganic source of carbon. Due to classification
it belongs to:
Auxotroph
Prototroph
Autotroph
Chemotroph
Heterotroph

The organisms which can use reduced inorganic compounds as electron donors are known as
_________
Chemotrophs
Organotrophs
Lithotrophs
Phototrophs
Autotrophs

The organisms which can use reduced organic compounds as electron donors are known as
_________
Chemotrophs
Organotrophs
Lithotrophs
Phototrophs
Autotrophs

The organisms which can obtain reduced equivalents obtained from organic compounds
Chemotrophs
Organotrophs
Lithotrophs
Phototrophs
Autotrophs

The organisms which use energy of chemical reactions are known as _________
Organotrophs
Chemotrophs
Lithotrophs
Phototrophs
Autotrophs

The organisms which use solar energy are known as _________


Organotrophs
Chemotrophs
Lithotrophs
Phototrophs
Autotrophs

The GasPak system is suitable for cultivating:


Aerobic bacteria
Anaerobic bacteria
Aerotolerant bacteria
Microaerophilic bacteria
NO correct answer

Choose the incorrect statement describing the principle of anaerobic bacteria cultivation:
Bottles or tubes filled completely to the top
Tightly fitting stopper
Use of shaker or bubbling sterilizer
Reducing agent added
Oxygen removed

GasPak system generates:


Mix of O2 and CO2
Mix of H2 and CO2
Mix of H2 and CO
Mix of N2 and CO2
None of the above

Oxygen in GasPak system is removed by:


is gradually used up through reaction with catalyst
is gradually used up through reaction with H2
is gradually used up through reaction with CO2
is gradually used up through reaction with N2
None of the above

The generation time for E. coli is ________


2 minutes
35 minutes
20 minutes
13 minutes
1 hour

Average for bacteria is the generation time of:


20 minutes
30-60 minutes
2 minutes
13 minutes
1 hour

The generation time for Mycobacterium tuberculosis is ________


2 hours
12 to 24 minutes
12 to 24 hours
10 to 30 days
30-60 minutes

The generation time for Mycobacterium leprae is ________


2 hours
12 to 24 minutes
12 to 24 hours
10 to 30 days
30-60 minutes

The generation time of 20 min is characteristic for:


Mycobacterium tuberculosis
Mycobacterium leprae
E.coli
It is average generation time for bacteria
None of the above

The generation time of 10 to 30 days is characteristic for:


Mycobacterium tuberculosis
Mycobacterium leprae
E.coli
It is average generation time for bacteria
None of the above

The generation time of 12 to 24 hours is characteristic for:


Mycobacterium leprae
Mycobacterium tuberculosis
E.coli
It is average generation time for bacteria
None of the above

The time required for a cell to undergo binary fission is called the:
exponential growth rate
growth curve
generation time
lag period
stationary phase

Which phase in a population growth curve is associated with no increase or decrease in


microbial population?
lag phase
logarithmic phase
stationary phase
stop phase
death phase

The phase of the growth curve where a rapid growth of bacteria is observed is known as
lag phase
logarithmic phase
stationary phase
stop phase
death phase

A short period of time prior to exponential growth of a bacterial population during which no,
or very limited, cell division occurs.
logarithmic phase
stationary phase
stop phase
lag phase
death phase

Lag phase is also known as:


transitional period
generation time
period of rapid growth
period of initial adjustment
death period

A cryptic growth is characteristic for:


Lag phase
Log phase
Stationary phase
Exponential phase
Anaphase

Which phase shows reproduction rate equal to the equivalent death rate?
Log phase
Stationary phase
Death phase
Lag phase
None of above

The portion of the growth curve where a rapid growth of bacteria is observed is known as
Lag phase
Logarithmic phase
Stationary phase
Decline phase
None of above

Which of the following method can be used to determine the number of bacteria
quantitatively?
Streak-plate
Spread-plate
Pour plate
Pour-plate and spread plate
e)All answers are incorrect

A __________ is used when the diluted liquid culture is mixed in cooled molten agar before
being poured into culture dishes.
Streak-plate
Spread-plate
Pour plate
Pour-plate and spread plate
All answers are incorrect

The growth is normally expressed as __________ in turbidimetric measurement


cells per ml
cfu/ml
mg N2 /ml
U/ml
optical density

“Tenericutes” is a __________.
Subfamily name
Species name
Family name
Genus name
Division name

Members of what taxonomic category nowadays belong to the domain “Archaea”?


Firmicutes
B Tenericutes
Mendosicutes
Gracilicutes
Actinobacteria

What taxonomic category represents the “basic unit” of taxonomy


Order
Domain
Species
Kingdom
Genus

“Gracilicutes” is a __________.
Subfamily name
Species name
Family name
Genus name
Division name

“Staphylococcus” is a __________.
Subfamily name
Species name
Family name
Genus name
Superfamily name
E.coli belongs to
Irregular, nonsporing gram-opositive rods
Regular, non-sporing grampositive rods
Facultatively anaerobis gram-negative rods
Anaerobic gram-negative cocci
Gram-positive cocci

Bacteria that lack a cell wall are classified as members of


Firmicutes
B Actinobacteria
C. Mendosicutes
Gracilicutes
Tenericutes

Bacteria that have a gram-negative a cell wall are classified as members of


Firmicutes
B Actinobacteria
C. Mendosicutes
Gracilicutes
Tenericutes

Bacteria that have a gram-positive a cell wall are classified as members of


Actinobacteria
Mendosicutes
Gracilicutes
Firmicutes
Tenericutes

Members of what taxonomic category share the highest number of characteristics?


Class
Genus
Domain
Order
Kingdom

Which statement describes the catalase enzyme?


acts with a plasma factor to convert fibrinogen to a fibrin clot
enzyme that catalyzes the conversion of hydrogen peroxide to water and oxygen
catalyzes decarboxylation or deamination of the amino acids lysine, ornithine, and arginine
catalyzes nitrate reduction by several mechanisms
is detected by growing the organism in the presence of substrates such as gelatin or
coagulated egg

Which statement describes the coagulase enzyme?


enzyme that catalyzes the conversion of hydrogen peroxide to water and oxygen
catalyzes decarboxylation or deamination of the amino acids lysine, ornithine, and arginine
catalyzes nitrate reduction by several mechanisms
is detected by growing the organism in the presence of substrates such as gelatin or
coagulated egg.
acts with a plasma factor to convert fibrinogen to a fibrin clot
Which statement describes the proteinase enzyme?
acts with a plasma factor to convert fibrinogen to a fibrin clot
enzyme that catalyzes the conversion of hydrogen peroxide to water and oxygen
catalyzes decarboxylation or deamination of the amino acids lysine, ornithine, and arginine
catalyzes nitrate reduction by several mechanisms
is detected by growing the organism in the presence of substrates such as gelatin or
coagulated egg.

Which test is related to lactose fermentation?


urease production
proteinase production
Voges-Proskauer test
Citrate utilization
O-Nitrophenyl-β-d galactoside breakdown

Which test is related to acetoin detection?


urease production
proteinase production
Voges-Proskauer test
Citrate utilization
O-Nitrophenyl-β-d galactoside breakdown

Formation of a red dye after addition of a benzaldehyde reagent describes the outcome of:
urease test
indole test
Voges-Proskauer test
Citrate utilization test
O-Nitrophenyl-β-d galactoside breakdown

Liberation of oxygen as gas bubbles can be seen in


indole test
Voges-Proskauer test
oxidase test
O-Nitrophenyl-β-d galactoside breakdown
catalase test

The black color of the salts formed with heavy metals such as iron are characteristic for?
indole test
hydrogen sulfide test
Voges-Proskauer test
oxidase test
O-Nitrophenyl-β-d galactoside breakdown

What test is used to differentiate Staphylococcus aureus from other, less pathogenic
staphylococci?
Voges–Proskauer test
Citrate utilization test
Catalase production test
Nitrate reduction test
Coagulase test

Choose the appropriate description of the term “identification”


classification of organisms in an ordered system that indicates a natural relationship
categorization of organisms with similar traits into taxonomic groups (taxons)
naming of an organism by international rules (established by a recognized group of medical
professionals) according to its characteristics
is practical use of a classification scheme to isolate and distinguish desirable organisms from
undesirable ones, verify the authenticity or special properties of a culture in a clinical setting,
and isolate and recognize the causative agent of a disease
none of the above

Choose the appropriate description of the term “nomenclature”


classification of organisms in an ordered system that indicates a natural relationship
categorization of organisms with similar traits into taxonomic groups (taxons)
practical use of a classification scheme to isolate and distinguish desirable organisms from
undesirable ones
verification of the authenticity or special properties of a culture in a clinical setting
naming of an organism by international rules (established by a recognized group of medical
professionals) according to its characteristics

Choose the description of the term “taxonomy”


categorization of organisms with similar traits into taxonomic groups (taxons)
classification of organisms in an ordered system that indicates a natural relationship
practical use of a classification scheme to isolate and distinguish desirable organisms from
undesirable ones
verification of the authenticity or special properties of a culture in a clinical setting
naming of an organism by international rules (established by a recognized group of medical
professionals) according to its characteristics

Choose the description of the term “classification”


classification of organisms in an ordered system that indicates a natural relationship
categorization of organisms with similar traits into taxonomic groups (taxons)
practical use of a classification scheme to isolate and distinguish desirable organisms from
undesirable ones
verification of the authenticity or special properties of a culture in a clinical setting
naming of an organism by international rules (established by a recognized group of medical
professionals) according to its characteristics

Which of the following describes the term “biovar” most appropriately:


group of organisms that reveal a determined degree of homogeneity
population of cells of a single species all descendent from a single cell
the population of microorganisms within the species which differ in one or more features
a taxonomic unit that unites organisms on the basis of morphological, physiological,
biochemical, genetic and other
characteristics
the population of microorganisms of the same species in the same place at the same time

The population of microorganisms within the species which differ by tissue tropism is called
morphotype
chemovar
pathovar
genus
serovar
The population of microorganisms within the species which differ by antigenic structure is
called
morphotype
chemovar
pathovar
genus
serovar

Choose the description of the term “pathovar”


the population of microorganisms within the species which differ by antigenic structure
the population of microorganisms within the species which differ according to morphological
characteristics
the population of microorganisms within the species which differ by tissue tropism
the population of microorganisms within the species which differ according to physiological
and biochemical parameters
the population of microorganisms of the same species in the same place at the same time

Choose the description of the term “chemovar”


the population of microorganisms within the species which differ by antigenic structure
the population of microorganisms within the species which differ according to morphological
characteristics
the population of microorganisms within the species which differ by tissue tropism
the population of microorganisms within the species which differ according to physiological
and biochemical parameters
the population of microorganisms of the same species in the same place at the same time

Choose the most appropriate definition of the term “species”


A culture containing a single kind of microorganism
A population of cells all descendent from a single cell
A population of bacterial cells which have grown from one bacterial cell on solid medium
Collection of strains that all share the same major properties and differ in one or more significant
properties from other collections of strains
A group of organisms of the same genus in the same place at the same time

Choose the most appropriate definition of the term “colony”


A culture containing a single kind of microorganism
A population of cells all descendent from a single cell
A population of bacterial cells which have grown from one bacterial cell on solid medium
Collection of strains that all share the same major properties and differ in one or more significant
properties from other collections of strains
A group of organisms of the same genus in the same place at the same time

Choose the most appropriate definition of the term “pure culture”


Group of organisms that reveal a determined degree of homogeneity
Population of cells of a single species all descendent from a single cell
A culture containing a single kind of microorganism
Collection of strains that all share the same major properties and differ in one or more significant
properties from other collections of strains
A group of organisms of the same species in the same place
at the same time

Choose the most appropriate definition of the term “population of microorganisms”


Group of organisms that reveal a determined degree of homogeneity
Population of cells of a single species all descendent from a single cell
A culture containing a single kind of microorganism
Collection of strains that all share the same major properties and differ in one or more significant
properties from other collections of strains
A group of organisms of the same species in the same place
at the same time

Choose the description of the term “morphotype”


A taxonomic unit that unites organisms on the basis of morphological, physiological,
biochemical, genetic and other characteristics
The population of microorganisms within the species which differ according to morphological
characteristics
Population of cells of a single species all descendent from a single cell
The population of microorganisms within the species which differ according to physiological
and biochemical parameters
Group of organisms that reveal a determined degree of homogeneity

Choose the description of the term “serovar”


The population of microorganisms within the species which differ according to morphological
characteristics
The population of microorganisms within the species which differ by tissue tropism
The population of microorganisms within the species which differ by antigenic structure
The population of microorganisms within the species which differ according to physiological
parameters
The population of microorganisms within the species which differ according to biochemical
parameters

The population of microorganisms within the species which differ according to physiological
and biochemical parameters
Morphotype
Chemovar
Pathovar
Genus
Serovar

The population of microorganisms within the species which differ according to morphological
characteristics:
Chemovar
Morphotype
Pathovar
Serovar
All of these

Collection of strains that all share the same major properties and differ in one or more significant
properties from other collections of strains:
Colony
Population
Morphotype
Species
Pathovar
A group of organisms of the same species in the same place at the same time:
Colony
Population
Morphotype
Species
Pathovar

“A genomically coherent group of individual isolates or strains sharing a high degree of


similarity in many independent features when comparably tested under highly standardized
conditions” refers to:
Colony
Population
Strain
Species
Family

Choose the most appropriate definition of the term “taxon”


Group of organisms all descendent from a single cell
Group of organisms which have grown from one bacterial cell on solid medium
Group of organisms that reveal a determined degree of homogeneity
Collection of strains that all share the same major properties and differ in one or more significant
properties from other collections of strains
None of the above

Choose the INCORRECT rule in nomenclature


Genus name species name
Genus name is never abbreviated
Genus name is capitalized
A species name is never used alone
Species name is never abbreviated

Choose the INCORRECT rule in nomenclature


Genus name is capitalized and may be abbreviated
Genus name may be used alone to indicate a genus group
Species name is capitalized
A species name is never used alone
Species name is never abbreviated

What approximate amount of pathogens can be identified using routine culturing techniques or
targeting the microbes with molecular probes?
1%
5%
10%
50%
80%

Housekeeping genes :
Sequence differences among them provide information on relatedness of widely divergent
groups of bacteria
Example of such are the genes encoding cytochromes
Are used for phylogenetic analysis
Can be elucidated with DNA sequencing
All statements are correct

Choose the INCORRECT statement:


GC content of closely related bacteria is similar
GC content of closely related bacteria is rather different
GC content of DNA is revealed by thermal denaturation midpoint
All statements correct
All statements Incorrect

Quantitative manifestation of microorganism pathogenicity is:


Adhesiveness
Toxicity
Invasiveness
Virulence
None of above

It is known, that between different groups of microorganisms there are a few types of co-
operations. How is the type of co-operation named, if there is strengthening of physiology
functions of members of association?
Metabiosis
Synergizm
Mutualizm
Sattelizm
Commensalizm

Which bacterial structure from listed below defends it from phagocytes?


Cellular membrane
Endospores
Capsule
Fimbriae
Pili

The chemical structure of the endotoxin is:


Peptidoglican
Protein
Lipid
Lipopolysaccharide
Lipoprotein

Microbial exotoxins transfer with blood at diphtheria. Consequently, in this case it is possible to
speak about:
Septicopyemia
Bacteriemia
Septicemia
Toxinemia
Intoxication

It was found that colon bacilla strain doesn’t have the H-antigen. What structure component is
absent in this strain’s cells?
Cell wall
Capsule
Outer membrane
Flagella
Mesosome

_______is produced by some organisms capable of destroying erythrocytes


Fibrinolysin
Hyaluronidase
Lysozyme
Hemolysin
Ig A1 proteases

Teichoic acid and lipopolysaccharides (LPS) are important components of bacterial cell walls.
Which of the following statements is true?
Teichoic acid is found only on gram (-) cell walls
Teichoic acid is also known as an endotoxin
LPS is a component of gram () cell walls
LPS is also known as an endotoxin
B and D are correct

The main cause of diarrhea and pseudomembranous colitis found in health care associated
infections is the presence of which of the following microorganisms?
Enterococcus pnemoniae
Bacillus cereus
Mycoplasma pneumoniae
Listeria monocytogenes
Clostridium difficile

Brucella is the cause of undulant fever, it is a _______ pathogen


Opportunistic
Coliform
Zoonotic
Enteric
None of the above

Fleas are carriers of :


Clostridium perfringens
E.coli
Shigella
Yersinia pestis
Smallpox

It is possible to catch _______ a skin disease, from your cat.


Influenza
A fever
AIDS
Cancer
Zoophilic dermatophytes

Which infectious diseases are NOT transmitted by dogs?


Leishmaniasis
Hydatid cyst
Plague
Zoophilic dermatophytes
None of above

A patient with diagnosis “Syphilis” was treated with antibiotics and become healthy. Then he
was infected by T. pallidum again. How is such type of infection called?
Recidivation
Reinfection
Superinfection
Secondary infection
Complication

Bacteriological examination of the feces of a 38-year-old woman, who had been ill with typhoid
fever 1.5 year ago, revealed Salmonella typhi. Name the form of the infectious process.
Disbacteriozis
Bacteria carrying
Reinfection
Superinfection
Recidivation

Malaria is a protozoal disease caused by _____ .


Trypanosoma
Toxoplasma
Plasmodium
Leishmania
Rotavirus

In what way is salmonella usually transmitted to humans?


Direct contact
Contaminated food or water
Insect bite.
Indirect contact

Which of the following two diseases are caused by viruses?


HIV/AIDS & cholera
Measles & smallpox
Malaria & cholera
Tuberculosis & HIV
Flu & stroke

Which of the following types of disease is caused when the immune system attacks the body?
Lifestyle
Disorder
Heart disease
Autoimmune
All of the Above

A man, who lived in the endemic region, suffered a three-day malaria. After moving to the non
endemic area in 1.5 years he got sick with malaria again. Which form of malaria is the most
likely in this case?
Reinfection
Relapse
Superinfection
Persistent infection
Secondary infection

Which of the contagious viruses has been eliminated from the world-wide population?
Chickenpox
Smallpox
Measles
Monkeypox.
All of these

Which type of pathogen causes malaria?


Virus
Bacteria
Protozoa
Fungi
Non above

The primary habitat of Vibrio cholerae is _____.


Intestine of humans
On water plants
Natural waters
Exoskeletons of crustaceans
Insect

_____ cause protozoal sexually transmitted disease


Entamoeba
Cryptosporidium
Trichomonas vaginalis
Treponema pallidum
Neiseria gonorrheae

During treatment with bismuth preparations a patient with syphilis developed gray spots on
his oral mucosa and nephropathy symptoms. What drug is used as an antidote to bismuth
preparations poisoning?
Nalorphine
Bemegride
Unithiol
Naloxone
Methylene blue

A 26-year-old woman with bronchitis has been administered a broad spectrum antibiotic as a
causal treatment drug. Specify this drug:
Interferon
BCG vaccine
Ambroxol
Dexamethasone
Doxycycline

Streptomycin and other aminoglycosides prevent the joining of formylmethionyl-tRNA by


bonding with the 30S ribosomal subunit. This effect leads to disruption
of the following process:
Transcription initiation in procaryotes
Transcription initiation in eucaryotes
Translation initiation in procaryotes
Translation initiation in eucaryotes
Replication initiation in procaryotes

To treat bronchitis the patient was prescribed a beta-lactam antibiotic. Its mechanism of
action is based on inhibition of murein production, which results in death of the causative
agent. Name this drug:
Bijochinol (Quinine bismuth iodide)
Ciprofloxacin
Azithromycin
Streptomycin
Penicillin G Sodium Salt

A patient with pulmonary tuberculosis is prescribed the most effective antituberculous


antibiotic. Name this drug:
Tetracycline
Streptocide
Rifampicin
Furasolidone
Bactrim (Co-trimoxazole)

A patient has been diagnosed with gonorrhea. As fluoroquinolones are the drugs of choice
for treatment of gonorrhea the patient should be prescribed with:
Furazolidone
Fluorouarcil
Sulfacarbamide (Urosulfanum)
Cefazolin
Ciprofloxacin

A patient developed pyoinflammatory process of periodontal tissues caused by activation of


the microorganisms inherent in the body, which are a part of oral mucosal microflora. What
type of infection is it?
Exogenous infection
Autoinfection
Reinfection
Superinfection
Relapse

Material obtained from a patient contains several types of microorganisms (staphylococci


and streptococci) causative of the patient’s disease. Name this type of infection:
Superinfection
Reinfection
Consecutive infection
Coinfection
Mixed infection

A patient has been diagnosed with URTI (Upper respiratory tract infection). Blood serum
contains immunoglobulin M. What stage of infection is it?
Prodromal
Incubation
Reconvalescence
Carriage
Acute

Protective function of saliva is based on several mechanisms, including the presence of


enzyme that has bactericidal action and causes lysis of complex capsular polysaccharides of
staphylococci and streptococci. Name this enzyme:
Alpha-amylase
Oligo-1,6-glucosidase
Collagenase
Lysozyme
Beta-glucuronidase

Choose antibiotic that targets bacterial protein synthesis


Norfloxacin
Rifampicin
Doxycycline
Vancomycin
Polymyxin E

Choose antibiotic that targets bacterial cytoplasmic membrane


Trimethoprim
Ciprofloxacin
Neomycin
Polymyxin B
Bacitracin

Choose antibiotic that targets bacterial cell wall


Azithromycin
Penicillin G
Sulfasoxazole
Amikacin
Tetracycline

Choose antibiotic that inhibits RNA synthesis


Streptomycin
Amphotericin B
Trimethoprim
Meropenem
Rifampicin

Choose antibiotic which inhibits DNA synthesis


Minocycline
Nalidixic acid
Ceftriaxone
Daptomycin
Sulfasoxazole

Choose antibiotic that targets folic acid synthesis


Polymyxin B
Doxycycline
Ciprofloxacin
Trimethoprim
Aztreonam
Choose antibiotic that targets 50S subunit of bacterial ribosome
Imipenem
Eperezolid
Cycloserine
Rifampin
Daptomycin

Choose antibiotic that targets 30S subunit of bacterial ribosome


Polymyxin E
Sulfasoxazole
Ertapenem
Ceftaroline
Gentamicin

Choose antibiotic that inhibits the nucleic acid synthesis


Doxycycline
Cefazolin
Norfloxacin
Polymyxin B
Ketoconazole

What agent is used to treat fungal infections?


Ertapenem
Clotrimazole
Tobramycin
Bacitracin
Penicillin G

Choose term for the transfer of resistance genes from one generation of bacteria to another by
reproduction
Horizontal gene transfer
Vertical gene transfer
Transduction
Transformation
Conjugation

Choose the term for the transfer of resistance genes through the temporary fusion of the
mating partners
Transduction
Vertical gene transfer
Transformation
Conjugation
Mutation

Choose the term for the transfer of resistance genes from bacteriophages to bacteria
Conjugation
Mutation
Transformation
Transduction
Vertical gene transfer
Choose the term for the transfer of resistance genes by receiving “free” DNA from
environment
Conjugation
Transformation
Transduction
Mutation
Vertical gene transfer
Choose the way of acquiring antibiotic resistance that is not dependent on the gene transfer
Conjugation
Transduction
Mutation
Transformation
Natural resistance

Transformation is a
transfer of resistance genes from bacteriophages
transfer of resistance genes by receiving “free” DNA from environment
transfer of resistance genes from one generation of bacteria to another
transfer of resistance genes through the temporary fusion of the mating partners
a stable heritable change of a gene leading to acquisition of resistance to particular antibiotic

Conjugation is a
a stable heritable change of a gene leading to acquisition of resistance to particular antibiotic
transfer of resistance genes from bacteriophages
transfer of resistance genes by receiving “free” DNA from environment
transfer of resistance genes from one generation of bacteria to another
transfer of resistance genes through the temporary fusion of the mating partners

Transduction is a
transfer of resistance genes from one generation of bacteria to another
a stable heritable change of a gene leading to acquisition of resistance to particular antibiotic
transfer of resistance genes from bacteriophages to bacteria
transfer of resistance genes through the temporary fusion of the mating partners
transfer of resistance genes by receiving “free” DNA from environment

Vertical gene transfer is a


transfer of resistance genes from one generation of bacteria to another
a stable heritable change of a gene leading to acquisition of resistance to particular antibiotic
transfer of resistance genes from bacteriophages to bacteria
transfer of resistance genes through the temporary fusion of the mating partners
transfer of resistance genes by receiving “free” DNA from environment

Horizontal gene transfer is a


transfer of resistance genes from bacteriophages to bacteria
transfer of resistance genes through the temporary fusion of the mating partners
transfer of resistance genes by receiving “free” DNA from environment
transfer of resistance genes from one generation of bacteria to another
answers A, B, C are correct

Choose bactericidal antibiotic from the list


Trimethoprim
Minocycline
Imipenem
Chloramphenicol
Sulfasoxazole

Which antibiotic from the list doesn’t possess bactericidal activity?


Meropenem
Ciprofloxacin
Penicillin G
Vancomycin
Spectinomycin

Choose broad-spectrum antibiotic from the list


Vancomycin
Aztreonam
Penicillin G
Bacitracin
Meropenem

Choose narrow-spectrum antibiotic from the list


Ertapenem
Tetracycline
Tobramycin
Aztreonam
Norfloxacin

Choose narrow-spectrum antibiotic from the list


Ertapenem
Tetracycline
c Penicillin
Tobramycin
Norfloxacin

Choose narrow-spectrum antibiotic from the list


Ertapenem
Tetracycline
c Bacitracin
Tobramycin
Norfloxacin

Choose broad-spectrum antibiotic from the list


Vancomycin
Aztreonam
Penicillin G
Bacitracin
Doxycycline

Choose broad-spectrum antibiotic from the list


Vancomycin
Tetracycline
Aztreonam
Penicillin G
Bacitracin

Choose antibiotic that doesn’t belong to beta-lactams


Ceftaroline
Minocycline
Penicillin
Meropenem
Aztreonam

Choose antibiotic that doesn’t belong to beta-lactams


Ceftaroline
Meropenem
Penicillin
Cycloserine
Aztreonam

Beta-lactamases confer resistance to


Azoles
Cephalosporines
Polymyxins
Macrolides
Tetracyclines

Mycoplasma are resistant to meropenem because of


Mutation leading to acquisition of resistance
Vertical transfer of resistance genes
Horizontal transfer of resistance genes
Natural resistance
Acquired resistance

Which antibiotic cannot be used against Gram-positive bacteria?


Bacitracin
Doxycycline
Rifampicin
Penicillin G
Aztreonam

What antibiotic causes the biggest number of allergic responses?


Daptomycin
Tetracycline
Penicillin
Gramicidin
Imipenem

Choose agents that inhibit fungal cell wall synthesis


Azoles
Aminoglycosides
Echinocandins
Cephalosporins
Quinolones

Choose agents that interfere with sterol synthesis in fungi


Echinocandins
Beta-lactams
Cephalosporins
Quinolones
Azoles

Which bacteria from the list are resistant to Doxycycline?


Streptococcus pneumoniae
Neisseria gonorrhoeae
Clostridium tetani
Pseudomonas aeruginosa
Escherichia coli

Erythromycin and clindamycin resistance is associated with:


Drug inactivation
Decreased Permeability
Activation of Drug pumps
Change in drug Binding site
Use of alternate Metabolic pathway

Sulfonamide and trimethoprim resistance is associated with:


Drug inactivation
Decreased Permeability
Activation of Drug pumps
Change in drug Binding site
Use of alternate Metabolic pathway

Alteration on the 50S ribosomal binding site confers resistance to:


Ceftaroline
Meropenem
Penicillin
Erythromycin
Aztreonam

Diabetes in human can be associated with the presence of


Helicobacter pylori
Coxsackievirus
Borna agent
Borrelia burgdorferi
Coronaviruses

A transition from Endospores to Vegetative Cells in the life cycle of bacteria was describes by
Robert Koch
Louis Pasteur
Ferdinand Cohn
Antoni van Leeuwenhoek
Robert Hooke

Which one of the listed microorganisms doesn’t have flagella?


Escherichia coli
Pseudomonas aeruginosa
Spirillum
Treponema pallidum
Staphylococcus aureus

Choose the CORRECT statement about the Generalized transduction


Is performed by lysogenic viruses
Random fragment of DNA is carried
A specific part of the host genome is packed
Includes integration with bacterial chromosome
Requires formation of a specific pilus

Which one of the listed microorganisms is monotrichous?


Salmonella typhi
Escherichia coli
Staphylococcus aureus
Vibrio cholerae
Treponema pallidum

Choose the CORRECT statement about Plasmids


Occur only as circular molecules
Always contain resistance genes
Can be transferred via vertical gene transfer only
Can carry transposons
Never confirm resistance against heavy metals

Choose the INCORRECT statement about the Specialized transduction


Is performed by lysogenic viruses
A specific part of the host genome is packed
Includes integration with bacterial chromosome
Random fragment of DNA is carried
Requires formation of a specific pilus

Choose the INCORRECT statement about PAI:


present in the genome of pathogenic bacteria but absent in nonpathogenic strains
mostly inserted in the backbone genome of the host strain in specific sites that are frequently
tRNA or tRNA-like genes
Mobility genes are present
GC content is usually similar to those in backbone genome genes
Sometimes include remnants of IS elements

Choose a disease that doesn’t belong to Re-emerging Infectious Diseases


Diphtheria
Malaria
Rabies
Cholera
Measles

A disproval of the concept of Spontaneous generation was made by


Ferdinand Cohn
Robert Koch
Louis Pasteur
Antoni van Leeuwenhoek
Robert Hooke
Schizophrenia can be associated with the presence of
Helicobacter pylori
Coxsackievirus
Borna disease virus
Borrelia burgdorferi
HIV

Solid media were implemented by


Ferdinand Cohn
Robert Koch
Louis Pasteur
Antoni van Leeuwenhoek
Robert Hooke

Which one of the listed microorganisms DOESN’T have flagella?


Escherichia coli
Pseudomonas aeruginosa
Spirillum
Treponema pallidum
Staphylococcus aureus

Naturally occurring genome editing systems in bacteria are called


Transposons
Insertion Sequenses
Pathogenicity Islands
Restriction Methylation Systems
CRISPR Cas system

Which one of the listed microorganisms has endoflagella?


Salmonella typhi
Escherichia coli
Vibrio cholerae
Neisseria gonorrhoeae
Treponema pallidum

Choose the CORRECT statement about Plasmids


Don’t contain their own origin of replication
Always contain resistance genes
Can occur in fungi
Can not replicate independently of the bacterial chromosome
Always are necessary for survival

Choose a structure that never is observed in bacteria


70S ribosome
Cytoskeleton
S-layer
Glycocalyx
Histons

Choose the INCORRECT statement about PAI


present in the genome of pathogenic bacteria but absent in nonpathogenic strains
mostly inserted in specific sites that are frequently tRNA or tRNA-like genes
integrases are fquently located at the beginning of the island
integrases in PAI determine the pathogenicity of host bacterium
GC content different from that of the core genome

Choose a disease that doesn’t belong to Re-emerging Infectious Diseases


Diphtheria
Malaria
Legionnaire disease
Rabies
Measles

Requires the attachment of two related species and the formation of a bridge that can transport
DNA
Recombination
Conjugation
Transformation
Transfection
Transduction

Which one of the listed microorganisms is atrichous


Salmonella typhi
Escherichia coli
Vibrio cholerae
Neisseria gonorrhoeae
Treponema pallidum

Choose the INCORRECT statement :


Having similar genotypes but living in different environment bacteria develop different
phenotype
Having similar phenotypes but living in different environment bacteria develop different
genotype
In biofilm bacteria can have different phenotype comparing to its planktonic state
Cryptic plasmid doesn’t influence the phenotype but changes the genotype of bacteria
RNA molecules are not a part of a genotype

Choose a structure that never is observed in bacteria:


Cytoskeleton
S-layer
Glycocalyx
Golgi apparatus
Nucleoid-associated proteins

In bacteria enzymes of respiration and ATP synthesis are located in


Mitochondrion
Adjacent to nucleoid
Cytoplasm
Cytoplasmic membrane
70S ribosome

Choose the INCORRECT statement about the causative agent of leprosy


Contains mycolic acid in a cell wall
Contains peptidoglican in a cell wall
Contains phospholipids in a Cytoplasmic Membrane
Contains LPS in a cell wall
Contains 70S ribosomes

Choose the CORRECT statement about archaea:


Lack cell wall
Have peptidoglican in a cell wall
Lack peptidoglican in a cell wall
Contain mycolic acid in a cell wall
Have LPS in Outer Membrane

Single layers of thousands of copies of a single protein linked together are


Polysomes
Slime layers
S-layers
Peptidoglycan molecules
Teichioic acid molecules

Diseases that emerged due to Phage Lysogenic Conversion of bacteria include


Diabetes
Gastric ulcers
Schizophrenia
Botulism
Yellow fever

Choose a disease that belongs to Re-emerging Infectious Diseases


Legionnaire disease
Diphtheria
Gastric ulcers
Ebola hemorrhagic fever
Diabetes

Choose microorganisms that were observed , discovered and described first


Bacteria
Yeasts
Molds
Viruses
All simultaneously discovered

A process when random pieces of DNA are transferred by viruses to other bacterial cells is
called
Conjugation
Transformation
Transfection
Generalized Transduction
Specialized Transduction

Which one of the listed microorganisms doesn’t have flagella?


Salmonella typhi
Escherichia coli
Vibrio cholerae
Streptococcus pyogenes
Treponema pallidum

Choose the INCORRECT statement :


Bacterial biofilms are multilayer
pH and oxygen conditions may be very different in different layers
In biofilm bacteria can have different phenotype comparing to its planktonic state
Cryptic plasmid doesn’t influence the phenotype but changes the genotype of bacteria
In biofilms bacteria communicate via lysogenic conversion

Choose a structure that never is observed in bacteria


Cytoskeleton
Endoplasmic Reticulum
S-layer
Glycocalyx
Nucleoid-associated proteins

The process of communication between bacteria is called


Conjugation
Lysogenic Conversion
Generalized Transduction
Specialized Transduction
Quorum Sensing

Choose the INcorrect statement about endotoxin:


Stimulates fever
Is toxic only in high amounts
Causes systemic effect
Is toxic in minute doses
Does not stimulate antitoxins

The diseases that emerged due to Phage Lysogenic Conversion of bacteria include
Gastric ulcers
Schizophrenia
Cryptococcosis
Diphtheria
Yellow fever

Choose the INCORRECT statement about exotoxin:


Toxic in minute amounts
Usually doesn’t stimulate fever
Is specific to a cell type or molecule
Is present in Gram positive bacteria only
Stimulates antitoxins

Choose a disease that DOES NOT belong to Re-emerging Infectious Diseases


Rabies
Diphtheria
Lyme borreliosis
Tuberculosis
Pertussis
The use of cotton for closing flasks and tubes was proposed by
Robert Koch
Ferdinand Cohn
Louis Pasteur
Antoni van Leeuwenhoek
Robert Hooke

Signs of disease DO NOT include


edema
lymphadenitis
blood pressure alterations
pain
presence of antibodies in a patient’s serum

Common for Bacterial pathogens Vibrio cholerae, Listeria monocytogenes, Campylobacter


jejuni, and enteropathogenic Escherichia coli (EPEC) as well as viral pathogens such as
norovirus and rotavirus is them being associated with
Tetanic seizures
Diarrhea
Running nose
Nosebleeding
Chest pain

Incubation period for influenza virus is


Approximately a week
Approximately 1–2 days
Approximately 5 days
Approximately 1–2 hours
Approximately 20 minutes

An enzyme allowing H. pylori is able to colonize the stomach and persist in its highly acidic
environment
Catalase
Oxydase
DNAse
Urease
Coagulase

A viruse that causes a disease that doesn’t go into a latent state after the acute infection
HSV-1
HSV-2
HV
VZV
EBV

Superinfection is
Subsequent infection by same organism in a host (after recovery).
Infection by same organism in a host before recovery.
It is a condition where due to infection at localized sites like appendix and tonsil, general effects
are produced.
Cross infection occurring in hospital.
It is one where clinical affects are not apparent
Focal infection is
Subsequent infection by same organism in a host (after recovery).
Infection by same organism in a host before recovery.
It is a condition where due to infection at localized sites like appendix and tonsil, general effects
are produced.
Cross infection occurring in hospital.
It is one where clinical affects are not apparent

Nosocomial infection is
Subsequent infection by same organism in a host (after recovery).
Infection by same organism in a host before recovery.
It is a condition where due to infection at localized sites like appendix and tonsil, general effects
are produced.
Cross infection occurring in hospital.
It is one where clinical affects are not apparent

Reinfection is
Infection by same organism in a host before recovery.
Subsequent infection by same organism in a host (after recovery).
It is a condition where due to infection at localized sites like appendix and tonsil, general effects
are produced.
Cross infection occurring in hospital.
It is one where clinical affects are not apparent

Subclinical infection is
Infection by same organism in a host before recovery.
Subsequent infection by same organism in a host (after recovery).
It is a condition where due to infection at localized sites like appendix and tonsil, general effects
are produced.
Cross infection occurring in hospital.
It is one where clinical affects are not apparent

Insects serve as a biological vector in


Dysentery
Typhoid fever
Malaria
Cholera
Syphilis

Choose a noncommunicable disease


Diphteria
Tetanus
AIDS
Influenza
Leprosy

Choose a weakly communicable disease


Diphteria
Tetanus
AIDS
Influenza
Leprosy

Choose a disease that is not zoonotic


Rabies
Plague
Anthrax
Tetanus
Salmonellosis

In Human Microbiome Project the species of microorganism living in human gastro-intestinal


tract are studied by
Pure culture isolation technique
Complex staining methods
Immunological methods
Metagenomics
Human DNA sequencing

Choose a bacteria able to survive inside the phagocyte


E. coli
S. aureus
Mycobacterium
All Gram-negative bacteria
Bacteria that lack a cell wall

Choose a bacteria able to survive inside the phagocyte


E. coli
Legionella
Protoplasts
All Gram-negative bacteria
Bacteria that naturally lack a cell wall

Krebs Cycle
Occurs only in Strict anaerobes
Occurs only in Strict aerobes
Occurs in Facultative anaerobes
Occurs only in Aerobic Respiration
Occurs only in Anaerobic Respiration

You might also like